You are on page 1of 78

GSP

GUARANTEED
SUCCESS PACKAGE

Marching Towards
THE TARGET
Solved Papers (2021-18) 0-48

HISTORY OF INDIA &


INDIAN NATIONAL MOVEMENT
ANCIENT HISTORY Later Vedic Economy 1-23
The Position of Women 1-24
Sources of Ancient Indian History Vedic Literature 1-25
The Nomenclature of India 1-3
Pre-Mauryan Age
Types of Sources 1-3
Emergence of New Religious Sects 1-26
Archaeological Sources 1-3
Literary Sources 1-5 Socio-Economic Developments 1-26
Accounts of Foreign Historians and Travellers 1-7 Jainism 1-26
Doctrines of Jainism 1-27
The Pre-Historic Age Philosophy of Jainism 1-27
Stone Age 1-8 Sects of Jainism 1-28
Mesolithic or Middle Stone Age (10000-7000 BC) 1-9
Spread of Jainism 1-28
Megalith Culture or Iron Age 1-10
Decline of Jainism 1-29
Indus Valley Civilisation Buddhism 1-29
The Discovery 1-11 Gautama Buddha 1-29
The Geographical Extent 1-11 Buddhist Councils 1-30
Features of Important Harappan Sites 1-11 The Buddhist Literature 1-31
Major Characteristics of Harappan Sites 1-14 The Decline of Buddhism 1-33
Drainage System 1-14 Other Heterodox Sects 1-34
Social Life 1-15
Political Life 1-15 Mahajanapadas
Economic Life 1-15 Emergence of the Mahajanapadas 1-34
Domestication of Animals 1-15 List of the Mahajanapadas and their Capitals 1-35
Trade, Crafts, Technology and Weight and Measures 1-16 The Republics of the 6th Century 1-35
Art and Architecture 1-17 The Pre-Mauryan Economic System 1-36
Decline and End of the Harappan Civilisation 1-18 Iranian or Persian Invasion 1-37
The Vedic Age Alexander’s Invasion (The Greek Invasion) 1-37
The Aryans 1-19 The Mauryan Empire
Early Vedic Age (1500-1000 BC) 1-19 Origin of the Mauryan Rule 1-38
The Rigvedic Polity (Tribal Polity) 1-20 Literary Sources for the Mauryan Period 1-38
Later Vedic Age (1000-600 BC) 1-22 Chandragupta Maurya (321-298 BC) 1-39

06
GSP
GUARANTEED
SUCCESS PACKAGE

Bindusara (298-273 BC) 1-39 The Rajputs in Northern India 1-67


Ashoka (273-232 BC) 1-39 Bhima II (1178-1239 AD) 1-69
The Later Mauryas 1-41 Practice Questions: Topicwise, Sectionwise,
The Mauryan Administration 1-41 UPSC Questions (2017-2005) 1-70 — 1-83
Mauryan Army 1-42
Decline of the Mauryan Empire 1-44 MEDIEVAL HISTORY
Post-Mauryan Age
Arrival of Arabs and Turks in India
Northern and Eastern India 1-44
Arab Conquest of the Sindh 1-84
The Sunga Dynasty (185-75 BC) 1-44
Turk Conquest 1-85
Hunas 1-44
Mohammed of Ghur (1175-1206) 1-86
The Kanva Dynasty (75-30 BC) 1-45
Chedi Dynasty of Kalinga Chetas (Chedis) 1-45 The Delhi Sultanate
The Satavahana Dynasty (230 BC-220 AD) 1-45 Foundation of the Muslim State 1-86
Indo-Greeks 1-45 The Slave Dynasty (1206-90 AD) 1-87
The Shakas (90 BC-100 AD) 1-46 The Khilji Dynasty (1290-1320 AD) 1-88
The Parthians (247 BC-224 AD) 1-46 Market Control and Economic Reforms 1-90
The Kushanas (45-73 AD) 1-46 The Tughlaq Dynasty (1320-1414 AD) 1-91
Impact of Central Asian Contact 1-46 The Sayyid Dynasty (1414-1451 AD) 1-93
Development of Art (200 BC-300 AD) 1-48 The Lodhi Dynasty (1451-1526 AD) 1-93
Geographical Discoveries in Post-Mauryan Period 1-50 Administration in the Sultanate Period 1-94
Economy in the Sultanate Period 1-95
South India
Relation Between the Hindus and the Muslims 1-96
The Sangam Age 1-50
Development of Music 1-97
Sangam Age Sources 1-50
The Early Kingdoms 1-51 The Emergence of Provincial Kingdoms
Sangam Administration 1-52 Bengal 1-97
The Chalukyas (543-753 AD) 1-54 Orissa 1-98
The Imperial Cholas 1-55 Kashmir 1-98
Chola Administration 1-56 Gujarat 1-98
The Cheras 1-57 Malwa 1-99
Jaunpur 1-99
The Gupta Empire (320-550 AD)
Marwar 1-99
Rise of the Guptas 1-57
Amber or Amer 1-99
Sources of the Gupta Rule 1-57
The Dynastic History of Guptas 1-58 The Vijayanagara and Bahamani Empire
Chandragupta II (380-412 AD) 1-58 Origin of the Empire 1-100
Kumaragupta (415-455 AD) 1-58 Sangama Dynasty (1336-1485 AD) 1-100
The Gupta Administration 1-59 Decline of the Sangama Dynasty 1-101
Importance of Land in Gupta Period 1-60 The Saluva Dynasty (1485-1505 AD) 1-101
Gupta Paintings 1-62 The Tuluva Dynasty (1505-1570 AD) 1-101
Developments in Science and Technology 1-62 The Aravidu Dynasty (1570-1652 AD) 1-102
Decline of the Guptas 1-63 Vijayanagara Administration 1-102
Vijayanagara Society 1-103
Post-Gupta Era/Early Medieval
Bahmani Kingdom 1-104
North India 1-63
Independent Kingdoms of the Deccan 1-104
The Gaudas 1-64
Art and Architecture 1-105
Kannauj After Harsha 1-65
The Rashtrakutas in Deccan 1-65 Bhakti and Sufi Movements
The Palas of Bengal 1-66 Bhakti Movement 1-106
The Senas of Bengal 1-67 Some Important Philosophers 1-107

07
GSP
GUARANTEED
SUCCESS PACKAGE

Nirguna Saints 1-107 The Dutch 1-151


Saguna Saints 1-107 The French 1-151
Maharashtra Saints 1-108 The British Occupation of Bengal 1-153
Sufism in India 1-109 The Nawabs of Bengal 1-153
Important Sufi Saints 1-110 Battle of Buxar (1764) 1-154
The Mughal Empire Anglo-Mysore Conflict 1-156
Mughal Rule 1-112 The Anglo-Maratha Wars 1-157
Babur (1526-30 AD) 1-112 The Sikhs 1-158
Humayun (1530-56 AD) 1-112 Anglo-Tibetan Relations 1-159
Sher Shah’s Administration 1-113 The British Administrative Structure and Economic Policies
Akbar (1556-1605 AD) 1-114 The Structure of the Government 1-160
Jahangir (1605-27 AD) 1-117 Acts and Amendments 1-161
Prince Salim’s Rebellion 1-117 Civil Services and Judiciary 1-162
Shahjahan (1627-58 AD) 1-118 Different Dimensions of Exploitation During British Rule1-163
Aurangzeb (1658-1707 AD) 1-119 Transport and Communication in India 1-165
Sikhs Revolt 1-120 Peasant and Tribal Movements
Policies of Aurangzeb 1-121 Peasant Uprisings 1-165
Mughal Administration 1-121 Pabna Rebellion (1873-85) 1-166
Provincial Administration 1-121 The Gadkari Revolt (1844) 1-167
Mansabdari System 1-122 All India Kisan Sabha 1-168
The Jagir System 1-122 Tribal Rebellions 1-169
Methods of Land Revenue Assessment 1-122 Important Uprisings in India 1-170
Jagirdari Crisis 1-124 Failure of the Tribal Uprisings 1-171
Categories of Agriculturists 1-124
Military Organisation of the Mughals 1-126 Labour Movements in India
Organisations Related to Labour Movements 1-172
The Maratha Empire Unity Among Trade Unions 1-172
The Marathas 1-127 Post War Period 1-173
Shivaji (1627-80 AD) 1-127
Maratha Administration 1-129 Socio-Religious Reform Movements in India
Maratha Power Under the Peshwas 1-130 Causes for Socio-Religious Reform Movements 1-174
Baji Rao I (1720-40 AD) 1-130 Reforms and their Leaders 1-174
Maratha Confederacy 1-131 The Brahmo Samaj 1-175
The Young Bengal Movement 1-175
Practice Questions: Topicwise, Sectionwise, The Prarthana Samaj 1-176
UPSC Questions (2017-2005) 1-132 — 1-141 The Arya Samaj 1-176
The Theosophical Society 1-177
MODERN HISTORY Other Significant Hindu Reform Movements 1-178
Later Mughals and the Rise of Autonomous States Reform Movements among Muslims 1-179
Bahadur Shah I (1707-12) 1-142 Sikh Reform Movements 1-179
Causes for the Decline of the Mughal Empire 1-143 Muslim Socio-Religious Movements and Organisation 1-181
Rise of Autonomous States 1-144 Growth of Press and Education in India
Tipu Sultan 1-145 Press Laws and Regulations in British India 1-182
The Jats 1-146 The Vernacular Press Act, 1878 1-182
Social and Economic Conditions in the 18th Century 1-147 The Press Enquiry Committee 1-183
Life in the Early 18th Century 1-147 Development of Modern Education 1-184
European Penetration and the British Conquest of India Wood’s Despatch, 1854 1-185
The Advent of the Europeans 1-148 Hunter Commission or Indian Educational
The Portuguese 1-148 Commission, 1882 1-185
The English 1-149 Indian Universities Act, 1904 1-185

08
GSP
GUARANTEED
SUCCESS PACKAGE

Saddler Commission, 1917-19 1-186 Governor-Generals and Viceroys of India


Hartog Committee, 1929 1-186 Some Famous Governor-Generals of the Company 1-230
Wardha Scheme of Education, 1937 1-186 Governor-Generals of India 1-233
Sergeant Plan, 1944 1-186 Viceroys of India 1-236
Kothari Education Commission, 1964 1-187 Important National Leaders 1-240
University Grants Commission (UGC), 1953 1-187 Timeline 1-247
The Revolt of 1857 Practice Questions: Topicwise, Sectionwise,
Causes of the Revolt 1-188 UPSC Questions (2017-2005) 1-251 — 1-281
Outside Influence on the Revolt of 1857 1-189
Impact of the Revolt/Mutiny 1-190 INDIAN ART AND CULTURE
The Government of India Act, 1858 1-190 Introduction: To Indian Culture
Reorganisation of the Army 1-190 Culture and Civilisation 1-282
Significance of the Revolt 1-191 A Preview of Indian Culture 1-283
Struggle for Independence First Phase (1885-1915) Characteristics of the Indian Culture 1-284
Rise and Growth of Nationalism 1-192 Languages and Literature
Formation of Indian National Congress (INC), 1885 1-195 Indian Language Families 1-285
Moderates and the Era of Early Nationalists (1885-1905) 1-196 Other Indian Languages and Indian Literature 1-289
Rise of Extremism in Indian Politics 1-197 Literature and National Ideology 1-290
Partition of Bengal, 1905 1-199 Indian Literature in English Language 1-292
Swadeshi Movement, 1905 1-199
Surat Split, 1907 1-200 Religion and Philosophy
Foundation of Muslim League, 1906 1-200 Proto-Historic Religion 1-292
Revolutionary Activities (First Phase) 1-201 Ancient Indian Religions 1-292
Ghadar Party (1913) 1-202 Theistic Religions 1-295
The Schools of Indian Philosophy 1-296
Struggle for Independence Second Phase (1915-1935)
Lucknow Pact, 1916 1-203 Art and Architecture
The Gandhian Era in India 1-204 The Indus Valley Period 1-299
The Impact of World War I 1-206 Mauryan Art 1-300
Jallianwala Bagh Massacre, 1919 1-207 Post-Mauryan Art 1-301
Khilafat Movement, 1919 1-207 The Gupta Art 1-302
Non-Cooperation Movement, 1920 1-208 Indo-Islamic Art and Architecture 1-308
Civil Disobedience Movement (First Phase) 1-212 Mughal Architecture 1-309
Communal Award, 1932 1-214 Post- Mughal Architecture 1-310
The Poona Pact, 1932 1-215 European Influence on Indian Art 1-311
Struggle for Independence Third Phase (1935-1947) Indian Painting
Government of India Act, 1935 1-216 Paintings 1-313
Contesting Provincial Elections 1-217 Cave Paintings 1-313
Cripps Mission, 1942 1-218 Miniature Paintings 1-314
Quit India Movement, August, 1942 1-219 Rajput Paintings 1-316
Rajagopalachari Formula (March, 1944) 1-221 Pahari Paintings 1-316
Cabinet Mission Plan (March, 1946) 1-223 Mysore Paintings 1-316
Partition of India 1-226 Tanjore Paintings 1-317
Integration of Princely States after Independence 1-226 Kalamkari Art 1-318
Leftist Movement Music, Dance and Drama
Communist Ideology: The Rise and Growth of Development of Music in India 1-319
the Left in India 1-228 Famous Classical Singers of India 1-323
Formation of the Congress Socialist Party 1-229 Emergence of Dance in India 1-323

09
GSP
GUARANTEED
SUCCESS PACKAGE

Classical Dances 1-324 Classification of Crafts 1-340


Folk Dances of India 1-327 Kinds of Handicrafts in India 1-340
Drama and Theatre 1-329
Modern Indian Theatre 1-331 Indian Cinema
Puppetry 1-331 The Era of Silent Films (1913-31) 1-341
Pre-Independence Talkies (1931-47) 1-342
Festivals and Fairs
Post-Independence Films (1947) (Till Today) 1-342
Festivals and Fairs 1-333
Statewise Fairs and Festivals of India 1-335 Various Types of Indian Cinema 1-342

Crafts Practice Questions: Topicwise, Sectionwise,


Crafts of India 1-339 UPSC Questions (2017-2006) 1-343 — 1-352
Crafts in Various Phases 1-339

INDIA & WORLD GEOGRAPHY


WORLD GEOGRAPHY Rainfall 2-40
Air Masses 2-41
The Universe Fronts 2-42
Study of the Universe 2-3 Cyclone 2-43
Solar System 2-5 World Climatic Types and Their Distribution 2-44
The Planets 2-5
Hydrosphere
The Earth Hydrological Cycle 2-51
Origin of the Earth 2-9 The Oceans and Seas 2-52
Motions of the Earth 2-10 Temperature and Salinity of Oceans 2-53
Time 2-13 Ocean Currents 2-55
International Date Line 2-13 Coral Reefs 2-57
Moon and Tides 2-14 Marine Resources 2-58
Geological Time Scale 2-15 Various Ocean Development Programmes in India 2-61
Lithosphere Government Institutions Regarding Research and
Interior of the Earth 2-16 Development in Ocean 2-62
Earthquakes 2-17 Rivers 2-62
Volcanism 2-18 Lakes 2-63
Rocks 2-20 Human Geography : World
Major Theories for Evolution of Landforms 2-21 Human Race 2-65
Forces and Associated Landforms 2-23 Ethnic Groups of the World 2-65
Mountains, Plateaus and Plains 2-27 World Population 2-66
Soil 2-30 Human Development 2-70
Atmosphere Migration 2-71
Composition of Atmosphere 2-32 Settlement 2-73
Insolation and Temperature 2-34 Natural Resources of the World
Atmospheric Pressure 2-36 Land Resources 2-74
Winds 2-37 Forest Resources 2-74
Humidity 2-39 Water Resources 2-74
Clouds 2-39 Minerals Resources 2-75
Precipitation 2-40 Energy Resources 2-77

10
INDIAN
ART & CULTURE
INTRODUCTION :
TO INDIAN CULTURE
As one perceives, culture can be described as a way of life. The
food we eat, the clothes we wear, the language that we speak
and the almighty whom we worship, are all aspects of culture.
Thus, culture is the embodiment of the way in which we think
and do things. At the same time, it is also something that we
have inherited as members of the society.
So, whatever a human being learns or inherits as members of
social groups can be termed as culture. For instance, art, music,
literature, architecture, sculpture, philosophy, religion and
science can be seen as aspects of culture. Also, it includes the
traditions, customs, festivals and certain ways of living.
According to Herskovits “Culture can be defined as part of our
environment created by human being.” Culture has two
distinctive components, namely–material and non-material.
Material culture consists of objects that are related to the
material aspects of our life such as our attire, food, household
goods etc.
Culture, representing a set of shared Non-material culture refers to ideas, thoughts, beliefs etc.
attitudes, values and practices, plays Culture varies from place to place and country to country. So,
a vital role in the development of a the people of any country are characterised by their distinctive
nation. India is diverse attested well cultural traditions.
by the plurality of its culture. India’s
cultural history is largely a result of CULTURE AND CIVILISATION
the customs, traditions or rituals so The words culture and civilisation are synonymous, but
absorbed by the early invaders or culture is bit different from civilisation in the sense that where
immigrants. This comingling or culture is the higher level of inner refinement within us, better
ways of living—socially or politically and better utilisation of
fusion of different cultures has thus
nature, is termed as civilisation. Culture and civilisation can be
contributed to what we have today. differentiated in one more way, as we can define civilisation in
physical aspects like buildings, city architecture etc whereas
culture is defined in non-physical terms e.g. music, tradition
etc. So, we define Harappan era as Harappan civilisation where
as Vedic period is defined as Vedic culture.

CULTURE AND HERITAGE


Culture is transmitted and carried forward from one generation
to the next generation. The culture inherited from the
predecessors is called Heritage. They are cherished, protected
and maintained with unbroken continuity and felt proud of e.g.
the Taj Mahal, Brihadeswara temple at Tanjore etc. Besides
the architectural creations, monuments, material artefacts,
intellectual achievements, philosophies, treasures of
ART AND CULTURE 1-283

knowledge, scientific inventions and discoveries also constitute the part


of Heritage. Culture is liable to change but heritage is not. Individuals
THE PERIOD OF STAGNATION
belonging to a culture or a particular group may acquire or borrow NORTH INDIA
certain cultural traits of other communities or cultures but their After the decline of Gupta empire, India saw a
belongingness to a particular culture remains unchanged. So, heritage is period of political unstability and during this
complimentary to the development of civilisation. period adventurous warlike tribes i.e. the Shakas,
the Hunas and the Gujjars spread throughout
India.
A PREVIEW OF INDIAN CULTURE They had already adopted the Hindu culture and
religion after their arrival and now claimed to be
CULTURE OF THE INDUS VALLEY the descendants of the old Kshatriyas and began
Before the excavation of the Indus Valley Civilisation, Indian culture to call themselves Rajputs. They revived the
was regarded as the youngest among the Ancient cultures. The Hindu society and created a stir and movement in
archaeological investigations of Harappa and Mohenjo-daro established its stagnant, intellectual and cultural life. Their
that civilisation had attained maturity around 5000 years ago. The courts became the centre of art, architecture,
excavations provide ample evidences that their religious beliefs and literature, poetry and drama.
practices are to some extent reflected in Hinduism. Yoga, worshipping
But the Rajputs were so obsessed with tribal
sacred animals and trees, importance of Mother Goddess etc are some of
consciousness that they seemed to have lost even
the aspects of Indus Valley Culture, which were passed on to later ages
the sense of national unity. The very idea of
and became vital part of Indian culture.
nationhood had completely disappeared. Rajput is
a broad term which involve Kshatriya clans of
DRAVIDIAN AND VEDIC CULTURE ancient India, descendants of invaders like
The Dravidian culture in South India had reached a high level of Shaka, Kushana etc and local tribes who
development in around 2000 BC. Archaeological findings proved a long established themselves as rulers.
period of commercial and cultural exchange between the Indus
valley and Dravidian cultures. Thus in form of Dravidian culture SOUTH INDIA
the continuity of Indian culture was maintained after decline of Indus South India remained free from the advancement
Valley Civilisation. Around 1000 BC, the Vedic culture which was earlier of civilisation and from any adverse effects of
simple, reached a stage of high development and changed considerably. foreign invasions. Favourable conditions in the
South preserved intellectual life from stagnation,
Now the Aryan mind gave a philosophical depth to the idea of unity
unlike the North. This period is marked by a
which had already begun in the Rigveda and the Vedic culture made the
remarkable movement and activity in religious
old Indian traditions (which had been taken mostly from the Dravidian
thoughts. In the 7th century, two orders of the
culture) as an integral part of its religion. So, a new common religion
Shaivite and Vaishnavite saints had translated
was developed, known as the Vedic Hindu Religion, which served as
the Puranas into Tamil verses, so to propagate a
the very foundation of a new culture known as the Vedic Hindu
cult of love and devotion which was later called
Culture.
Bhakti.
HETERODOX CULTURE The contacts of Muslim Arabs with South India
had already begun in the later part of the
After 400 years, in the 6th century BC there was a period of religious
preceding period. But the Muslim influence on
unrest. This period saw the rise of two great religions—Buddhism and
Indian culture in the South and West was
Jainism. During the later centuries, six schools of Hindu Philosophy
negligible. It was in the North, (where the long
emerged as new currents of thought in religious and philosophical
series of invasions by the Muslim Turks in the
thought. Buddhism and Jainism acquired much prominence and
10th century continued till the establishment of
acceptance amongst the people of the time. Not only did both contributed
the Delhi Sultanate) that the Muslim culture had
to a ritual and superstition free society, but also, to a great extent, led to
a profound and direct influence on every sphere
a great cohesion and taught how to rise above the then prevalent varna of life.
hierarchies.

THE PERIOD OF EXCELLENCE CULTURAL SYNTHESIS


Though the Delhi Sultanate united India
In the Mauryan period, though religion continued to dominate the politically; especially Northern India; the Hindus
society, the process of differentiation amongst the various spheres of life could not regard it as their own state because the
became more pronounced. Science, Literature and Fine Art were now rulers in general made no positive attempt to
free to move outside the narrow circle of religious subjects. Under the bridge the gulf between themselves and their
patronage of the Guptas, secular literature made remarkable progress. Hindu subjects. But the relationship between the
Great scholars of this period exercised a direct influence on the scientific common Muslims with the Hindus improved
thoughts of Arab countries and indirectly on Europe. considerably long before because of the historic
mediating role played by the Muslim Sufis and
1-284 GSP General Studies (Paper 1)

Hindu saints of the Bhakti school. The Hindus found Sufi ideas very civilisation has gone hand in hand. In fact, a
similar to those of the Vedantic philosophy and were naturally remarkable feature of Indian culture is that
attracted by them. But the greatest attraction for the lower castes of alongwith a continuity, it has also kept on
Hindus was the social organisation of Islam which was founded on the changing, with a continuance of the basic spirit of
basis of equality and fraternity. Another great force which created a our culture.
general atmosphere of religious harmony between the Hindus and the
Muslims was the Bhakti Movement. They could not merge the
VARIETY AND UNITY
currents of Hinduism and Islam on the surface, but they showed that A large number of languages and dialects are
the springs which freed them, did meet somewhere below it. spoken in our country, which has led to the growth
of a great variety of literature. The second
ART AND ARCHITECTURAL SYNTHESIS important reason for the variety in our culture is
the intermingling among various ethnic groups.
In aesthetic consciousness, which is deeply influenced by the physical
Since time immemorial, people from far and near
environment, Hindus and Muslims came still closer to each other.
have been coming and settling here.
Indian music captured the heart of Muslims, though paintings were
discouraged by few rulers. In the very 1st century of the Delhi Secular Outlook The secular character of Indian
Sultanate, a Hindu-Muslim style of Architecture had come into culture also happens to be the result of the
being which was adopted in the 14th century with various degrees of intermingling of the people belonging to diverse
modifications by the Muslim kings of Bengal, Gujarat and the Deccan cultural groups over a long period of time. There
and also by the Hindu rajas of Bundelkhand and Rajasthan. According have been instances of occasional conflicts
to Fergusson, the design of Jama Masjid in Ajmer had been taken though, but by and large, people have lived
from the Jaina temple of Mount Abu. together peacefully for centuries.
Universalism India has had a universal outlook
TRANSITION PHASE and it has been promoting the message of peace
and harmony to the entire world.
In the 19th century, the sub-continent was exposed to the full force of
Western influence. It stimulated every sphere of life of the Indian MATERIALISTIC AND SPIRITUALISTIC CULTURE
people. The soul of India responded deeply to the call of the West, for it Spiritual development of a race or nation in the
corresponded closely to what was already there, but had grown inert. field of mind, interest, conduct, thought, art, skill
The highway which took India from her medieval to modern phase is and civilisation takes place in the country. India is
marked by convenient milestones in the form of great personalities. popularly known to be a land of spirituality
There was a glorious outburst of the Indian renaissance and the particularly to the West. However, Indian history
succession of religious teachers who discovered the ancient truth in a from ancient times to the present day shows that
new form, aspect by aspect and layer by layer.The architecture of the developments of materialistic and non-
India’s destiny evolved through the integral philosophy of the modern materialistic culture have been going on
Indian culture during the 1st and a half centuries. simultaneously.

CHARACTERISTICS OF THE INDIAN CULTURE CONTINUITY OF THE INDIAN


Indian culture has as many colours as life and includes intellectual CULTURE
and social aspects of any human being. It also takes account of the
To prove the continuity and fundamental similarity
aesthetic instinct as well as the spiritual impulses of human beings. It
of Indian culture over a period of thousands of
has also, in effect, an appeal to the subconscious as a force, making for
years, we can observe some cultural aspects of the
the formation of character.
Indus Valley Civilisation prevalent in the present
We see people around us speaking different languages, having times.
different religions and practising different rituals. We also observe ˜
Female Deities were an important part of the
diversities in their food habits and dressing patterns. Besides, there is religious beliefs and worship of the people of the
a myriad of dance forms and music in our country. But within all these Indus valley. Even now, we have several female
diversities, there is an underlying unity, which acts as a cementing deities with their worship forming a unique aspect
force. This unity in diversity is the basic character of Indian culture. of Indian culture.
The intermingling of people has been steadily taking place in India ˜
Swastika and the Phallic forms were important
over the centuries. A number of people of different racial stocks, ethnic symbols in the Indus valley. We still have the
backgrounds and religious beliefs have settled down here. Swastika and the Shiva Lingam as important
CONTINUITY AND CHANGE constituents of our culture.
Many great cultures have developed in different countries and regions
˜
Sindoor (a cosmetic powder applied by women
of the world. Many of them have perished or have been replaced by along the parting of their hair) has been seen even
other cultures. However, Indian culture has had an enduring in the statues of the Indus valley.
character. Despite major changes and upheavals, significant threads ˜
There are evidences that even the people of Indus
of continuity can be traced throughout the course of Indian history valley practiced certain forms of Yoga.
right up to the present day. The continuity and change in our
ART AND CULTURE 1-285

SANSKRIT
LANGUAGES AND The Sanskrit language happens to be the mother of many Indian
languages. The Vedas, Upanishadas, Puranas and Dharmasutras
LITERATURE are all written in Sanskrit. Sanskrit is the most ancient language of
our country. The literature in Sanskrit is vast, beginning with the
most ancient thought embodied in the Rigveda, the oldest literary
Richness of any culture can be seen and heritage of mankind.
appreciated but when it comes to language It is noteworthy that Zend Avesta is believed to be written initially
and literature, it is to be read and in Sanskrit. The great grammarian Panini, analysed Sanskrit and
comprehended. Language is a medium its word formation in his unrivalled descriptive grammar
Ashtadhyayi.
through which one express thoughts while
literature is a mirror that reflects ideas and THE VEDAS
The Vedas are the earliest known literature of India. The Vedas
philosophies which govern society. were written in Sanskrit and were handed down orally from one
generation to the other. To be able to keep such a vast literary
wealth as the Vedas intact, when the art of writing was not there
INDIAN LANGUAGE FAMILIES and there was a paucity of writing material is unprecedented in
world history.
In India, many major languages exist along with
other minor languages usually spoken by a small There are four Vedas, namely, the Rigveda, Yajurveda, Samaveda
number of people. The Indian languages can be and Atharvaveda. Each Veda consists of the Brahmanas, the
divided into the following four families Aranyakas and the Upanishadas. The Rigveda, Samaveda and the
1. Indo-Aryan Speakers of languages from this Yajurveda are collectively known an Trayi. In later years, the
family account for around 70% of the population Atharvaveda was incorporated in this group.
of the country. Most of the languages in this STRUCTURE OF THE VEDAS
family are derived from Sanskrit. Hindi, Bengali, Each Veda consists of four parts: the Samhitas (hymns), the
Punjabi, Gujarati, Rajasthani, Assamese etc are Brahmanas (rituals), the Aranyakas (theologies) and the
part of this family. Upanishadas (philosophies). The collection of mantras or hymns is
2. Dravidian This is the second largest family of called the Samhita. The Brahmanas are ritualistic texts and
languages in India and consists of languages include precepts and religious duties. Each Veda has several
mostly spoken in Southern India like Tamil, Brahmanas attached to it.
Kannada, Malayalam and Telugu. The Dravidian
languages are older than the Indo-Aryan The Aranyakas (forest texts) intend to serve as objects of meditation
languages and many of them have classical for ascetics, who live in the forests and deal with mysticism and
language status. symbolism. The Upanishadas form the concluding portions of the
Veda and therefore are called the Vedanta or the end of Veda and
3. Austric This is the oldest and most indigenous
contain the essence of Vedic teachings. The Upanishadas and the
language in the language family of India. Its
Aranyakas are the concluding portions of the Brahmanas, which
speakers are concentrated in the hilly and tribal
discuss philosophical problems.
areas of Central and Eastern India. Santhali,
Nicobarese, Khond etc are the examples of SANSKRIT LITERATURE
languages from this family. We also have a large body of books dealing with various sciences, law,
4. Sino-Indian This includes languages spoken in medicine and grammar. To this class belong the law books called the
the North-East and Northern parts of India. It Dharmasutras and Smritis, together known as Dharmashastras.
includes Naga, Bodo, Tibetan, Ladakhi, Karbi etc. These books are as under
Apart from this, a fifth family of languages spoken in ˜
Gautama’s Dharmasutras were compiled between 500 and 200 BC.
South Andaman is Ongan, which includes the two These lay down duties for different Varnas as well as for the kings
languages of Onge and Jarawa. The Great and their officials. They prescribe the rules according to
Andamanese is another language family, which is which property had to be held, sold and inherited. They also
almost extinct. The Nihali language spoken by prescribe punishments for persons guilty of assault, murder and
around 2000 people in Madhya Pradesh and adultery.
Rajasthan is an isolated language, not a part of any ˜
The Manusmriti was written in narrative form, in which a dialogue
family. takes place between Manu’s disciple, Bhrigu and an audience of his
The Sidi language, spoken by Sidi people in Gujarat own students.
till the middle of the 20th century, is not extinct, and ˜
This book projects an ideal society and ideal human conduct as the
was derived from Swahili and also constituted to be a basis to establish an orderly society and diverse centered life. It
part of the Niger-Congo language family. should be noted that Manusmriti is just one of the various Smritis
1-286 GSP General Studies (Paper 1)

written in ancient India, which was Among the important Jaina


given prominence by Europeans to scholars, reference may be made
DRAVIDIAN LANGUAGES
highlight the negative aspects of varna to Haribhadra Suri (8th century The four Dravidian languages Tamil,
system of ancient Indian society. AD) and Hemchandra Suri Telugu, Kannada and Malayalam
(12th century AD). Jainism developed their own literature. Tamil
˜
Kautilya’s Arthashastra is an
helped in the growth of a rich being the oldest of these langauges began
important treatise belonging to the
literature comprising of poetry, to be written earlier, leading to the
Mauryan times. It reflects the state of
philosophy and grammar. These production of the Sangam literature the
society and economy at that time and
works contain many passages, oldest literature in Tamil.
provides rich material for the study of
ancient Indian polity and economy. which help us in reconstructing TAMIL OR SANGAM LITERATURE
˜
Books written on Astronomy by the political history of Eastern There are about 30000 lines of poetry,
Varahamihira and Aryabhatta and Uttar Pradesh and Bihar. The which are arranged in eight anthologies
on Astrology by Lagdhacharya had all Jain texts refer repeatedly to called Ettuttokoi. There are two main
achieved prominence. No other work can trade and traders. groups the Patinenkil Kanakku (the
compete with Varahamihir’s Brihat CANONICAL AND eighteen lower collections) and Pattupattu
Sawitha, Aryabhattiya and Vedanga NON-CANONICAL LITERATURE (the ten songs). The former is generally
Jyotisha of Lagadhmuni. assumed to be older than the latter and
CANONICAL LITERATURE
˜
Books on medicine were written by considered to be of more historical
These are linked to the Buddha. significance.
Charaka and on surgery by Sushruta.
The canonical literature is best
Madhava wrote a book on Pathology The Sangam literature is a collection of
represented by the tripitakas i.e.
Madhavnidian. long and short poems composed by various
the three baskets Vinaya Pitaka,
Sutta Pitaka and Abhidhamma poets in praise of numerous heroes and
Post-Medieval Period of Pitaka. The Vinaya Pitaka deals heroines. They are secular in nature and
Sanskrit Literature with rules and regulations of daily are of a very high quality. Three such
The Post-Medieval period in Northern India life. Sangams were held. The poems collected
saw the rise of Sanskrit literature in Kashmir. in the first Sangam have been lost. In the
The Sutta Pitaka contains second Sangam, about 2000 poems have
Somadeva’s Kathasaritsagar and Kalhana’s
Rajatarangini are of historical importance. It dialogues and discourses on been collected. The contributions of Tamil
gives a vivid account of the Kings of morality and Dharma. The saints like Thiruvalluvar who wrote
Kashmir. Abhidhamma Pitaka deals with Kural, which has been translated into
Gita Govinda of Jayadeva is the finest poem philosophy and metaphysics. It many languages are noteworthy.
of Sanskrit literature of this period, besides includes discourses on various
the numerous works on different aspects of subjects such as Ethics, TELUGU LITERATURE
art and architecture, sculpture, iconography Psychology, theories of knowledge The Vijayanagara period was the golden
and related fields. and metaphysical problems. age of the Telugu literature. Nachana
NON-CANONICAL LITERATURE Somanatha, a court poet of Bukka I,
PALI LITERATURE produced a poetical work titled
These are commentaries on the
Uttaraharivamsamu.
Pali is an Indo-Aryan language, used for canonical texts. The non-canonical
composing the earliest Buddhist literature is best represented by Krishnadevaraya (1509-1529), the
scriptures. Pali literature is mainly the Jatakas. The Jatakas are the greatest of the Vijayanagara Emperors,
concerned with Theravada Buddhism. most interesting stories of the was a poet of great merit. His work
previous births of the Buddha. It Amukta Malyada is regarded as an
BUDDHIST AND JAINA LITERATURE was believed that before he was excellent prabandha in Telugu literature.
The religious books of the Jains and the finally born as Gautama, the There were Eight Telugu literary
Buddhists mostly refer to historical Buddha practising Dharma, luminaries, popularly known as the
persons or incidents. The earliest passed through more than 550 Ashtadiggajas.
Buddhist works were written in Pali, births, in many cases even in the Dhurjati, a devotee of Shiva, composed
which was spoken in Magadha and form of animals. two poetical works of great merit known
Southern Bihar. The Buddhist works can Each story of his birth is called a as Kalahasteeswara Mahatmayam and
be divided into canonical and Jataka. The Jatakas throw Kalahasteeswara Satakam.
non-canonical. The Jaina texts were invaluable light on the social and
written in Prakrit and were finally Ramakrishna was the author of
economic conditions ranging from Panduranga Mahatmayam, which was
compiled in the 6th century AD in Valabhi the 6th century BC to the 2nd
in Gujarat. The important works are considered one of the greatest poetical
century BC. They also make works of Telugu literature.
known as Angas, Upangas, Prakirnas, incidental references to political
Chhedab Sutras and Malasutras. Ramarajabhushana was the author of
events in the age of the Buddha. Vasucharitram. He was also known
as Bhattumurti.
ART AND CULTURE 1-287

KANNADA LITERATURE Bhojpuri got its name from His Sare Jahan Se Achha Hindustan
Kannada is spoken in Karnataka. Bhojpur region in Bihar and is Hamara is sung and played at many of
Kannada language developed fully after spoken in Northern Bihar and the national celebrations in India.
the 10th century AD. The earliest known regions of Eastern Uttar Pradesh Urdu was patronised by the Nawabs of
literary work in Kannada is neighbouring Bihar. That form of Lucknow, who held symposiums in this
Kavirajamang written by the Hindi which is spoken in Meerut, language. Slowly, it became quite
Rashtrakuta King Nripatunga Ambala, Delhi is known as Khari popular. Pakistan has adopted Urdu as
Amoghavarsha I. Pampa, known as the Boli and there are many conflicting its State language.
Father of Kannada literature wrote his views about the origin of term Khari
PERSIAN LITERATURE
great poetic works Adi Purana and Boli.
In the medieval period, Persian was
Vïkramarjiva Vijaya in the 10th century World class literature is written in adopted as the court language of the
AD. above forms of Hindi language e.g. Mughals. Several historical accounts,
Madhava wrote Dharmanathapurana on Sursagar is written by Surdas administrative manuals and allied
the 15th Tirthankara. Another Jain in Braj Bhasha, similarly literature in this language have come
scholar, Uritta Vilasa, wrote Dharma Ramcharitmanas whose popularity down to us. The Mughal rulers were
Parikshe. The other eminent poet is the is unmatched by any other literary great patrons of learning and
great Sarvajna, popularly known as the work in world’s history is written in literature. Babur wrote his Tuzuk
people’s poet. His aphoristic tripadi Avadhi by Goswami Tulsidas. (autobiography) in Turkish language,
(three-lined) compositions serve as a Similarly in Maithili, which is but his grandson Akbar got it
source of wisdom and ethics. A special spoken in Mithila region of Bihar. translated into Persian. Akbar
mention may be made of Honnamma, Vidyapati wrote his Keertilata, patronised many scholars. He got
perhaps the first outstanding poetess in Keertipataka and Padavali. Mahabharata translated into Persian.
Kannada. Her Hadibadeya Dharma (Duty Though, Khusrow has used Khari
Jahangir’s autobiography (Tuzuk-i-
of a Devout Wife) is a compendium of Boli in his compositions in the 13th
Jahangiri) is also in Persian and is a
ethics. century, its extensive use in unique piece of literature. Quite a fair
literature began only in the 19th amount of Persian literature has been
MALAYALAM LITERATURE century. The language also shows produced by the Mughal courtiers.
Malayalam is spoken in Kerala and its some influence of Urdu.
adjoining areas. The language of HINDI LITERATURE
Malayalam emerged around the 11th URDU LITERATURE Hindi is a direct descendant of the
century AD. By 15th century, Malayalam Urdu emerged as an independent Sanskrit language through Prakrit and
was recognised as an independent language towards the end of the Apabhramsha. It has been enriched by
language. Bhasa Kautilya, a commentary 14th century AD. Arabic and incorporation of words from languages
on Arthashastra and Kokasandisan are Persian were introduced in India such as Persian, English, Arabic etc. It
two great works. Rama Panikkar and with the coming of the Turks and is spoken largely in North India in
Ramanujan Ezhuttachchan are the Mongols. Persian remained the Delhi, Haryana, Western Uttar Pradesh,
well-known authors of Malayalam court language for many centuries. North-Eastern Madhya Pradesh and
literature. Urdu became more popular in the parts of Eastern Rajasthan.
early 18th century. People even Hindi Literature started as religious
NORTHERN INDIAN LANGUAGES wrote accounts of later Mughals in and philosophical poetry in mediveal
Urdu. Gradually, it achieved a
AND LITERATURE status, where the literature both
period in dialects like Avadhi and Braj.
A large number of people speak Hindi in The literature of Hindi language can be
poetry and prose started being divided into four different styles i.e.
its different forms that include Braj composed in Urdu. The last Mughal
Bhasha and Avadhi (spoken in Oudh Bhakti (devotional), Sri Nagar
emperor Bahadur Shah Zafar wrote (beauty), Veer-Gatha (praising brave
region), Haryanvi, Bhojpuri, Magadhi and poetry in it. Some of his couplets warriors) and Adhunik (modern).
Maithili (spoken around Mithila) and have become quite well-known in Evolution of Hindi literature can be
Rajasthani and Khari Boli (spoken around the Hindi and Urdu speaking areas. better understood through the four
Delhi). This classification is done primarily
Urdu as a language was born out of stages of Adi Kal, Bhakti Kal, Riti Kal
on the basis of the geographical region
the interaction between Hindi and and Adhunik Kal.
where these forms of Hindi language were
spoken. e.g. Braj Bhasha got its name Persian. Urdu was given its pride of ADI KAL (1050-1375 AD)
from Braj Mandal, the area where it is place by a large number of poets,
This period is also known as
who have left inimitable poetry for Veer-Gatha Kal. Prithviraj Raso by
spoken, Braj Mandal includes Mathura,
posterity. Chandra Bardai is considered as one of
Agra, Aligarh, Bareilly etc. Similarly
Avadhi is spoken in Avadh region of Uttar Among other well-known poets are the first works in Hindi literature.
Pradesh which includes Lucknow, Gonda, Ghalib, Zauq and Iqbal. Iqbal’s Other literary works from this period
Balrampur, Behraich etc. Urdu poetry is available in his belong to the Siddha, Nathpanthi, Jain
collection called Bang-i-dara. and Rasau poetical works.
1-288 GSP General Studies (Paper 1)

BHAKTI KAL (1375-1700 AD) BENGALI LITERATURE PUNJABI LITERATURE


This period witnessed the rise of Bhakti Raja Rammohan Roy, wrote in Punjabi is a language with several
Kavyas or devotional poetry. This form Bengali besides English which gave an shades. It is written in two scripts,
of poetry has been divided into two impetus to Bengali literature. Ishwar Gurmukhi and Persian. Guru
schools: Nirguna and Saguna schools, Chandra Vidyasagar (1820-91) and Nanak was the first poet in Punjabi.
depending upon the devotional attitude Akshay Kumar Dutta (1820- 86) were Some other contemporary poets,
of the poets towards the Lord. two other writers of this early period. mostly Sufi saints, used to sing in this
The Nirgunas believed in a formless A part from them, Bankim Chandra language. Guru Gobind Singh, the
God, while the Sagunas believed in a Chatterjee (l834–94), Sharat Chandra tenth guru, was educated in Patna
human incarnation of God. Chatterjee (l876-1938) and Romesh (Bihar), where he learnt Persian and
Chunder Dutt, a noted historian and a Sanskrit.
Kabir (1399-1518 AD) was the most
prose writer, all contributed to the He has composed two savaiyyas in
important poet of the Nirguna School.
making of the Bengali literature. Punjabi, but these are not a part of
The Saguna poets were either the
But the most important name that the Adi Granth. Similar is the
followers of Lord Rama or of Lord
influenced the whole of India was that of popularity of Bulley Shah, who was
Krishna.
a Sufi saint. He has left a large
Tulsidas is the foremost among Rabindranath Tagore (1861-1941).
number of songs.
the Ram Bhakt poets. He depicted Novels, dramas, short stories, criticism,
music and essays, all flowed from his One of his popular forms of
Rama as the ideal man in his classical
pen. He won the Nobel Prize for compositions was called Kafi, which
works Ramacharitamanasa, Gitavali,
literature in 1913 for his Geetanjali. was sung in a classical musical form.
Kavitavali and Vinaya Patrika.
Kafis are sung by people with great
ASSAMESE LITERATURE fervour. In the 20th century, Punjabi
Meera Bai (1498-1557) Like Bengali, Assamese also developed had come into its own.
She is the most celebrated of the women in response to the Bhakti movement.
poets of medieval times. Several works like Bhai Vir Singh composed an epic,
Shankaradeva, who introduced named Rana Surat Singh. Puran
Narsiji ki Mahera, Gita Govinda Ki Tika, Rag
Govinda, Garva Geet Raga-vihaga are
Vaishnavism in Assam helped in the Singh and Dr Mohan Singh are
attributed to Meera Bai. growth of Assamese poetry. Even the among the best known writers.
Puranas were translated in Assamese. Essays, short stories, poetry, novels,
RITI KAL (CE 1700 TO 1900) The earliest Assamese literature criticism and all other forms of
Literature in this period emulated the consisted of the buranjis (court writing have adorned the Punjabi
Sanskrit rhetorical tradition and tackled chronicles). literary scene.
different aspects of poetics like rasa, Shankaradeva has left several
alankara and nayika bheda through devotional poems, which people sang GUJARATI LITERATURE
Saviyas and Kavithas. Saptashati with rapturous pleasure, but it was only Early Gujarati literature is available
written by Bihari represents the literary after 1827, that more interest was in the form of Bhakti songs of the
work of this era. shown in producing Assamese 14th and 15th centuries. It still
literature. Two names, Lakshmi Nath follows the old tradition, which is
Bezbarua and Padmanaba Gohain popular in Gujarat. Narsi Mehta’s
Ghanananda (1699-1740) name is the foremost in this respect.
Barua cannot be forgotten in the same
Bodha (b.1747) and Thakur (1766-1823) were
the leading names of this genre of poetry.
regard. Narmad’s poetry gave a fillip to the
Ghanananda (Sujana-Sagara, Rasa-Kelivalli Gujarati literature. Saraswati
and Kripa-Kanda) is by far the best writer of
ORIYA LITERATURE Chandra, a novel by Govardhan
the non-rhetorical tradition of Hindi poetry. Oriya language shows the maximum Ram, has become a classic. Narsi
influence of Sanskrit. It originated in Mehta’s songs in praise of Krishna
ADHUNIK KAL (1900 ONWARDS AD) the 9th century AD and its literary not only made him a very popular
development took place in the figure, but also made the Gujarati
The most important development of this
13th century. Names worth mentioning language popular.
period was the evolution of Khari Boli
in Oriya literature are Fakirmohan
prose and proliferation of the use of
Senapati and Radhanath Ray, whose SINDHI LITERATURE
Khari Boli in poetry in place of Braj
writings deserve considerable attention Sindh was one of the important
Bhasha.
in the history of Oriya literature. centres of the Sufis, who established
This period has been divided into four khanqahs at various places. The Sufi
The works of Upendra Bhanja
phases: the age of Bharatendu or the singers with their devotional music
(1670-1720) were important as they
Renaissance (1868-1893), Dwivedi Yug made the language popular. The
ushered a new period of Oriya
(1893-1918), Chhayavadi Yug credit for creating literature in Sindhi
literature. In Odisha, the works of
(1918-1936) and the Contemporary goes to Mirza Kalish Beg and Dewan
Saraladaise are regarded as the first
Period (1937 onwards). Kauramal.
works of Oriya literature.
ART AND CULTURE 1-289

MARATHI LITERATURE MANIPURI LITERATURE


In Maharashtra, large number of local dialects Manipuri is a language of Tibetan- Burmese origin. Its literature up to the
were in use. Marathi grew out of these local 19th century belongs mainly to the Folk tradition.Examples of literature
dialects. The earliest Marathi poetry and prose from this period are–Numit Kappa (10th century), Naotinkhon Phambal
is by Saint Jnaneshwar (Gyaneshwar) who Kaba (16th-17th century), Lelthak Lekharol (17th century). Adaptations
wrote a long commentary on the Bhagavad and translations of Sanskrit texts were also done in this period. From
Gita. He was the one who started the Kirtan 20th century, modern Manipuri literature started developing. Important
tradition in Maharashtra. works from this period are ‘Le Paren’ and ‘Madhabi’ by Kamal Singh,
He was followed by Namdev (1270-1350), ‘Kamsa Badha’ by Ashangbam Minaketan Singha etc.
Gora, Sena and Janabai. All these were sang ROLE OF MODERN INDIAN LITERATURES
and popularised the Marathi language. Their
In almost all the Indian languages, the modern age begins with the first
songs are sung even today by the Varkari
struggle for India’s freedom in 1857, or near that time. The impact of
pilgrims on their way to Pandharpur
Western civilisation, the rise of political consciousness and the change
pilgrimage. Almost 2nd centuries later, Eknath
could be seen in what was written during that time. Contact with the
(1533-99) came on the scene. He wrote the
Western world resulted in India’s acceptance of Western thoughts on the
commentaries on the Ramayana and the
one hand and rejection of it on the other and resulted in an effort made to
Bhagavata Purana. His songs are very popular
revive her ancient glory and Indian consciousness.
all over Maharashtra.
Then came Tukaram (1598-1650). He is
supposed to be the greatest Bhakti poet of them OTHER INDIAN LANGUAGES AND INDIAN
all. Ramdas (1608-81), who was the Guru of
Shivaji, is the last of these hymn writers. He
LITERATURE
was the devotee of Rama. He inspired Shivaji. BODO
KASHMIRI LITERATURE It is a language from the Sino-Tibetan family. Presently, it is written in
Kashmir shot into literary prominence, when the Devanagari script. Even though, it is an ancient language its written
Kalhana wrote Rajatarangini in Sanskrit , but literature began only in the second decade of the 20th century. Initially,
this was in the language of the elite. the Christian missionaries published books on grammar and dictionary.
For locals, Kashmiri was the popular dialect. An early book was JD Anderson’s A Collection of Bodo Folktales and
Here, also the Bhakti Movement played its role. Rhymes, promoted by Bodo organisations. It is now a language for
Lal Ded, who lived in the 14th century was children upto the secondary school level.
probably the first to sing in the Kashmiri
language. She was a Shaivite mystic.
MAITHILI
It is an Indo-Aryan language spoken in Nepal and Northern India.
After Islam spread in this area, the Sufi
Earliest Maithili literature in the form of ballads, songs and dohas, was
influence also came to be visible. Habba
written between 700 and 1350 CE. Some popular writers of the period
Khatoon, Mahjoor, Zinda Kaul, Noor-ud-Din
were Sarahapad, Umapati and Jyotirishwar Thakur.
also known as Nand Rishi. Akhtar Mohiuddin,
Sufi Ghulam Mohammed and Dina Nath Nadim Sir George Graham Grierson researched Maithili folklore and transcribed
wrote devotional poetry in Kashmiri. These its grammar. In recent times, many prominent Maithili writers have
people contributed to the growth of Kashmiri emerged such as Satyananda Pathak, Ramesh Jha, Narendra Jha and
literature. Triloknath Mishra.

KONKANI LITERATURE DOGRI


Konkani language is close to Marathi and Hindi It is an Indo-Aryan language spoken mainly in Kashmir. It is presently
and is written in Devanagari script. Most of the written in the Devanagari script. Most of the initial work in Dogri was in
early work in Konkani has been lost. the form of translation. In 1873, Jyotishi Vishveshar translated
Mahabharata and Ramayana were translated Bhaskaracharya’s Lilavati into Dogri. The New Testament was also
in the 16th century by Krishnadas Sharma translated into Dogri by Christian missionaries.
from the Marathi language. A lot of Christian
In recent times, Karan Singh has written some important works in the
literature was written in the 17th century in
Dogri language such as New India and Welcome the Moonrise.
Konkani including Riglo Jesu Mollantum by
Father Joachim de Miranda and Papience
Xerathini by Dona Barreto. Modern period of
SANTHALI
Konkani literature began from the 20th century This language is a part of the Austric family. It is related to Ho and
onwards. Notable Konkani writers of modern Mundari. It is spoken by groups in Jharkhand, Bihar, Odisha, Tripura,
period are Balakrishna Bhagwant Borkar, West Bengal and Assam. During British times, it was written in the
M Sardesai and Reginaldo Fernandez. Roman script and later in the Devanagari script.
1-290 GSP General Studies (Paper 1)

Presently, the language is written in Shri Aurobindo, through his poetry and
its own script known as Ol Chiki,
THE LITERATURE OF philosophical treatise, ‘The Life Divine’,
which was invented by Pandit NATIONALISM presents the prospect of the ultimate
Raghunath Murmu in 1925. He also Patriotic writings proliferated almost revelation of divinity in everything. He
wrote over 150 books in this language. spontaneously in different languages, as wrote mostly in English.
a resistance of the community against
LITERATURE AND foreign rule. Rangalal (in Bengali,) and PROGRESSIVE LITERATURE
Bharatendu Harishchandra (in Hindi) The Progressive Writers
NATIONAL IDEOLOGY expressed themselves as the patriotic Association was originally established
voice of that era. This voice was, on the in 1936 by some expatriate writers in
A large number of writers opted for a
one hand, against colonial rule and on London, one of whom was Mulk Raj
synthesis between Indianisation and
the other, for the glorification of India. Anand (English). However, soon it
Westernisation, in their search for a
National Ideology. All these attitudes became a great Pan-Indian Movement
were combined to bring about the
BIRTH OF THE NOVELS that brought together Gandhian and
renaissance in 19th century, India. The birth of novels is associated with Marxist insights into society.
But it was a renaissance in a country the social reform-oriented movement of The movement was especially
which was under foreign domination. the 19th century. This new genre, conspicuous in Urdu, Punjabi, Bengali,
So, it was not that kind of renaissance, borrowed from the West, is Telugu and Malayalam, but its impact
which had spread in 14th-15th century characterised by a spirit of revolt, right was felt all over India.
Europe, where scientific reasoning, from its adoption into the Indian
It compelled every writer to re-examine
individual freedom and humanism system.
his/her relationship with social reality.
were the dominant characteristics. The first Tamil novel, Pratap In Hindi, romanticism of Chhayavada
The Indian renaissance took a Mudaliyar Charitram (1879) by was challenged by a progressive school
different shape, in the context of the Samuel V Pillai, the first Telugu that came to be known as Pragativada
Indian race, moment and milieu and novel, Shri Ranga Raja Charitra (1872) (progressivism).
as a result, nationalistic, reformistic by Krishnamma Chetty and the first
and revivalistic thinking found its way Malayalam novel, Indu Lekha (1889) by DALIT LITERATURE
into literature, which slowly turned Chandu Menon were written with One of the most significant features of
itself into a Pan-Indian movement, didactic intentions and to re-examine the post-modernist era was the
spearheaded in different parts of the evil social customs and practices like emergence of writings of the outcasts as
country by renaissance leaders untouchability, caste distinctions, a major literary force. The word ‘Dalit’
like Raja Rammohan Roy (1772-1833), denial of widow remarriage etc. means the downtrodden. The Dalit
Bankim Chandra Chatterjee, Movement was started in literature by
HISTORICAL NOVELS
Vivekananda, Madhav Govind the Marathi, Gujarati and Kannada
They were written by Bankim
Ranade, UV Swaminatha Iyer, Gopal writers under the leadership of Dr
Chandra Chatterjee (Bengali), Hari
Krishna Gokhale, Kandukuri Bhimrao Ramji Ambedkar.
Narayan Apte (Marathi) and others, to
Virasalingam Pantulu, Narmada It came into the limelight because of
describe the glorious past of India and
Shankar Lalshankar Dave and others. progressive literature moving nearer to
to instill nationalist fervour in her
The leaders of the renaissance, in fact, people. It was during this time that the downtrodden. It is a literature of
succeeded in instilling nationalistic Tagore started writing the novel Gora militant protest against the upper caste
fervour in the people and induced in (1910), to challenge the colonial rule, literature that upholded Brahmanical
them a desire for social reform and a colonial criteria and colonial authority values. Omprakash Valmiki is a
sentimental yearning for their past and to give a new meaning to Indian contemporary author known for his
glory. nationalism. writings with insights of Dalit
consciousness.
NEWSPAPERS AND INDIAN ROMANTICISM
PERIODICALS The trend of Indian romanticism ushered USE OF MYTHOLOGY
The birth of newspapers and in by three great forces influenced the In order to bridge the gap between
periodicals in Indian languages destiny of modern Indian literature. urban and rural consciousness,
between 1800 and 1850 was extremely These forces were Shri Aurobindo’s between the past and the present,
important for the development of (1872-1950) search ‘for the divine in another trend, which is very much
prose. The emergence of prose as a man’, Tagore’s quest ‘for the beauty in visible in the post-modernist poetry
powerful medium brought a kind of nature and man’ and Mahatma scene, is the use of mythology to
change that coincided with the process Gandhi’s ‘experiments with truth and present the modern predicament.
of modernisation. non-violence’. The mythical past affirms man’s
relationship to the transcendent. It has
a value structure. It is a rediscovery of
ART AND CULTURE 1-291

the past for the present and an in life around them. Even for the used for writing Punjabi) scripts. It
adaptation for the future. Indian English writers, English is no was also used for writing Sanskrit. It
One comes across many instances of more a colonial language. is now rarely used.
writers trying to explore their roots, Amitabh Ghosh, Shashi Tharoor, Nagari Script It was an Eastern
funding their moorings and probing Vikram Seth, Upamanyu Chatterjee, variant of the Gupta script. It is an
whole areas of experience, blurred Arundhati Roy and others are using it early form of the Devanagari script. It
during a period of extreme modernism without showing a lack of commitment branched off into many other scripts
during the last several decades. In to Indianness. Those writers who are such as Devanagari, Bengali, Tibetan
Contemporary Indian poetry, along aware of their inheritance, complexity etc. It was used to write both Prakrit
with a sense of urbanity, an attitude of and uniqueness, express in their work, and Sanskrit.
irony, frequent use of mythological without any conscious effort, both Devanagari Script It is the main
sequences as structural images and a tradition and the actuality. script at present to write standard
continuous involvement with the Hindi, Marathi and Nepali as well as
problems of expediency and eternity,
are very visible. INDIAN SCRIPTS Santhali, Konkani and many other
Indian languages. It is also used
Indus Script It refers to the script presently to write Sanskrit and is one
CONTEMPORARY used by the people belonging to the of the most used writing systems in
LITERATURE Indus valley civilisation. It has not the world. It is composed of Deva
been deciphered yet. Some people meaning,(God) and Nagari meaning,
The present day crisis in India is the
have argued that this script was the (city), which meant that it was both
conflict between expediency and
predecessor of the Brahmi script. This religious and urbane or sophisticated.
universality and as a result, a large
script is an example of
number of writers are in the process of Kalinga Script Kalinga was the
Boustrophedon style as in one line it
identifying a pattern of problem-solving ancient name of Odisha and this
is written from left to right while in
within the traditional system, vigorous script was used to write an ancient
other it is written from right to left.
enough to generate and sustain an form of Oriya. It is visually closest to
indigenous process of modernisation, Brahmi Script Brahmi is the the original Brahmi. Oriya language
which does not need readymade originator of most of the present presently uses a different script,
external solutions and is in accord with Indian scripts, including Devanagari, which has been derived from Bengali
indigenous needs and attitudes. Bengali, Tamil, Malayalam etc. script.
In the Uttara Adhunika (post-modern) It developed into two broad types in Grantha Script It is one of the
era, the effort is to be natural, to be Northern and Southern India, with earliest Southern scripts to originate
Indian, to be near to the common man, the Northern one being more angular from Brahmi. It branched off into
to be socially conscious. and the Southern one being more Tamil and Malayalam scripts, which
circular. It was deciphered in 1937 by are still used to write those
The third generation of Malayalam
James Princep. Its best examples languages, It is also the predecessor
writers like N Prabhakaran and
are found in the rock-cut edicts of of the Sinhala script used in Sri
P Surendran prefer the term
Ashoka. Lanka. A variant of Grantha called
Anti-Modernism to Post-Modernism
and are content simply to narrate Kharosthi Script It is the sister script Pallava was taken by Indian
human tales without any explicit social and contemporary of Brahmi. It was merchants to Indonesia, where it led
message or philosophical pretensions. written from right to left. It was used to the development of many
in the Gandhara culture of North- South-East Asian scripts. It was used
Kamleshwar, Nirmal Verma, Mohan in Tamil Nadu to write the Sanskrit
Western India and is sometimes also
Rakesh are known for the realistic Granthas and hence, was named
called the Gandhari Script. Its
portrayal of post-independence India in Grantha.
inscriptions have been found in the
Hindi literature.
form of Buddhist texts from present Vatteluttu Script It was a script
Recently Indian novels by Jayamohan day Afghanistan and Pakistan. derived from the Brahmi and was
(Tamil), Debes Ray (Bengali) and used in the Southern part of India. It
Gupta Script It is also known as the
Shivprasad Singh (Hindi), dealing with was used to write Tamil and
Late Brahmi script. It was used for
various neglected regions and the Malayalam. It removed those signs
writing Sanskrit in the Gupta period.
spoken dialect of that area, carving a from Brahmi, which were not needed
It gave rise to the Nagari, Sarada and
composite picture of a total India, for writing the Southern languages.
Siddham scripts which in turn gave
pulsating with new experience and Presently, both Tamil and Malayalam
rise to the most important scripts of
struggling to hold onto the old values have moved on to their own Grantha
India such as Devanagari, Bengali
and in the process sometimes also derived scripts.
etc.
discarding them, are also easily
discernible. The new crop of writers are Sarada Script It was a Western Kadamba Script It is a descendant of
concerned with the truth as they see it variant of the Gupta script. It evolved Brahmi and marks the birth of the
into Kashmiri and Gurmukhi (now dedicated Kannada script. It led to
1-292 GSP General Studies (Paper 1)

the development of modern Kannada and Telugu scripts. It was


used to write Sanskrit, Konkani, Kannada and Marathi.
Tamil Script It is the script used to write Tamil language in
India and Sri Lanka. It evolved from Grantha, the Southern
RELIGION AND
form of Brahmi. It is a syllabic language and not alphabetic. It
is written from left to right.
PHILOSOPHY
Religion is a transforming experience, not a
INDIAN LITERATURE theory of God. It is spiritual consciousness, belief
IN ENGLISH LANGUAGE and conduct, rites and ceremonies. Dogmas and
The English language came to India with the British and soon authorities are subordinate to the art of self
became a language of formal education. Some of the early Indian discovery and contact with the divine. There is a
works in English were written by Raja Rammohan Roy, Henry
Vivian Derozio and Madhusudan Dutt. Aurobindo Ghosh wrote long tradition of religious pluralism i.e. all the
his epic poem Savitri a Legend and a Symbol in English. major religions of the world are professed in
India’s only Nobel Laureate in literature Rabindranath Tagore India.
wrote some of his work originally in English and did some of his
own English translations from Bengali. There are four religions, which trace their origin to the
Sarojini Naidu and Jawaharlal Nehru also wrote in English. Indian sub-continent, namely Hinduism, Buddhism,
Nehru’s The Discovery of India and Glimpses of World History Jainism and Sikhism. At the same time, some religious
are quite popular. Some notable Indian poets who write in beliefs and systems have came from outside, They are
English are Nissim Ezekiel, Dom Moraes, Arun Kolatkar and Islam, Christianity, Zoroastrianism, Judaism and Bahai.
Dilip Chitre. Other notable Indian writers are Khushwant Singh, Indian philosophy refers to any of the several traditions
Salman Rushdie, Vikram Seth, Arundhati Roy, Kamala of philosophical thought that originated in the Indian
Markandaya, Kiran Desai, Jhumpa Lahiri etc. sub–continent, including Hindu philosophy, Buddhist
philosophy and Jaina philosophy. It is disciplined and its
goal is to improve human life.
CLASSICAL LANGUAGES
The government of India currently follows the following criteria
to determine the eligibility of language to be considered for
PROTO-HISTORIC RELIGION
The earliest civilised inhabitants of India the Harappans
classification as ‘classical language.’
worshipped, the chief male deity known as Pashupati
1. High antiquity of its early texts/recorded history over a period
Mahadeva and the chief female deity known as Mother
of 1500-2000 years.
Goddess. They also worshipped Gods in the form of
2. A body of ancient literature/texts, which is considered a trees, animals, snakes and birds.
valuable heritage by generations of speakers.
There is also evidence of both phallic and yoni worship.
3. The literary tradition be original and not borrowed from
The salient features of the Harappan religion appeared
another speech community.
in a new form in a later date. From this, we can conclude
4. The classical language and literature being distinct from
that it never died, but was practiced by some people,
modern, there may also be a discontinuity between the
gradually developing because of their contact with other
classical language and its later forms or its offshoots.
doctrines and cults, until it gathered enough faith to
reappear and influence the old faith of the Aryans.
CURRENT CLASSICAL LANGUAGES
Tamil was the first Classical language of India. The government
declared Tamil (in 2004), Sanskrit (in 2005). These two languages ANCIENT INDIAN RELIGIONS
are undoubtedly parental sources for many languages belonging
to the Indo-European family and the Dravidian family of VEDIC RELIGION
linguistic groups. Later the government declared Kannada and From the archaeological findings, it seems that the
Telugu (in 2008) as classical languages of India. In 2013, people during these times believed in the sanctity of the
Malayalam was also given status of classical language. In 2014, creative force and venerated the male and female
Oriya was also given the status of Classical language. With this aspects of divinity. It appears that they were the
the following six languages are included in the list of Classical worshippers of the forces of nature like the Sun and the
Languages Moon. This belief is also partly substantiated by the
Tamil (since 2004) Sanskrit (since 2005) Telugu (since 2008) early literature of the Aryans. The nature of the
Kannada (Since 2008) Malayalam (since 2013) Oriya (since 2014). religious beliefs and practices of the Aryans is also
known from the Rigveda. They believed in many Gods
like Indra, Varuna, Agni, Surya and Rudra.
ART AND CULTURE 1-293

Sacrifices and ritual offering of food and There were 24 Tirthankaras, who It has no beginning or end, but time is
drink to fire in honour of the Gods, actually established the Jain cyclical in nature with progressive and
constituted the main religious practices. darshan.The first Tirthankara realised regressive spirituality phases.
This ritualism was further elaborated in that the source of Jain philosophy The Jainas divided time into
the Brahmanas. was Adinath. The 24th and the Utsarpinis and Avsarpinis that
The Atharvaveda contained a great deal last Tirthankara was Vardhaman constituted one time cycle. Every
of animistic beliefs. It was during the Mahavira, who gave a great impetus to Utsarpini and Avsarpini was divided
later Vedic period that definite ideas and Jainism. Mahavira was born in 599 into ‘Aras’ or unequal periods, which
philosophies about the true nature of BC. are as follow
soul or the Atman and the cosmic He left worldly life at the age of 30 and
AHIMSA
principle or the Brahman, who led a very hard life to gain true
represented the ultimate reality knowledge. After he had attained the It is central to Jain belief. Compassion
developed. Polytheism was challenged by truth, he was called Mahavira. He towards all fellow living beings (along
monotheistic ideas and the various strongly believed in the importance of with humans) is central to Jain belief.
deities were introduced as different ways Celibacy or called Brahmacharya. Jainism is the only religion wherein
of naming one eternal entity. It must be all the followers, both monks and the
noted that the Aranyaka and Upanishad THREE JEWELS practicing lay persons of all the sects
sections of the Vedic literature envisage The path of Dharma (meaning truth, and traditions are required to be
a progressive outlook. teaching) that Mahavira advocated vegetarian.
was one of strict asceticism,
The Upanishadas represent the early ANEKANTAVADA
renunciation and moral cultivation. He
stage in the origin and development of It is one of the foundation pillars of
instructed his followers to cultivate the
the religions metaphysical concepts, Jain philosophy. Anekantavada is
three jewels
which were used later in ancient and defined as a (multiplicity of views and
1. Right belief.
medieval India. stresses and looking at things from the
2. Right knowledge.
3. Right conduct. other person’s perspective).
Unorthodox
SYADVADA
Religious Movements FIVE VOWS
It is the theory of conditioned
In the middle of the 1st Millennium BC, the 1. Ahimsa (Non-violence)
predication. It states that, since reality
religious movements associated with the 2. Satya (Truthfulness)
Mahavira and the Buddha fall under this is complex, no single proposition can
3. Asteya (Non-stealing) express the nature of reality fully.
category. There were many other creeds
preached by some elements during this 4. Aparigraha (Non-acquisition) Thus, the term syat meaning (may be)
time that were not keeping with the vedic 5. Brahmacharya (Chaste living) should be prefixed before each
tradition. There are two forms of the five vows proposition giving it a conditional
Both Buddhism and Jainism were atheistic 1. Mahavrata The five great vows point of view and thus, removing any
creeds in the beginning. However, dogmatism in the statement.
Buddhism endorsed the doctrine of the law
followed by Jaina monks and nuns.
of karma and upheld the belief in the 2. Anuvrata The lesser vows NAYAVADA
rebirths of the embodied Skandhas and the followed by Jain lay people. These It is the theory of partial standpoints
inevitability of suffering in the very are the less strict version of the or viewpoints. It says that an object
existence of beings. These view points great vows.
were also strengthened by the major has infinite aspects to it, but when we
Upanishadas. JAIN BELIEFS describe it in practice, we describe
only the relevant ones and ignore the
According to Jainism, the destiny of
others.
every being is a consequence of his
JAIN PHILOSOPHY actions. Souls are unborn and The standpoints we adopt are thus, an
Jain or Jina means the conqueror, Jains uncreated. They are also eternal and outcome of the purposes that we are
do not believe in the Vedas, but they equal. They exist in both animate and pursuing. Jaina philosophy is based
admit the existence of a soul. They also inanimate objects of existence. upon eternal, universal truths,
agree with the orthodox tradition, that according to its followers.
They all are capable of becoming free
suffering can be stopped by controlling During the first and last two Areas,
or attaining moksha, through their
the mind and by seeking the right these truths lapse among humanity
personal efforts. The liberation of each
knowledge and perception and by and then reappear through the
soul depends upon its own karma and
observing the right conduct. teachings of the enlightened humans,
purity of effort.
The Jain philosophy was first those who have reached enlightenment
According to Jain beliefs, the universe
propounded by the first Tirthankara or total knowledge (Kevala Jnana),
was never created, nor will it ever
Rishabha Deva. The names of Ajit Nath during the 3rd and the 4th (Jain
cease to exist. Therefore, history of the
and Aristanemi are also mentioned with Councils) Aras.
universe is shaswat.
Rishabha Deva.
1-294 GSP General Studies (Paper 1)

Traditionally, in our universe and in our time, Lord Buddha’s teachings are very practical and suggest how to
Rishabha is regarded as the first to realise the truth. Lord attain peace of mind and ultimate liberation from this
Vardhaman Mahavira was the last Tirthankara to attain material world.
enlightenment. The sacred texts of Jainism are known as
Agamas and are based on Mahavira’s teachings. THE FOUR NOBLE TRUTHS
The knowledge realised by the Buddha is reflected in the
The Kalpasutra is a Jain sacred text, which describes the
following four noble truths
biography of the Tirthankaras, mainly Parshvanath and
Mahavira. 1. There is Suffering in Human Life When Buddha saw
human beings suffering from sickness, pain and death,
JAIN THEORY OF REALITY he concluded that there was definitely suffering in
The Jains believe that the natural and supernatural things human life.
of the universe can be traced back to 9 fundamental 2. There is a Cause of Suffering The second noble truth
elements, namely is related to the cause of suffering. It is desire that
1. Jiva (Soul or living beings) motivates the cycle of birth and death.
2. Ajivaa (Non-living substances) 3. There is Cessation of Suffering The third noble truth
3. Asrava (Cause of the influx of Karma) tells that when passion, desire and love of life are totally
destroyed, pain cesses.
4. Astikaya (Bondage of Karma)
4. Path of Liberation The fourth noble truth leads to a
5. Samvara (A mest of the influx of Karma)
way that takes one to liberation. Thus, initially starting
6. Nirjana (Exhaustion of the accumulated Karma) with pessimism, the Buddhist philosophy finally leads to
7. Moksha (Total liberation from Karma) optimism.
8. Punya (Virtue) An important guiding principle of Buddhist practice is the
9. Paap (Sin) middle way (madhyama-pratipada), which is said to have
When life ends, the body dies, but not the soul. The soul been discovered by Gautama Buddha prior to his
moves from life-to-life until it attains Moksha or Nirvana. In enlightenment. The middle way is described as the practice of
order to achieve Nirvana, Jainism explains the law of non-extremism or a path of moderation away from the
Karma and shows the path of Moksha. extremes of self-indulgence and self-mortification.

SECTS OF JAINISM SECTS OF BUDDHISM


There are two main sects in Jainism. The followers of the During the 4th Buddhist Council, Buddhism split into the
23rd Tirthankara Parsvanath, Bhadrabahu were known as sects of Hinayana and Mahayana.
Svetambaras, while those following Mahavira came to be Hinayana It means the inferior vehicle. This sect considered
called as Digambaras. Buddha as a man and gave his teachings an ethical value.
Svetambaras and Digambaras agree on most of the basic Mahayana This sect subscribed to the theory of eternal
principles of Jainism and disagree on the following Buddhas, which resembled the Gods of theistic sects.
˜
Digambaras stress one the practice of nudity as an absolute
THE EIGHT FOLD PATH TO LIBERATION (NIRVANA)
pre-requisite to the mendicant’s path and to the attainment
Eight fold path to liberation are as follows
of salvation, the Svetambaras assert that the practice of
complete nudity is not essential to attain liberation. 1. Right Vision One can attain right vision by removing
ignorance.
˜
Digambaras believe that a woman lacks the adamantine
body necessary to attain Moksha i.e. liberation and hence, 2. Right Resolution It is the strong will-power to destroy
must be reborn as a man before such an attainment is thoughts and desires that harm others. It includes
possible. But, the Svetambaras hold the contrary view and sacrifice, sympathy and kindness towards others.
maintain that women are capable in the present life time 3. Right Speech Man should know, how to control his
itself of the same spiritual accomplishments as men. speech.
˜
According to the Digambaras, once a saint becomes Kevali 4. Right Conduct It means to avoid activities, which harm
or Kevala-jnani i.e. omniscient, he needs no morsel of food. life.
But this view is not acceptable to the Svetambaras. 5. Right Livelihood Right livelihood means to earn one’s
bread and butter by the right means.
BUDDHA PHILOSOPHY 6. Right Effort It is also necessary to avoid bad feelings
The Buddha presented simple principles of life and practical and bad impressions.
ethics that people could follow easily. He considered the 7. Right Mindfulness It means to keep one’s body, heart
world as full of misery. and mind in their real form.
Man’s duty is to seek liberation from this painful world. He 8. Right Concentration If a person pursues the above
strongly criticised blind faith in the traditional scriptures seven rights, he will be able to concentrate properly and
like the Vedas. rightly.
ART AND CULTURE 1-295

SHAIVISM and the various denominations of


THEISTIC RELIGIONS protestantism. With about 2.2 billion
The theistic character evolved almost Panini refers to a group of Shiva
worshippers as Shiva Bhagavatas. adherents, Christianity is the world’s
simultaneously with the non-theistic largest religion.
religions. The important deities of these They were characterised by the iron
religions were not primarily vedic ones, but lances and clubs that they carried The Old Testament and the New
those that came from unorthodox sources. along with their skin garments. Testament are considered as the sacred
The Shaiva movement in the South texts of Christianity.
The primary factor that motivated these
creeds was Bhakti, the single-soul flourished at the beginning through In Christianity, trinity refers to the
devotion of the worshipper to a personal the activities of many of the 63 teaching that God comprises three
God with some moral link. This led to the saints known in Tamil as distincts, eternally co-existing persons,
evolution of different religious sects like Nayanars (Siva-bhakts). that is the Father, the Son and the
Vaishnavism, Shaivism and Shaktism, Their appealing emotional songs in Holy Spirit.
which came to be regarded as components Tamil were called Tevaram Confirmation is a rite of initiation
of orthodox Brahminism. These sects in Stotras and also as Dravida Veda in Churches carried out through
course of time, came to have a significant and ceremonially sung in the local anointing, the laying on of hands and
impact on the popular forms of Buddhism Shiva temples. It must be noted prayers, for the purpose of best owing
and Jainism. that the Nayanars hailed from all the gift of the holy spirit.
the castes. This was supplemented Ordination is the process in
FOLK CULTS on the doctrinal side by a large Christianity by which individuals are
The worship of Yakshas and Nagas and number of Shaiva intellectuals, set apart as clergy to perform various
other folk deities constituted the most whose names were associated with religious rites and ceremonies.
important part of primitive religious several forms of Shaiva movements Confession is the practice by which
beliefs, in which Bhakti had a very like Agamanta, Shudha and men and women may confess their sins
important role to play. There is ample Vira-shaivism. committed after Baptism and to get
evidence about the prevalence of this form
them absolved by a Pope.
of worship among the people.
OTHER RELIGIONS SIKHISM
VAISHNAVISM
The history of the above movement from ISLAM The Sikhs, mainly belonging to Punjab,
the end of the Gupta period till the first form a sizeable group of our population.
It is a monotheistic religion
decade of the 13th century AD is concerned The orthodox Sikhs believe that their
articulated by the Quran
mainly with South India. In South India, religion was revealed by the God to
(considered as the verbatim word of
Vaishnava poet saints known as Alvars Guru Nanak, whose spirit entered the
God) and the teachings and
preached single- minded devotion 2nd and the subsequent gurus and till
normative examples of Prophet
(Ekatmika Bhakti) for Vishnu and their the 10th Guru. Guru Gobind Singh
Mohammed (considered as the last
songs were collectively known as ordained the Sikhs to treat the Adi
prophet of God). With about 1.57
Prabandhas. Granth, as his successor, elevating it to
billion followers.
Guru Granth Sahib.
A Sutra in Panini’s Ashtadhyayi refers to Islam is the second largest religion
the worshippers of Vasudeva (Krishna). The Sikhs basically believed in a
in the world. According to
The Chhandogya Upanishad also speaks of formless God, equality of the mankind,
traditional Islamic view, Quran
Krishna, the son of Devaki, a pupil of the need of a Guru and the pahul tradition.
began with revelations to
sage Ghora Angirasa, who was a Sometimes, the gurudom was conferred
Mohammed when he was 40 years
Sun-worshipping priest. A large number of on the son and sometimes on the best
old, that is in 610 AD. Most
people worshipped Vasudeva Krishna disciple.
Muslims belong to two
exclusively as their personal God and they denominations Shia and Sunni. The 5th Guru, Guru Arjan Dev,
were at first known as the Bhagavatas. Islam arrived in India in the 8th provided the Sikhs, three things–Adi
The Vasudeva- Bhagavata cult grew century AD with the Arab traders. Granth, standardised script for
steadily, absorbing within its fold other Gurmukhi, the site and the foundation
Vedic and Brahminic divinities like Vishnu CHRISTIANITY of the Harmandir Sahib or the Golden
(primarily, an aspect of the Sun) and Temple and the Akal Takht at
It is a monotheistic religion based
Narayana (a cosmic God). Amritsar.
on the life and teachings of Jesus.
From the late Gupta period, the name The Christians believe that Jesus is The 10th Guru, Guru Gobind Singh
mostly used to designate this Bhakti cult the son of God and the saviour of created the Khalsa, which means, the
was Vaishnava, indicating the humanity. Pure, in 1699. He also ordained the
predominance of the Vedic Vishnu element Sikhs to take 5 vows, namely, keeping
The three largest groups of
in it with emphasis on the doctrine of of kesh, kangha, kada, kirpan and
Christianity are roman catholic
incarnations (avataras). kaccha.
church, eastern orthodox church
1-296 GSP General Studies (Paper 1)

Consequently, these symbols became the distinguishing


marks of a Sikh. He further added that after his death
BAHAI FAITH
the Adi Granth will be the Guru of the Sikhs and they It is a monotheistic religion founded by Baha‘u’llah in 19th
have to pay obedience to this holy book. century Persia which emphasises the spiritual unity of all
humankind. The core principles of Bahai’s are unity of God, unity
ZOROASTRIANISM of religion and unity of humankind.
The Parsi or Zoroastrian religion was founded by There are an estimated five to six million Bahai’s around the world
Zarathushtra or Zoroaster, in the 8th century BC. He in more than 200 countries and territories. The Bahai’s believe
preached monotheism in the region now known as that religious history has unfolded through a series of divine
Persia. messengers each of whom established a religion which was suited
He taught the worship of fire and the presence of good for the needs and capacities of the people of the time. The key
and bad in the form of Ahura Mazda and Angra scriptures of Bahai’s are Kitab-i-Aqdas and Kitab-i-Iqan.
Mainyu. He also taught the ethical doctrine of kindness
and charity. These doctrines are enshrined in the
SHINTOISM
Avesta. It is an indigenous spirituality of Japan. It refers to a set of
practices to be carried out diligently so to establish a connection
JUDAISM between present day Japan.
It is a monotheistic religion originating in the Hebrew
Bible. It is one of the oldest monotheistic religions and THE SCHOOLS OF INDIAN PHILOSOPHY
spans a period of more than 3000 years. Jewish religious According to a traditional principle of classification, the schools or
texts consist of Tanakh, works of the Talmudic era, systems of Indian philosophy are divided into two broad classes,
Midrashic literature, Halakhic literature etc. namely, the orthodox (astika) and heterodox (nastika).
The basis of Jewish law and tradition is contained in the To the first group belong the six chief philosophical systems
Torah. There are 613 commandments in the Torah. (popularly known as Shadadarshana), namely mimamsa,
Other important texts are Talmud and Tanakh. There vedanta, samkhya, yoga, nyaya and vaisesika. These are regarded
are around 13.4 million Jews in the world with around as orthodox (astika), not because they believe in God, but because
42% of them in Israel and a similar number in United of their acceptance of the authority of the Vedas.
States.
Under the other class of heterodox systems, the chief three are
CONFUCIANISM the schools of the materialists like the Charvakas, the Buddhas
and the Jainas. They are called Heterodox (nastika) because
It is a Chinese ethical and philosophical system based on
they do not believe in the authority of the vedas.
the teachings of Chinese Philosopher Confucius, who
lived between 551 and 479 BC.
THE SAMKHYA SYSTEM
The core belief of Confucianism is humanism, which is
It is a philosophy of dualistic realism, attributed to the sage
the belief that human beings are teachable, improvable
Kapila. It admits two ultimate realities namely, purusa and
and perfectible through personal and communal
prakrti, which are independent of each other in respect of their
endeavour. Confucius taught six books to his followers,
existence.
namely the Classic of Poetry, Book of Documents, Book
of Rites, Book of Music, I-Ching and Spring and Autumn REALISATION OF THE SELF
Annals. It is the self which is quite distinct from the body, the senses and
Ren is one of the basic virtues promoted by confucianism the mind. It is beyond the whole world of objects and is the
which means an obligation of altruism and humanness eternal consciousness which witnesses the changes and activities
for other individuals within a community. Filial piety is going on in the world, but does not itself act and change in any
considered as one of the greatest virtues in confucianism way. Therefore, there must be the purusa or the self which is
and it must be taught both the living and the dead. distinct from prakrti or primary matter, but is the enjoyer
(bhokta) of the products of the prakrti.
TAOISM There are many different selves related to different bodies, for
It is a philosophical and religious practice which when some men are happy, others are unhappy, some die, but
believes in harmonising oneself with nature. The literal others live. The Prakrti is the ultimate cause of the world. It is an
meaning of Tao is way, but it can also road, channel, eternal unconscious principle (jada), which is always changing
path, doctrine or line. The key text of Taoism is Tao Te and has no other end than the satisfaction of the selves, sattva,
Ching which consists of teachings attributed to Laozi. rajas and tamas are the three constituents of prakrti, which holds
Other important texts in Taoism are Zhuangzi, Daozang them together in a state of rest or equilibrium (samyavastha). The
and Mozi. The three treasures or three Jewels of taoism three are called the gunas.
are basic virtues in Taoism namely; compassion, But they are not qualities or attributes in any sense. Rather, they
moderation and humility. are three substantial elements which constitute the prakrti like
three cords making up a rope. The existence of the gunas is
ART AND CULTURE 1-297

inferred from the qualities of pleasure, This is the state of liberation or freedom He did not create the world out of
pain and in difference which, we find in from suffering which has been variously nothing, but out of eternal atoms;
all the things of the world. described as mukti, apavarga, kaivalya space, time, ether, minds and souls.
The same sweet is liked or disliked or etc. It is possible for us to attain this This world has been created in order
treated with in difference by the state while alive in this world that individual souls (jivas) might
same man in different conditions. The (jivanmukti) or after this life in the enjoy pleasure or suffer pain according
other world (videhamukti). to the merit or demerit of their actions
course of evolution is as follows.
According to the Samkhya philosophy, in other lives and in other worlds.
From prakrti arises the great gem of
this vast universe which is called, the the existence of God cannot be proved in
great one (mahat). any way. We need not admit the THE YOGA SYSTEM
existence of God to explain the world ; The sage Patanjali is the founder of
SELF-CONSCIOUSNESS for prakrti is the adequate cause of the the Yoga philosophy. The yoga is
The consciousness of the self is reflected world as a whole. closely allied to the samkhya. It mostly
on this and makes it appear as accepts the epistemology and the
conscious. It represents the awakening THE NYAYA SYSTEM metaphysics of the samkhya with its
of nature from her cosmic slumber and It is the work of the great sage twenty-five principles, but admits also
the first appearance of thought and Gautama. It is a realistic philosophy to the existence of God. The special
therefore, it is also called the Intellect based mainly on logical grounds. It interest of this system is in the
(buddhi). admits four separate sources of true practice of yoga as the means to the
knowledge, viz perception (pratyaksha), attainment of vivekajnana or
It is the creative thought of the world to
inference (anumana), comparison discriminative knowledge, which is
be evolved. Ahankara, the second
(upamana) and testimony (sabda). held in the Sankhya to be the essential
product, arises by a further
condition of liberation.
transformation of the intellect. The
OBJECTS OF NYAYA SYSTEM
function of the ahankara is the feeling FUNCTIONS OF THE YOGA SYSTEM
The objects of knowledge, according to
of ‘I and mine’ (abhimana). According to it the yoga consists in the
the Nyaya are the self, the body, the
Owing to its identification with this senses and their objects, cognition cessation of all mental functions
principle, the self considers itself to be (buddhi), Mind (manas), activity (cittavrtti nirodha). There are five
an agent (karta), which it really is not. (pravritti), mental defects (dosa), levels of mental functions (cittabhumi).
From ahankara, with an excess of the rebirth (pretyabhava), the feelings of The first is called Skipta or the
element of sattva, arise the five organs pleasure and pain (phala), suffering dissipated condition, in which the
of knowledge (jnanendriya), the five (duhkha) and freedom from suffering mind flits among objects.
organs of action (karmendriya) and the (apavarga). The second is mudha or the stupefied
mind (manas), which is at once an condition as in sleep. The third is
organ of knowledge and activity IMPORTANT COMPONENTS
called viksipta or the relatively
(ubhayendriya). With an increase of The Nyaya, like many other systems of
pacified condition. The yoga is not
tamas, the ahankara produces, on the the Indian philosophy seeks to deliver
possible in any of these conditions. The
other hand, the five subtle elements the self from its bondage to the body,
fourth and the fifth levels are called
(tanmatra), which are the potentialities the senses and their objects. According
Ekagra and Nirodha.
of sound, touch, colour, taste and smell. to it, the self is distinct from the body
and the mind. The mind (manas) is a (i) The first is a state of
From the five subtle elements come the
subtle, indivisible and eternal substance concentration of the mind on
five gross elements of akasa ether, air,
(anu). It serves the soul as an some object of contemplation.
fire, water and earth in the same
order. Thus, we have altogether instrument for the perception of psychic (ii) The other is the cessation of even
twenty-five principles in the samkhya. qualities like pleasure, pain etc. It is the act or function of
therefore, called an Internal Sense. contemplation.
REALISATION OF THE SELF AND The self (atman) is another substance, (iii) The last three levels of the mind
THE NON-SELF which is quite distinct from the mind (cittabhumi) are conducive to the
Once we realise the distinction between yoga.
and the body. It acquires the attribute
the self and the non-self including the
body and the senses, the mind, the of consciousness when it is related to TYPES OF YOGA
intellect and the ego (Vivekajnana), any object through the senses. The There are two kinds of yoga or
ourself ceases to be affected by the joys existence of God is proved by the samadhi, viz samprajnata and
and sorrows, the ups and downs of life. Naiyayikas by several arguments. God asamprajnata. In the first, we have
It rests in itself as the dispassionate yoga in the form of the mind’s perfect
is the ultimate cause of the creation,
observer of the show of events in the concentration on the object of
world without being implicated in maintenance and destruction of the contemplation and therefore, involving
them. world. a clear apprehension of that object.
1-298 GSP General Studies (Paper 1)

In the second, there is the complete cessation of all mental There are altogether twenty-four kinds of qualities, viz colour,
modifications and consequently, the entire absence of all taste, smell, touch, sound, number, magnitude, distinctness,
knowledge including that of the contemplated object. There conjunction, fluidity (dravata), viscidity (sneha), cognition
are eight steps in the practice of yoga. (buddhi), pleasure, pain, desire, aversion, striving (prayatna),
These are yama or restraint, niyama or moral culture, heaviness (gurutva), tendency (samskara), merit (dharma)
asana or posture, pranayama or breath control, pratyahara and demerit (adharma) etc.
(withdrawal), dharana (conception), dhyan (deep
meditation) samadhi (concentration) etc. The yoga system is
THE MIMANSA SYSTEM
called the Theistic (sesvara). The Mimansa (or purva-Mimansa) school was founded by
Jaimini. Its primary object was to defend and justify vedic
PHILOSOPHY OF YOGA ritualism. In course of this attempt, it had to find a philosophy
The yoga argues for the existence of God on the following supporting the world view, on which ritualism depends.
grounds The authority of the vedas is the basis of ritualism in the
˜
Whatever has degrees must have a maximum. This Mimansa, which formulates that the Vedas were not the
maximum is represented by God. works of any person and were free from errors that the human
˜
There are degrees of knowledge; therefore, there must be authors commit. The vedas are eternal and self-existing; the
such a thing as perfect knowledge or omniscience. written or pronounced vedas are only their temporary
˜
He who has omniscience is God. manifestations through particular seers.
˜
The association of purusa with the prakrti is what initiates For establishing the validity of the vedas, the Mimansa
the evolution of the world and the cessation of this leads to discusses very elaborately the theory of knowledge, the chief
dissolution. object of which is to show that the validity of, every knowledge
is self-evident. When there are sufficient conditions,
˜
Neither the association nor the dissociation is natural to
knowledge arises.
prakrti and the purusa. Therefore, there must be a
supreme being, who is able to bring about these relations When the senses are sound, objects are present to them and
between the prakrti and purusa according to the moral when other auxiliary conditions also prevail, there is
deserts of individuals souls. perception. When there are sufficient data, there is inference.
Similarly by reading the vedas, we have at once a knowledge
THE VAISESIKA SYSTEM and also belief in what they say. The validity of Vedic
It was founded by the sage Kanada, also named as Uluka. knowledge is self-evident like that of every other knowledge. If
It is allied to the Nyaya system and has the same end in any doubts arise, they are removed with the help of Mimansa
view namely, the liberation of the individual self. It brings arguments and the obstacles being removed, the vedas
all the objects of knowledge i.e. the whole world, under the themselves reveal their contents to the reader. The authority
of the vedas, thus becomes unquestionable.
following categories of substance (dravya), quality (guna),
action (karma), generality (samanya), particularity (visesa), PHILOSOPHY OF MIMANSA SYSTEM
the relation of inherence (samavaya) and non-existence The soul in the body has different kinds of knowledge. One
(abhava). school of Mimansa founded by Prabhakara admits five different
sources of knowledge (pramanas), namely perception (pratyaksa),
PHILOSOPHY OF VAISESIKA SYSTEM inference (annumana), comparison (upamana), testimony
These are four kinds of atoms earth, water, fire and air, (sabda) and postulation (orthapatti). The first four are admitted
which are invisible and indestructible particles of matter. as in the Nyaya system.
The atoms are uncreated and eternal entities, which are
The Mimansa believes in the reality of the physical world on
obtained by resolving any material object into smaller and the strength of perception. It is therefore realistic. It believes
smaller parts till to they cannot be further divided. in the reality of souls as well. But, it does not believe that
Akasa, space and time are imperceptible substances, each of there is a supreme soul or God, who has created the world. It
which is one, eternal and all pervading. does not hold like other orthodox systems that there is a cycle
The mind is an eternal substance, which is not all of creation and dissolution. The world has always been as it is.
pervading, but infinitely small like an atom. It is the It has neither a beginning nor an end. The world’s objects are
internal sense, which is directly or indirectly concerned in formed out of matter in accordance with the Karmas of the
all physical functions like cognition, feeling and will. The souls. The law of Karma is an autonomous, natural and moral
supreme soul or the God is inferred as the creator of the law that rules the world.
world of effects. The atoms are made to compose a world The Mimansa also admits that when any man performs any
that befits the unseen moral deserts of individual souls and ritual, there arises in his soul a potency (apurva), which
serves the purpose of moral dispensation. This is the atomic produces in future the fruit of the action at an opportune
theory of the vaisesikas. moment.
ART AND CULTURE 1-299

THE VEDANTA SYSTEM


This system arises out of
Upanishadas, which mark the culmination
the
ART AND ARCHITECTURE
of the vedic speculation and are fittingly
called the Vedanta or End of the Vedas. Indian art and architecture, which have evolved through centuries,
Of all the systems, the vedanta, especially is the result of socio-economic and geographical conditions.
as interpreted by sankara, has exerted the
greatest influence on Indian life and it still Different types of Indian art and architectural styles include forms
persists in some form or other in different
parts of India. Sankara interprets the
of expressions over space and time, transformed by the forces of
Upanishadas and the Brahmasutra to history considered unique to India. As a result of vast diversities, a
show that pure and unqualified monism is
taught therein. vast range of architectural specimens have evolved, retaining a
EXISTENCE OF GOD certain amount of continuity across history.
God is the only reality, not simply in the
sense that there is nothing except God, but Art is a subjective concept and has no universal definition. We can define art as
also in the sense that there is no the power, skill, ability and talent that an individual possesses to express
multiplicity even within God. himself or herself. Art is simply an acquired skill that can be brought into use
make another person know and understand, what you think and how you feel.
The denial of plurality, the unity of the ‘‘Art happens to be a yet another word for expression’’.
soul and God, the assertion that when God
is known, all is known and similar views
found in the Upanishadas, in fact, the THE INDUS VALLEY PERIOD
general tone that pervades their The forms of art found from various sites of the Indus Valley Civilisation include
teachings, cannot be explained terracotta figurines, pottery, seals, ornaments etc.
consistently if we believe even in the
existence of many realities within God. TERRACOTTA FIGURINES
PHILOSOPHY OF VEDANTA SYSTEM A large number of terracotta figurines of men, women and animals discovered
from the sites of Indus Valley Civilisation comprises a remarkable group. The
In view of this, Sankara finds it necessary
figurine of the great Mother Goddess is one of the most typical evidence of the
to distinguish three different points of
civilisation.
view the ordinary or empirical
(Vyavaharika), the transcendental or real The survived works consist of the high-crested head-dress, the chaplets round
(paramarthika) and the apparent reality the neck, the long series of pendant necklaces, a beautiful miniature bronze girl
or unreality (pratibhasika). with thin limbs who holds a bowl against her thigh etc. The humped bull, dog,
sheep, elephant, rhinoceros, pig, monkey and several birds, as well as wheeled
The first is the standpoint of
model carts and whistles constitute the wide variety of clay toys that the Indus
unenlightened persons, who regard the
valley people fashioned for their children’s enjoyment.
world as real; our life of practice depends
on this; it is rightly called, therefore, the
Vyavaharika or practical point of view.
POTTERY
The variety of wheel-turned pottery baked in round kilns, reveals the amazing
From this point of view, the world appears
expertise of the Indus valley craftsmen. The great variety of painted
as real; God is thought to be its
decorations and pictorial motifs, including leaf patterns, scales, chequers,
omnipotent and omniscient creator,
lattice-work, wave patterns, trees and floral designs were their most important
sustainer and destroyer.
creations.
Thus, God appears as qualified (Saguna)
by many qualities. God in this aspect is SEALS
called by Sankara as the Saguna One of the most distinctive artefacts of the Indus Valley Civilisation, the seals
Brahman or the isvara. of the period were made of steatite (soapstone). The seals had animal motifs
Paramarthika It is an entity which is and various signs. These seals are rectangular, circular or even cylindrical in
present everywhere and at all times. shape. The seals outline the culture and civilisation of the Indus valley people.
This is the true and eternal reality. In particular, they indicate
Pratibhasika It has neither basis nor any
˜
the dresses, ornaments and the hairstyles of people.
existence. It is the illusion and a good ˜
skill of the artists and sculptors.
example is the reality during dream. ˜
trade contacts and commercial relations.
˜
religious beliefs.
˜
scripts of the people.
1-300 GSP General Studies (Paper 1)

IMPORTANT SEALS EVOLUTION OF STUPAS


THE PASHUPATI SEAL
MAURYAN ART Stupa architecture developed gradually.
In the Mauryan period, stone culture Earlier stupas were made up of bricks as
This seal depicts a yogi, probably
dramatically emerged as the principal was Piprahva stupa but in later stages
Lord Shiva (Pashupati) as claimed
medium of the Indian artists. Some stupas were built up of stone.
by John Marshal. A pair of horns
evidence is put forward by John Irwin Mahastupa of Sanchi and Stupa of
crowns his head. He is surrounded by
that Ashokan columns may be the Bharhut are important examples of
a rhino, a buffalo, an elephant and a
culmination of the ancient pre-buddhist stupa architecture.
tiger.
religious tradition in India of a cult of
Under his throne, are two deers. This one cosmic pillar of axis mundi. With PILLARS
seal shows that Shiva was the Mauryan empire, came a change in The pillars built by Ashoka furnish the
worshipped and that he was the art forms as well. finest remains of the Mauryan art. The
considered as the Lord of Animals Earlier, wood was the chief material for pillars with Ashoka edicts inscribed on
(Pashupati). most of the art forms, but it was them were placed either in sacred
THE UNICORN SEAL changed to stone during the Mauryan enclosures or in the vicinity of towns.
The Unicorn is a mythological empire. Even the present day National The pillars were made of two types of
animal. The Unicorn seal shows that Emblem of India, the Lion Capital of stone : the spotted Red and White
at a very early stage of civilisation, Ashoka at Sarnath belongs to the sandstone from the region of Mathura
the humans had produced many Mauryan empire. and the Buff coloured fine grained
creations of imagination in the shape hard sandstone usually with small black
of birds and animal motifs that
STUPAS spots quarried in Chunar near Varanasi.
survived in the later art as well. Stupas are Buddhist religious The palace of Chandragupta Maurya
monuments and were originally only a excavated in Kumrahara near Patna is
THE BULL SEAL simple mound of mud or clay to cover an example of high level of stone art. The
This seal depicts a humped bull of the supposed relics of the Buddha and stone was transported from Mathura
great vigour. The figure shows the other prominent Buddhist monks. and Chunar to the various sites where
artistic skills and a good knowledge of These were basically funeral mounds the pillars have been found and here the
animal anatomy. which were low and circular mounds stone was cut and carried by craftsmen.
ringed by the boulders.
ORNAMENTS Each pillar had three parts: the Prop
Stupa construction was a Buddhist art, under the foundation, the Shaft or the
The ornaments made of gold, silver, however even the Jains also seemed to Column and the Capital. The Prop is
copper and other metals were worn have built stupas. After the passing buried in the ground. The Shaft made of
by both men and women. Men wore away of the Buddha, his remains were a single piece of sandstone supports the
necklaces, finger rings and armlets of cremated and the ashes divided and Capital made of another single piece of
various designs and shapes. buried under eight stupas with two sandstone.
The women wore a head dress, further stupas encasing the urn and the
embers. The thin–round and slightly tapering
earrings, bangles, girdles, bracelets shaft is highly polished and very
and anklets. The rich people wore A railing surrounded this is called as graceful in its proportions. The capital,
expensive ornaments made of gold vedika. These railings were constructed which is the third part of the pillar,
while the poor had ornaments made of stone but resembled the look of the consists of some finally executed animal
of shell, bone or copper. wooden railings of the past in design. figures such as the lion or the elephant.
The Buddha’s relics were placed in a
ARCHITECTURE casket chamber in the centre of the CAVES
The layouts of Harappan cities were dome. At the base of the dome, is a high The architectural remains of the cave
much planned and the streets were circular terrace probably meant for architecture ascribed to the Mauryan
rectilinear. The houses of the earlier parikrama or circumambulation and an period are few. The Chaitya halls and
settlements were with an upper encircling the Balustrade. At the the Stupa do not exist in their original
storey, made of mud and baked bricks ground level is a stone-paved procession form except the excavated chaitya halls,
in standardised sizes. path and another stone balustrade and bearing inscriptions of Ashoka and
two flights of steps leading to the Dasaratha, in the Barabar caves.
The streets, along with the houses
circular terrace.
were wide, with drains collecting The monolithic rail at Sarnath in
from bathrooms and latrines by Access to the same is through four grey and polished Chunar sandstone has
means of chutes. exquisitely carved gateways or Toranas been erected under the patronage
in the North, South, East and West. of the Emperor Ashoka himself. Its
The granaries and the Great Bath at
The diameter of the stupa is 36.60 m architectural form is similar to the
Mohenjo-daro seem to have been
and its height is 16.46 m. rails of Bharhut.
constructed for special rituals.
ART AND CULTURE 1-301
The Bharhut stupa must have been STUPAS strategic location, the Gandhara school
literally transferred into stone from The stupas at Amaravati were made of imbibed all kinds of foreign influences
contemporary wooden originals. a distinctive white green marble and like Persian, Greek, Roman, Saka and
The plinth or the alambana, the probably began about the time of the Kushana.
horizontal bars and the coping have all Christ and received its final carved The origin of the Gandhara art can be
been just carved out of what must have faces and railings from about 150-200 traced to the Greek rulers of Bactria
been a huge slab of stone. The altar or AD. The diameter of the stupa at the Parthia and North-West India. But it
the bodhimanda suited at Bodhgaya is base was 51 m. The height of the dome was during the reign of Kanishka that
traditionally associated with Ashoka. was 31 m and its outer railing was 5 the art received great patronage.
The Bharhut altar consists of four meters wide. The stupendous stupa
could not withstand the ravages of time
CONTRIBUTION OF
pilasters. It is argued that one of the GANDHARA SCHOOL
caves in the Barabar hills called the and its ruins are preserved in the
London museum. The Gandhara School of Art is also
Sudama Cave, was dedicated by known as the Graeco-Buddhist
Ashoka to the Ajivika monks of the Amaravati also had a great influence
School of Art since, Greek techniques
Ajivika sect. Their interior walls are over South Indian sculpture. Its
of art were applied to the Buddhist
polished like mirror. sculptures show its mastery in stone
subjects. The most important
It has rightly been said that Ashoka sculpture. The monuments at
contribution of the Gandhara School of
inaugurated a style of architecture Jaggayyapeta, Nagarjunakonda and
Art was the evolution of beautiful
which spread in different parts of the Amaravati are examples. The Andhra
images of the Buddha and
country and itself at its best in the sculpture is also known as the
Bodhisattavas, which were executed in
magnificent masterpiece of Karla, Amaravati school. Other pieces of art
black stone and modelled on identical
Ajanta, Ellora and Elephanta. from the Amaravati school include the
characters of Graeco-Roman pantheon.
ornate bull or the Nandiswara that
Hence it is said, ‘the Gandhara artist
was situated in the Amreswara temple.
had the hand of a Greek, but the heart
POST–MAURYAN ART of an Indian.’
SUNGA AND KANVA ART KUSHANA ART
The most characteristic trait of
During the rule of the Kanva and The period of the Kushana
Gandhara sculpture is the depiction of
the Sunga dyansty, a plenty of empire marked a progress in
Lord Buddha in the standing or seated
cave-temples, chaityas and stupas were architecture, sculpture and painting.
positions. The seated Buddha is always
built. Kanishka was a patron of art, as is
shown cross-legged in the traditional
evident by the cities of Mathura, Taxila
STUPAS Indian way.
and Peshawar and the innumerable
The stupas of Bharhut, Bodhgaya and stupas, chaityas and viharas built Another typical feature of the
Sanchi and the amazing cave art of during his reign. Gandhara art is the rich carving,
Udayagiri and Khandagiri remind us elaborate ornamentation and complex
Kanishka erected a multi-storeyed
of the heights reached in sculpture. symbolism. The best specimens of
chaitya and monastery on the mortal
Human figures, dakshas-yakshas, Gandhara art are from Jaulian and
remains of the Buddha at his capital,
figures of birds and beasts, plants and Dharmarajika stupa at Taxila and from
Purushpur or Peshawar. Four specific
creepers were made in wonderfully Hadda near Jalalabad in modern
schools of art flourished in Mathura,
intricate patterns. Afghanistan. The tallest rock-cut
Sarnath, Amaravati and Gandhara
statue of Lord Buddha is also located at
during his rule. The Gandhara art, a
SATAVAHANA ART blend of Greek, Roman and Indian art
Bamiyan in modern Afghanistan and
Fine examples of architecture and dates back to the 3rd-4th century AD.
forms, reached its pinnacle of success
sculpture of the Satavahana period
constitute the Karle cave-chaitya,
during this period. The renaissance that CHARACTERISTICS OF GANDHARA
Buddhist caves in Nasik, Kalyan and
took off with Kanishka flowered in the SCHOOL OF ART
Gupta period. Below are the some characteristics of
elsewhere and the stupas of Amaravati.
Gandhara School of Art
GANDHARA SCHOOL OF ART
AMARAVATI ART (50 BC TO 500 AD) 1. Moulding human body in a
In Amaravati, situated in the Eastern realistic manner with minute
The region extending from Punjab to
Deccan, a different type of art form attention to physical features like
the borders of Afghanistan known as
evolved and flourished for nearly six muscles, moustache and curly
the Gandhara region was an important
centuries commencing from the 200-100 hair.
centre of Mahayana Buddhism upto
BC. Patronised first by the 2. Thick drapery, with large and bold
the 5th century AD. The region became
Satavahanas and later by the fold lines.
famous throughout the world since a
Ikshvakus and also by other groups
new school of Indian sculpture known 3. Rich carving, elaborate
(feudatories, officials and merchants),
as the Gandhara school developed ornamentation and symbolic
four periods of activity are easily
during that period. Owing to its expressions.
discernible.
1-302 GSP General Studies (Paper 1)
4. In Early Gandhara School blackish grey sandstone was The standing Buddha figures resembles the yaksha figures
used and in later school mud and lime plaster was used. and indicates the Kushana influence. The seated figures
5. The main theme was the Mahayana Buddhism. are in the padmasana posture. Here statues were made
Gandhara Art lost its pre-eminence and newer schools of with delighted facial expressions and graceful postures.
sculpture evolved, the finest example being the beautiful The Mathura school not only produced beautiful images of
Buddhist statue of Sarnath. A few other such wonderful the Buddha, but also of the Jain Tirthankaras and Gods
specimens are the Manjushree Avalokiteshwar statue at and Goddesses of the Hindu pantheon. Many scholars
Sarnath, the Bodhisattava at Sanchi and the bronze statue of believe that the Mathura School of Art, although of
Buddha found at Mathura. indigenous origin, was greatly influenced by the Gandhara
School of Art. The Guptas adopted the Mathura School of
MATHURA SCHOOL OF ART Art and further improvised and perfected it.
The Mathura School of Art flourished at the holy city of
Mathura during the reign of Kushana especially between CHARACTERISTICS OF MATHURA SCHOOL OF ART
1st-3rd century AD. Below are the some characteristics of Mathura School of
Art
CONTRIBUTION OF MATHURA SCHOOL
1. The Buddha image exhibit the spiritual feeling on his
It established the tradition of transforming the Buddhist
face which was largely absent in the Gandhara school.
symbols into the human form. Buddha’s first image can be
2. The Mathura school also carved out the images of
traced to Kanishka’s reign (about 78 AD). The earliest
Shiva and Vishnu along with their consorts Parvati
sculptures of Buddha were made keeping the Mathura Style
and Lakshmi.
yaksha prototype in mind. They were depicted as strongly built
with the right hand raised in protection and the left hand on 3. The female figures of yakshinis and apsaras of the
the waist. The figures produced by this school of art do not Mathura school were beautifully carved.
have moustaches and beards as in the Gandhara Art. These 4. Earliest statues of Buddha were made in Mathura
figures can be seen in the museum of Mathura. School of Art.
Difference Between Mathura Art, Gandhara and Amravati Art
Factor Mathura Art Gandhara Art Amravati Art
Origin • No foreign influence, however, later it • Stong Greek influence. • It was developed indigenously
cross fertilised with the Gandhara • It was based on Graeco-Roman norms. It is
school. known as Graeco-Buddhist School of Art.
• Its development took place • Assimilating various traits of Achaemanian,
indigenously. Parthian and Bactrian traditions into the
• Initially inspired by Yaksha images. local tradition.
• Inspired by Hellenistic features.
Material Used • Spotted red sandstone • Blue-grey mica schist/grey sandstone. • White marbles were used in the sculptures of
Amravati.
Image Features • Early period: Light volume, having a • Finer details and realistic images. • Predominantly Buddhist features.
fleshy body. • Buddha carved out in various mudras. • Satvahana ruler by emphasion individual
• Later period: Flashiness reduced. • Curly hair, anatomical accuracy, spatial features of Budha, sculphtures generally depit
• Not much attention to detalied depth and foreshortening. life stories of Budha
sculpting, Buddha is stout. • Buddha is sometimes thin.
• Hindu, Jain and Buddhist influence • Mainly Buddhist influence on images.
on images
Halo • The halo around the head of Buddha • Not decorated. Satvahana rulers
was profusely decorated. • The images are very expressive.
• Image are less expressive
Patrons • Kushana rulers • Kushana rulers • Less emphasis on individual features of Buddha,
sculptures greatly depict life-stories of Buddha
Area • In area around Mathura • In North-West, frontier area in Afghanistan • In and around Amravati and Nagarjunakonda
(Kandhar)

THE GUPTA ART building temples and books were being written on this
craft. Notable examples of such temples are Koteshwar,
This period is regarded as the most creative phase so far as art
was concerned. Old art forms improved and new ones evolved. Maninag, Sanchi and the Vishnu temple at Tigva, the
Buddhist, Jaina and Hindu cave-temples were carved out of Shiva temple at Bhumara, the Parvati temple at Kubir and
the Dashavatara temple at Deogarh.
rocks, the best examples being the ones at Ajanta, Ellora and
Udayagiri. MAIN FEATURES OF THE GUPTA ART
In the history of Indian art and architecture, the Gupta
TEMPLE SCULPTURE period occupies an important place. Both the Nagara and
Temple construction too attained new heights in the Gupta Dravidian styles of art evolved during this period.
period. For the first time bricks and rocks were used in
ART AND CULTURE 1-303
The temple at Deogarh near Jhansi and the sculptures in the
temple at Garhwas near Allahabad remain important specimens
PALLAVA ART
of the Gupta art. There was no influence of the Gandhara style. The Pallava architecture is known for their four specific
But the beautiful statue of the standing Buddha at Mathura styles
reveals a Greek style. The Buddha statue unearthed at Sarnath 1. Mahendra Style The influence of the cave style of
was an unique piece of the Gupta art. The Bhitari monolithic architecture is to be seen in an ancient pillar
pillar of Skandagupta is also remarkable. Metallurgy also made a engraved in the Ekambaranatha (Kanchipuram)
wonderful progress during the Gupta period. The craftsmen were temple.
efficient in the art of casting metal statues and pillars. 2. Mamalla Style The seven Pagodas are small
The gigantic copper statue of the Buddha, originally found at temples, each of which is chop out of a single rock
Sultanganj has been preserved now at the Birmingham museum, boulder. They lie near Mahabalipuram, founded by
and was about seven and a half feet height and nearly a ton in Narasimhavarman.
weight. The Delhi Iron Pillar of the Gupta period is still free from These monolithic temples are complete with all the
rust though remaining completely exposed to sun and rain for so details of an ordinary temple and stand as an
many centuries. undying testimony to the superb quality of the
Pallava art.
Schools of Temple Architecture 3. Rajasimha Style The most famous temple of this
There were three major schools of temple architecture Nagara, style is the Kailasha temple, Kanchi. It has a
Dravidian and Vesara. pyramidal tower, a flat-roofed mandapam and a
Nagara Architecture
series of cells surround it resembling rathas. This
Nagara temples have three distinct features
style is a very elaborate one foreshadowing the
ornate Chola architecture.
¢
In plan, the temple is a square with a number of graduated
projections in the middle of each side giving a cruciform shape with 4. Aprajita Style This is more ornate resembling
a number of re-entrant angles on each side. the Chola architecture. A few temples built in the
¢
In elevation, a Shikhara i.e. tower gradually inclines inwards in a style are found at Dalavanur. The noteworthy
convex curve, using a concentric rotating squares and circles principle. feature of some shrines is that they are adorned by
¢
The projections in the plan are also carried upwards to the top of the beautiful life-like images of Pallava kings and
Shikhara and thus, there is strong emphasis on vertical lines in elevation. their queens.
Dravidian Architecture IMPORTANT FEATURES OF THE PALLAVA
Dravidian style temples consist almost invariably of the four following ARCHITECTURE
parts, differing only according to the age in which they were executed ˜
The Pallava architecture shows the transition from
¢
The principal part, the temple itself, is called the Vimana (or the rock-cut architecture to the stone built temples.
Vimanam). It is always square in plan and surmounted by a pyramidal ˜
The earliest examples of the Pallava art are the
roof of one or more stories; it contains the cell where the image of
the God or his emblem is placed. rock-cut temples of the 7th century AD, while the later
¢
The porches or Mandapas which always cover and precede the door examples are of the structural temples built in the 8th
leading to the cell. and the 9th century.
¢
Gopurams, which are the principal features in the quadrangular ˜
The rock-cut reliefs of the Pallavas are the earliest
enclosures that surround the more notable temples. surviving royal portraits after the Kushana images.
¢
Pillared halls or Chaultris–properly Chawadis used for various purposes ˜
Mahendravarman I introduced the rock-cut
and which are the invariable accompaniments of these temples. temples. This style of Pallava temples are seen at
Vesara Architecture places like Mandagappattu, Mahendravadi,
Vesara architecture have four distinct features Mamandur, Dalavanur, Tiruchirappalli, Vallam,
¢
This is also known as Chalukya style or Karnataka style. Siyamangalam and Tirukalukkunram.
¢
It has features of both Nagara and Dravidian styles. ˜
The five rathas, popularly called as the
¢
It has two principal components Vimana and Mandapa joined by Antrala. Panchapanadava rathas, signifies five different
¢
Ladkhan temple of Aihole is an example of this style. styles of temple architecture. The mandapas contain
beautiful sculptures on its walls. The most popular of
these mandapas are Mahishasuramardhini Mandapa,
KACHCHHAPAGHATA ART Tirumurthi Mandapa and Varaha Mandapa.
The Kachchhapaghata dynasty ruled part of north-western India
between 10th to 12th century AD. They continued the tradition of IMPORTANT PALLAVA TEMPLES
temple building famished during the reign of Chandela dynasty. Mandagappattu Temple This is the first Pallava
Some of their architectural contributions are rock-cut temple, built by Mahendra I in the early 7th
˜
Sasbahu temple, Chausath Yogini temple (Morena), Kakanmath, century. An inscription on the temple calls it the
sihoniya are some of temples built during this reign. Laksitayatna and dedicates it to Brahma, Isvara, and
Vishnu. The facade of the rectangular mandapa has
˜
Chausath Yogini temple is dedicated to Lord Shiva and its
four massive pillars and is flanked by niches
circular design has given rise to popular belief that it was
containing large dvarpalaka images carved in relief.
inspiration behind Indian Parliament building.
1-304 GSP General Studies (Paper 1)

Kailasanatha of Kanchipuram The statues and small structures


temple is credited to the initiative uncovered on the shore like the large
HOYSALA ART
and enterprise of the Pallava ruler stone lion left sitting uncovered on After succeeding the Chalukyas, the
Narasimhavarman II or Rajasimha Mahabalipuram’s beach. Hoysala rulers evolved a new style of
(also known as Ajiranakanta, architecture.
Ranadhira and a Kshatriya CHALUKYAN ART The most notable temples of this period
Simheshvara) who reigned between The Chalukyan were great patrons of are those constructed in Belur, Halebid
690-728 AD. The Kailasanatha art. The Vesara style in the building of and Somnathpur.
temple is the finest structural project structural temples was developed by
of the Pallavas. It looks as if a chariot MAIN FEATURES OF THE TEMPLES
them. However, only under the
from heaven has descended on the ˜
The base of the temple is star shaped
Rashtrakutas and the Hoysalas, did the
Earth. and not square-shaped.
Vesara style reached its culmination.
Vaikuntha Perumal Temple This
˜
Open mandapa, closed by a compound
The structural temples of the
temple is situated at Kanchipuram in wall.
Chalukyas exist at Aihole, Badami and
Tamil Nadu and was built in the 8th Pattadakal. Cave temple architecture
˜
Miniature shrines within the
century AD. It was built by the compound wall.
was also famous under the Chalukyas.
Pallava emperor, Nandivarman Their cave temples are also found in
˜
Temple wall profusely decorated with
Pallavamalla in the 8th century AD. Ajanta, Ellora and Nasik. The best royals, sages, dancers and even
This temple is famous for its unique Kamasutra figures.
instance of Chalukya paintings can be
architectural grandeur and religious Seven-storeyed Gopuram.
seen in the Badami cave temple and in ˜

importance.
the Ajanta caves. The reception given ˜
Two vimana and three vimana
This temple is one of the 108 Divya to a Persian embassy by Pulakesin II is structure- made from single granite.
Desam dedicated to the Lord Vishnu. depicted in a painting at Ajanta.
This temple is also called the Tiru IMPORTANT TEMPLES
The Chalukya temples may be divided
Parameswara Vinnagaram. KIRTINARAYAN TEMPLE
into two stages: The first stage is
The Shore temple at Mamallapuram represented by the temples at Aihole This temple is located in Talakkad
was built during the reign of the and Badami. Among the seventy near Mysore, in Karnataka. It was
Pallava King Rajasimha (c 700-728). temples found at Aihole, four are built by the Hoysala king
Its three sanctuaries are dedicated to important Vishnuvardhana. This temple is
Vishnu and Shiva. known for a single granite vimana, a
1. Ladh Khan temple is a low,
The Shore temple is also flat-roofed structure consisting of huge statue of Nandi open mandapa
acknowledged for being the first stone a pillared hall. garbhagriha. In recent years, the ASI
structure made by the Pallavas. has restored this temple.
2. Durga temple resembles a Buddha
Before this, the monuments used to
Chaitya. SOMNATHPUR TEMPLE
be carved out of the rocks or stones.
This group of temple is an UNESCO 3. Huchimalligudi temple at Aihole. It was built by Vinaditya Ballal in
World Heritage site. 4. The Jain temple at Meguti. about 1043 AD and is the earliest of
The Seven Pagodas This term is The second stage is represented by the this type. The temple though small was
closely associated with the Shore temples at Pattadakal. There are ten exquisitely carved with three
Temple of Mahabalipuram. temples here, four in the Northern pyramidal vimanas surmounting the
According to a popular belief, once style and the remaining six in the three shrines.
upon a time six other temples stood Dravidian style.
HOYSALESWARA TEMPLE
along with the now famous Shore The Papanatha temple is the most It is located in Halebid in Hassan
temple on the shores of notable in the Northern style. The district, Karnataka. Built of grey
Mahabalipuram’s rock strewn Sangamesvara temple and the soapstone, best suited for fine carving,
beaches. In the late 2004, during the Virupaksha temple are famous for their each of the temples has star shaped
tsunami that struck the coast, the
Dravidian style. vimanas with projections on its three
sand deposits of around 500 m from
The Virupaksha temple is built on the sides.
the Shore temples were gulped by the
sea. During this time, tourists and model of the Kailasanatha temple at CHENNA KESAVA TEMPLE
residents were reported to have seen Kanchipuram. It was built by one of the It is located in Belur and was built by
a long, straight row of large rocks queens of Vikramaditya II. Vishnuvardhana of the Hoysala
emerge from the water just before the Sculptors brought from Kanchi were dynasty in 1117 AD to commemorate a
waters rushed back again. The employed in its construction. It is victory won over the Cholas its Talkad
tsunami also made some immediate, located in Hampi, Karnataka on the in 1116 AD.
lasting changes to the coastline, banks of the Tungabhadra river.
which left a few previously covered
ART AND CULTURE 1-305

style in South India during the


Temples of Odisha Rajput period. Both sculpture and
VIJAYANAGARA ART
The temples of Odisha are of the Nagara architecture attained a high degree of The Vijayanagara style of architecture
style. The main temple, known as the evolved in the 14th century AD and
excellence.
Vimana or the Deula, is the sanctum reached its culmination during times
enshrining the deity. And the porch or the IMPORTANT FEATURES of Krishnadevaraya and
Jagamohana is a congregation place for Achyutadevaraya.
the devotees. Salient features of temples
˜
Famous temples of Khajuraho
of Odisha are as follows. have been built in Chandela style. The architecture reflected the power of
¢
Interior walls kept plain but interior walls ˜
The pattern and style of their rulers. Under them, the temples
decorated lavishly. architecture in Chandela dynasty became an elaborate institution
¢
Iron girders are used for supporting roof have a rugged simplicity and are containing a large number of structures.
and pillars are not used in the porch. quite distinct from the works of the The Vijayanagara-era architecture can
¢
Rekhadeuls They are almost vertical later Pratiharas. Indeed, the be broadly classified into Religious,
roofs which curved inwards sharply,
Chandela architecture is different Courtly and Civic architecture.
similar to Shikharas.
Boundary walls are constructed in South from their sandstone successors at
¢

Khajuraho.
MAIN FEATURES
Indian Dravidian style. They surround
the temple.
˜
The Vijayanagara architecture is a
˜
The temple complex at Khajuraho
Square–shaped ground plan for the harmonious combination of the
¢
was built by Chandela rulers
main temple. Chalukya, Hoysala, Pandya and the
between the 10th and 11th
Chola style which evolved from prior
Important Temples centuries in the Bundelkhand
empires in earlier centuries. It is also
Jagannath Temple, Puri region of Madhya Pradesh. Most
influenced by the later Deccan and
Belonging to the 11th century and important among them is the
Dravidian styles. It uses granite as its
enjoying the honour of being one of the Kandariya Mahadeva temple.
main building material.
char dhams, the holy sight of Lord ˜
Decorated carvings on interior as
Jagannath, accompanied by Subhadra and ˜
Vijayanagara temples are
wells as exterior walls, with
Balabhadra raises loads of joy among the characterised by ornate pillared halls
devotees. It was built by Anantverma sculptures based on theme of
and rayagopurams, monumental
Chodganga. The structural design of the Kamasutra.
towers adorned with life-sized figures
temple can be found to be in a pyramidal ˜
Most of the temple are North or of Gods and Goddesses that stand at
shape. East facing. the entrance of the temple.
Konark Sun Temple ˜
Mountain range type of impression ˜
The Kalyan mandapa or marriage
Built in the 13th century by King is made because subsidiary shrines hall which is an open pavalion is
Narasimhadeva, the temple is designed in also have shikharas.
the shape of a colossal chariot with seven
another feature of temple
˜
High platform is preferred for architecture. Its interior surrounded
horses and twelve wheels, carrying the
Sun God, Surya, across heavens.
building temples. by impressive colours, contains a
platform in the centre for
The Lingaraja Temple SOLANKI ART performance of sacred dance. Hall
The great Lingaraja temple, believed to
This art form was developed under was used for ceremonies involving
have been built around 1000 AD, is a later
product of this revivalist movement and Solanki rulers in the parts of Gujarat symbolic marriage of the temple’s
has been acclaimed by many as the finest and Rajasthan. divinity to his consent.
example of Nagara style. It stands in a ˜
Vitthala Swamy temple and Hajara
cluster of sixty-five smaller shrines in a IMPORTANT FEATURES
Rama temple of Hampi are examples
spacious compound measuring 520 ft by ˜
Variety of materials were used like
of this style.
465 ft and its mighty tower (the vimana) sandstone, black basal and soft
dominates the landscape for miles around. marbel.
˜
The courtly architecture of
Vijayanagara is generally made of
˜
Temple are east facing.
mortar mixed with stone rubble and
CHANDELA ART ˜
Torans : Decorated arched gateway,
often shows secular styles with
a special feature of this art form.
The Rajput rulers had a longing sense of Islamic-influenced arches, domes
˜
Temple wallas are simple in nature,
beauty in art and architecture which is and vaults.
devoid of interior and exterior
seen in the artistic excellence of their ˜
The Vijayanagara School of Painting is
carvings.
temples, forts and palaces. renowned for its frescoes of Hindu
˜
Step tank : This is also a peculiar
The Nagara style of architecture feature, at the steps of the tank, Gods and Goddesses and scenes from
developed in North India and Vesara temples are constructed. e.g. Hindu mythology on temple walls and
style in Upper Deccan and the Dravidian Modhera Sun Temple, Konark. ceilings.
1-306 GSP General Studies (Paper 1)

IMPORTANT TEMPLES ˜
The artistic builders of the Rajputs The pendant of the dome is a perfect
designed major architectural styles gem; where it drops from the ceiling it
VITTHALA TEMPLE AT HAMPI
which are located in cities like looks like a cluster of half open
One of the most splendid of temples at Jaisalmer, Udaipur, Jaipur and lotuses.
Vijayanagara is the Vitthala temple, Jodhpur.
near the Tungabhadra river and is ˜
The most significant architectural RASHTRAKUTA ART
also considered as the most ornate of designs in Rajasthan include Jantar The rulers of the Rashtrakuta dynasty
the Vijayanagara temple. The temple Mantar, Dilwara Temple, Lake Palace were great patrons of art and
is dedicated to Vishnu worship. Hotel, Chittorgarh Fort and Jaisalmer architecture and were great builders
VIRUPAKSHA TEMPLE AT HAMPI Havelis. and a half century of the Rashtrakuta
rule, witnessed a very high
The Virupaksha or the Pampapathi IMPORTANT TEMPLES achievement in the fields of sculpture
temple is the main center of DILWARA TEMPLES AT MOUNT ABU and architecture.
pilgrimage at Hampi. It is fully intact
and incorporates some earlier The Dilwara Temples have been described Now after more than a millennium,
structures. This temple has three as a dream in marble. whatever remains, is in a dilapidated
towers; the Eastern tower rises to a Mount Abu, a popular hill station, located condition. Even then, the Ellora and
height of 160 ft and is nine tiered. It in Rajasthan is on the Ahmedabad, Jaipur Elephanta caves provide splendid
dates back to the first half of the 15th and New Delhi railroad, off the Abu Road, examples of that era which excelled in
century and was renovated in the the nearest railhead. Other attractions carving out huge monuments and
16th century by Krishnadevaraya. here include the Nakki lake, the Adhar monolithic sculptures.
Devi temple, the Achalgarh Shiva temple In Ellora (Maharashtra), we have
SIMHACHALAM TEMPLE and the Gau Mukh Shiva temple. rock-cut cave temples of all faiths then
Kulottunga Chola I of Tamil Nadu, flourishing in India Buddhist, Jain,
made endowments to this temple, as THE JAIN TEMPLES AT MOUNT ABU
Shaiva and Vaishnava.
evidenced from the inscriptions It was built in the Nagara style and is
among the finest monuments of India. The Out of the 34 caves, 12 are Buddhist,
dating back to the year 1087. The
first of these ornate temples dates back to (in the South) 17 are Hindu
Vengi Chalukyas of Andhra Pradesh
1032 CE, approximately the period in (Vaishnava, Shaiva and Shakta) and 5
renovated the original shrine in the
which the grand Brihadeeswara temple at caves on the Northern side are Jain.
11th century. Much of the structure
stiu stands intact. Thanjavur in Tamil Nadu (South India) MAIN FEATURES
was built in the Dravidian style of ˜
The features of the Rashtrakuta
RAJASTHAN ART architecture. sculpture reflect the Dravidian style.
The Rajasthan architecture THE VIMALA VASAHI TEMPLE ˜
The features of Rashtrakuta
significantly depended on the Rajput This is the earlierst and the most sculpture are apparent in the
architecture school which was a important of the temples here and is rock-cut architectures.
mixture of the Mughal and the Hindu dedicated to the first Jain Tirthankara, ˜
The caves at Elephanta and Ellora
structural design. Grand Havelis, Adi Nath. It stands in an open courtyard nestled in Maharashtra bear
astonishing forts and elaborately defined by 58 subordinate cells which testimony to the artistic wizardry of
carved temples constitute the vital contain small icons duplicating the saint’s the Rashtrakutas.
portions of a architectural heritage of image in the main shrine. ˜
The sculptures that adorn the
Rajasthan. The Rajputs carries the monuments of the Rashtrakutas
credit of creative builders. Few of The plan of the temple resembles that of
the Kashmiri Sun temple at Martand. have been drawn from Hindu
most striking and splendid forts along mythologies.
the places with parched Aravali land Elaborately columned porticoes surround
the main shrine and front the cells lining ˜
The sculpture of Kasivisvesvara
clearly depicts the history of temple has also been built by
Rajasthan’s celebrated heritage. the courtyard. The entire temple is carved
out of white marble. following the vocabulary of the
MAIN FEATURES Rashtrakuta architecture.
THE TEJAPALA TEMPLE ˜
The figure sculptures of dancers,
˜
Rajasthan architecture arouses
It resembles the architectural plan of the musicians, Gods and Goddesses from
from the Rajput school of
Vimala Vashi temple, (built nearly 200 Hindu Puranas, nymphs and others
architecture, a perfect mixture of
years earlier), it nevertheless stands as are common on the outer walls of
Hindu and Mughal structural
the last of the monuments built in the these rock-cut caves.
pattern.
Solanki style, which came to an end with
˜
The state of Rajasthan hosts a few of
the occupation of Gujarat by the Muslims IMPORTANT TEMPLES
the splendid palaces and forts of the
towards the end of 13th century. THE KAILASHA TEMPLE
whole world. Ornamented havelis,
elaborately carved temples and also The striking feature of the Tejapala It is a marvelous piece of rock
magnificent forts are a section of the temple is its dome which stands on 8 architecture and sculpture. It was
Rajasthan’s architectural heritage. pillars. built by the Rashtrakuta King
Krishna I in 8th century AD.
ART AND CULTURE 1-307

A complete hillside has been separated It has been regarded as a ‘landmark in


from a range of mountains and a huge the evolution of building art in South
NAYAK ART
temple has been excavated out of it. India’ and its vimana as ‘touchstone of The culmination of the Dravidian style
The main temple is supported on the Indian architecture as a whole.’ is to be seen in the period of the Nayak
backs of elephants. rulers of Madurai who continued the
THE TEMPLE OF building style and technique of the
THE CAVE TEMPLES GANGAIKONDACHOLAPURAM Pandyas and improved on it.
These are on the island of Elephanta It was built by Rajendra I, and was ˜
The Nayak dynasty emerged in South
near Mumbai and were also built by completed in 1035. Its 53m vimana India after the downfall of the
the Rashtrakuta rulers. The main cave (sanctum tower) has recessed corners Vijayanagara empire in 1565, when
temple belonging to the Brahminical and a graceful upward curving the Nayak military Governors
group of cave temples is dedicated to movement, contrasting with the declared independence and ruled from
Shiva and is noted for its fine straight and severe tower at the 16th–18th century.
sculpture. The temple has a large Thanjavur. It has 6 pairs of massive, ˜
The Nayak architectural style was
mandapa supported on 20 pillars on monolithic dvarapalas statues characterised by elaborate hundred-
the periphery. Flights of steps lead to it guarding the entrances and bronzes of and thousand-pillared mandapas, the
from the courtyards in front and at the remarkable beauty inside. high gopurams with painted
back. The temple is famous for the stucco statues on the surface and long
THE AIRAVATESVARA TEMPLE AT corridors.
huge Trimurti depicting the three
DARASURAM
aspects of the Supreme Creator, ˜
The Nayak dynasty is also known for
Preserver and Destroyer. It was built by Rajaraja II, features a its mural and wall paintings. Tanjore
24-m vimana and a stone image of paintings from this time are known
CHOLA ART Shiva. The temples testify to the for their surface richness, vivid
Cholas brilliant achievements in colours and compact composition.
Many temples had been built by the
architecture, sculpture, painting and
Chola kings throughout their
kingdoms. The temples of the early
bronze casting. Nataraja Statue fuses MAIN FEATURES
in a single image of Lord Shiva’s role as ˜
The hundred pillared mandapas.
Cholas are found in large number in
the creator, preserver and the The lofty gopurams embellished with
the former Pudukkottai region. These ˜

destroyer of the universe and conveys figures in their minute detail.


Chola temples reveal the gradual the Indian conception of the
evolution of the Chola art and never-ending cycle of time. Although, it
˜
The closed Prakramas with huge
architecture. The Chola kings earlier appeared in sculpture as early as the pillars on either side.
built brick temples. Later, they built 5th century, its present, world famous ˜
The beautiful corbel brackets as in
stone temples. form evolved under the rule of the Ramanatha temple in Rameshwaram.
Cholas. Shiva’s dance is set within a ˜
Full-sized figures of animals and
MAIN FEATURES
flaming hole. riders on rearing horses in the
˜
The dvarapalas or guardian figures, Srirangam temple.
at the entrance to the mandapa or The God holds in his upper right hand
hall which started from the Pallava the damaru (hand drum that made the IMPORTANT TEMPLES
period became an unique feature of first sounds of creation). His upper left RAMESHWARAM RAMANATHASWAMY
the Chola’s temples. hand holds agni (the fire that will TEMPLE
destory the universe). With his lower
˜
The Dravidian style got fully Rameshwaram temple is situated on
right hand, he makes the
developed after a transition from the the Island of Rameshwaram, off the
abhayamudra (the gesture that allays
rock-cut structures of the Pallava Sethu coast of Tamil Nadu and is
fear).
period. reached via the Pamban bridge across
The dwarf-like figure being trampled the sea. The huge temples are known
IMPORTANT TEMPLES by his right foot represents apasmara for its long ornate corridors, towers and
BRIHADEESWARA TEMPLE purusha (illusion, which lead the 36 theerthams.
The celebrated Shiva temple at mankind astray). Shiva’s front left
Thanjavur, appropriately called the hand, pointing to his raised left foot, PANDYAN ART
Brihadeeswara and the signifies refuge for the troubled soul. Sometime during the 10th century, the
Daksinameru, is the grandest The energy of his dance makes his hair Cholas raided and captured Madurai,
creation of the Chola emperor fly to the sides. The symbols imply thus ending the golden age of the
Rajaraja (985-1012 AD). It was that, through belief in Shiva, his Pandya rule. However, by 1223 CE, the
inaugurated by the king himself in his devotees can achieve salvation. Pandyas returned to Madurai’s throne
19th reignal year (1009-10 AD) who Krishnadevaraya, the Vijayanagara and the growth of Madurai continued
named it after himself as Rajesvara monarch visited this temple in the year afresh. The Pandya kings took a keen
Peruvudaiyar, architecturally, it is 1516, as seen from inscriptions here. interest in art and architecture, as well
the most ambitious structural temple There are as many as 525 inscriptions as the development of the Tamil
built of granite. in this temple. language.
1-308 GSP General Studies (Paper 1)

Madurai or ‘the city of nectar’ is the oldest This invasion marked the very ˜
Cementing agent used was mortar
and second largest city of Tamil Nadu. It beginning of the Islamic architecture. not stones and wood.
is located on Vaigai River. Madurai once The introduction of the glazed tile ˜
They avoided human and animal
was the capital of Pandyan rulers. The decoration by the Arabs contoured the figures on their buildings as Islam
Pandyan king, Kulasekhara had built a initial architectural development of prohibits it.
gorgeous temple around his lotus shaped the Islamic architecture. The second ˜
They replaced sculptures with
city. It has been a centre of learning and attack occurred in the second half of calligraphy as a means of
pilgrimage, for centuries. the 10th century, much later than the decoration.
first attack.
MAIN FEATURES ˜
Arabesque Method It is an
Mahmud of Ghazni attacked Punjab ornamental style which is depicted
˜
Sanctum Sanctorium- Garbhagriha. from the way to Lahore, where a by a stem which continuously gets
˜
Huge Gopurams. Viceroy occupied an important group of split regularly producing a series of
˜
High outer walls. palaces and government buildings for branching patterns.
˜
High Pyramidal Vimanas full of his purpose. ˜
Jaali Works Their buildings have
sculptures and decorations. Later in 1192 AD Muizzuddin of Ghor intricate Jaali works, it signifies the
˜
Pillared halls. (Muhammad Ghori) defeated Rajput importance of light in Islamic
˜
Antaralya—the passage between the King Prithviraja III in Second Battle of religion.
hall and Garbhagriha. Terain and laid the foundation of ˜
Use of Water In the premises and
Sultanate in Northern India. This
IMPORTANT TEMPLES marked beginning of a new era in
courtyard water is used for various
MEENAKSHI TEMPLE reasons like religious (for cleaning),
Indian architecture.
cooling effect, decorative purpose
˜
It is the most elegant example of the
ect.
Dravidian architecture. FEATURES OF ISLAMIC ART ˜
Charbagh Style This style was
˜
This temple was originally built by the The ruins of ancient brick and
introduced by Islamic rulers in
Pandyan King Kulasekara but it was timbered structures provide the clue of
which a square block was divided
plundered and destroyed by Sultanate’s the style of buildings that then
into four adjacent identical gardens.
Muslim Commander Malik Kafur. It succeeded. Wooden doors and doorways
along with the ornamental elements
˜
Pietra Dura This technique was
was rebuilt by the Nayakar ruler
persisted and were incorporated not used for the inlay of precious stones
Vishwanath Nayakar in the 16th
infrequently in the Indo− Islamic art and gems into the stone walls.
century.
which developed shortly afterwards. ˜
Foresightening Technique. This
˜
It is an excellent architecture, a
Muslim structures can be classified technique makes the inscriptions
masterpiece which is the centre of
under two types–religious and secular. appear of the same size from any
activity for the promotion of art and
Religious structures are mosques, place.
culture in Madurai.
tombs and mausoleums. Secular ones ˜
They used tomb in their buildings
˜
It is the chief attraction of the city and
are forts, palaces, gateways and extremely for both strengthening as
pilgrims gathered here in the search of
gardens. well as decorating.
spirituality and peace.
The salient features of Muslim
˜
There are wonderful views of stone
architecture are as follows
THE SULTANATE
carved sculptures of different deities on PERIOD ART
the wall of the temples.
˜
Mosques An open courtyard with a
pillared verandah around, a The Delhi or the imperial style of
˜
The temple is dedicated to the Goddess Indo Islamic architecture flourished
sanctuary near the wall facing Mecca,
Meenakshi (Parvati) and to the Lord between 1191-1557 AD and covered
a pulpit and a minaret above the wall
Sundareswara (Shiva). the Muslim dynasties viz the Slaves
to summon people to prayer. The
sanctuary has a large dome above it. dynasty (1191-1246 AD), Khiljis
INDO-ISLAMIC ART AND ˜
Bulbous domes, open courtyards, (1290-1320 AD), Tughlaq (1320-1413
AD), Sayyids (1414-1444 AD) and the
ARCHITECTURE large buildings, tall and slender
Lodis (1451-1557 AD).
minarets (minar).
The beginning of Islamic architecture is The first Islamic Sultanate
˜
Use of coloured stone and glazed
the tale of the Muslim invasion in India structures were built of disparate
tiles.
which to a great extent reshaped the dismantled pieces of Hindu temples,
whole architectural element of India. The ˜
Arcuate Style They used arch and
after which came an era of carefully
earliest attack in India took place in the dome structures in their buildings/
planned structures and precincts,
8th century, when much of the lower architecture. It is known as arcuate
later assimilating and incorporating
Indus territories came under the power style of architecture, it replaced the
Hindu elements and workmanship.
of the Caliphs of Baghdad, the Arab trabeate style.
invaders.
ART AND CULTURE 1-309

IMPORTANT MONUMENTS BENGAL ARCHITECTURE MUGHAL ARCHITECTURE


QUWWAT-UL-ISLAM MASJID The Adina and Pandua mosques are the The art and architecture of this period
It was built by Qutubuddin earliest architectural examples in have a characteristic of Indo-Islamic-
Aibak and it is considered as the Bengal. The tomb of Akhi Surajuddin, Persian style that flourished on the
earliest mosque in India. It was the Kotwali Darwaza, the Dakhil Indian sub-continent during the Mughal
built by the material obtained after Darwaza and the tomb of Sultan empire (1526-1857).
destroying around twenty-seven Jalaluddin Mohammed Shah (1414-1431 This new style combined elements of
temples and many Hindu features AD), known as the Eklakhi Tomb, Islamic art and architecture, which had
can be seen in this mosque. served as prototypes for the subsequent been introduced to India during the Delhi
The mosque has beautiful Islamic Islamic architecture of Bengal. The other Sultanate (1192–1398) and had produced
calligraphy, the arabesque designs important buildings of Bengal include great monuments such as the Qutub
and pillars with pre-Islamic Hindu Tantipara Masjid ( 1475 AD), Chamkatti Minar, with features of Persian art and
motifs. Later, its construction was Masjid (1475 AD), Lotan Masjid (1480 architecture. Mughal monuments are
extended by Iltutmish. AD), and Chota Sona Masjid (1510 AD). found chiefly in Northern India, but
there are also many remains in Pakistan.
QUTUB MINAR JAUNPUR ARCHITECTURE
The construction of this was started BABUR
Under the Sharqi dynasty, Jaunpur The Mughal dynasty was established
by Qutubuddin Aibak in 1192 became a great centre of art, culture and
which was eventually completed by with the crushing victory of Babur at
architectural activity. During the rule of Panipat in 1526. During his short
Iltutmish in 1230. The Qutub Shamsuddin Ibrahim (1402-1436 AD)
Minar, built to commemorate the five-year reign, Babur took considerable
several palaces, mosques, tombs and interest in erecting buildings, though few
entry of Islam, was essentially a other buildings came up, the most
victory tower, decorated with have survived. He constructed the
prominent being the Atala Masjid built in mosque at Rohilakhand and Panipat.
several calligraphic inscriptions. 1378.
The diameter of the Qutub Minar is Later, other important buildings were HUMAYUN
14.32m at the base and about produced that include a Khalis Mukhlis Humayun’s era was very turbulant even
2.75m at the top. It measures a Masjid (1430 AD), Jhangiri Masjid (1430 then he built some famous buildings like
height of 72.5m and contains a AD) Lal Darwaja Masjid (1450 AD) and Mosques of Agra and Fatehabad and
spiral staircase of 379 steps. the Jami Masjid (1470 AD). most importantly Din-Panah city which
ADHAI-DIN-KA-JHOPRA is also known as Fifth Delhi.
It was also built by Qutubuddin MALWA ARCHITECTURE AKBAR
Aibak in 1153 AD and it was a This is a very unique and adaptive In architecture, the first great Mughal
Sanskrit school converted into architecture form of 15th century. They
monument was the mausoleum of
mosque in 1198 AD. It is located in used stones and marbles of different
Humayun, erected during the reign of
the Ajmer district of Rajasthan. colours in their buildings and decorated
Akbar (1556–1605).
with arches and pillars. They used locally
ALAI DARWAZA available materials. Few important The tomb, which was built in the 1560s,
It was built by Allauddin Khilji. features of their architecture are was designed by a Persian architect
It is well decorated and served as presence of large windows, use of Mirak Mirza Ghiyas. It was set in a
an entrance gateway to the mosque artificial reservoirs and no use of minars. garden at Delhi, and has an intricate
at the Qutub complex, marking the Some of their prominent buildings are ground plan with central octagonal
evolution of another innovative Jahaz Mahal, Raani Roopmati Pavillion chambers, joined by an archway with an
feature in the Indo-Islamic etc. elegant facade and surmounted by
architecture. Cupolas, Kiosks and Pinnacles. At the
BIJAPUR ARCHITECTURE same time Akbar was building his
fortress-palace in his capital, Agra.
PROVINCIAL This style developed under Adil Shah,
one of the prominent kings of Bahamani Native red sandstone was inlaid with
ARCHITECTURE dynasty/kingdom. He built various white marble, and all the surfaces were
ornately carved on the outside and
tombs, palaces and mosques.
The Provincial style of architecture sumptuously painted inside. Akbar went
encompasses the architectural Some important features of their
buildings are bulbous dome, use of on to build the entire city of Fatehpur
trends and developments noticed in Sikri (City of Victory) in which extensive
cornice, three arched facade and use of
different provincial capitals in use was made of the low arches and
iron clamps for providing support to the
India, but specifically in the bulbous domes that characterise the
ceilings. Some of the prominent buildings
following Architectural forms Mughal style.
are Gol Gumbaj, Bijapur Fort etc.
1-310 GSP General Studies (Paper 1)

Built in 1571 the choice of the site of to erect huge buildings and therefore ˜
The Chota Imambara or Husainabad
Sikri reflected Akbar’s gratitude to a he has the credit of constructing very Imambara, built by Muhammed Ali
Muslim saint at Sikri for the birth of few buildings such as Bibi Ka Shah as his tomb and the Shahnajaf
his son. Courtiers soon followed suit Maqbara (Rabia Durani). It is an Imambara, housing the tomb of
and built homes surrounding the imitation of Taj Mahal at Aurangabad, Nawab Ghaziuddin Haidar are other
palace and mosque. The new city Maharashtra. note-worthy buildings following this
became the capital of the empire, but in Apart from this the Badshahi Mosque style of architecture.
1585 it was abandoned. at Lahore and the Moti Masjid at Lal ˜
Major General Claude Martin
Quila, Delhi are other two specimens (1735-1800) built a large and
JAHANGIR pretentious building at Lucknow, then
of his contribution towards Mughal
Though Jahangir was more interested known as Constantia, which was one
architecture.
in paintings. Under his reign more of the first large buildings of the
emphasis was given to album and
individual portrait than illustrated
POST-MUGHAL European style built in North
India. This building, which had
manuscript. Some important buildings ARCHITECTURE several Palladian elements, set the
were made during his rule. First of After Aurangzeb, there is a witness of tone for the development of a hybrid
such buildings is Mausoleum of Akbar Zinat-un-Nissa Begum, daughter of style of architecture in Lucknow
in Sikandara near Agra, which does Aurangzeb, who built the Zinat-ul- depicting a degenerative combination
have a dome and has features of Masjid in Daryaganj, Delhi which of Mughal and Gothic styles.
Buddhist architecture. It was started consisted of large courtyard above a ˜
The best examples of this style of
by Akbar himself. Another important number of rooms in the basement. architecture are the Chattar Manzil
building is tomb of I’timed-ud-Daulah built by Nawab Naziruddin Haider
The mosque had two gateways at
built by Noorjahan, in this building for and the Kaiser Bagh built by Nawab
opposite directions; the Southern one
first time Pietra Dura style was used. Wajid Ali Shah between 1845 and
serving as the main basement.
1850. The Chattar Manzil is an
SHAHJAHAN This masjid although was based on the
imposing building, whose main
It was Shahjahan (1628–58) who architectural style of Jama Masjid, yet attractions are the underground
perfected Mughal architecture and it failed to imitate the magnificence rooms and a beautiful dome
erected at Agra, its most noble and and coherence of the latter. Other surrounded by a gilt umbrella.
famous building, the tomb of his specimens of this type of poor
favourite wife, which is known as the caricature of Mughal architecture are PUNJAB STYLES
Taj Mahal. A huge white marble the Sunehri Masjid, built by Nawad
In Punjab, a discrete style of
building of simple, symmetrical plan, it Qudsiya Begum in 1751, Safdar-Jung
architecture developed under the
is inlaid with colourful semiprecious tomb of Delhi built by Mirza Muquim
influence of the Mughal style. It was
materials and is set in an equally Abdul Mansoor Khan etc. distinguished by certain primordial
beautiful and symmetrical garden. features like the multiplicity of chattris
The Taj Mahal continues the tradition AVADH (OUDH) STYLE or kiosks, the use of fluted dome
of Mughal garden tombs, of which During the later half of the 18th generally covered with copper or
Humayun’s tomb was the first. century and the first half of the 19th brass-gilt and enrichment of arches by
Shahjahan established (1638) Delhi as century, a distinct form of architecture numerous foliations. This architecture
his capital and built there the famous came into existence under the Nawabs was best manifested in the construction
Red Fort, which contained the imperial of Avadh (Oudh), whose first of the Golden Temple at Amritsar in
Mughal palace. Paintings also exposition was noticed in the form of 1764 built by the fourth Sikh Guru
flourished during Shahjahan’s reign. Safdar Jung’s tomb, built in the Ramdas.
Portraiture was most highly developed honour of Safdar Jung (1739-1753),
at his sophisticated court and ink who was the nephew of the first JAIN ART
drawings were of high quality. Nawab of Oudh. The contribution of Jain art to the
AURANGZEB mainstream art in India has been
MAIN FEATURES considerable. Every phase of Indian art
Under the orthodox Aurangzeb ˜
The city of Lucknow witnessed large is represented by a Jain version and
(1659–1707) the decline of the arts building enterprises under Nawab each one of them is worthy of
began, although his ornate Pearl Asaf-ud-Daulah (1775-1795). meticulous study and understanding.
Mosque (1662) at Delhi is worthy of The most representative
˜
Jain architecture cannot be accredited
mention. During his reign, the Mughal architectural achievement of this with a style of its own, for in the first
academy was dispersed. period is the Bara Imambara built place, it was almost an offshoot of
Many artists then joined the Rajput by the Nawab in 1784. Absence of Hindu and Buddhist styles. In the
courts, where their influence on Hindu pillars in the main hall and initial years, many Jain temples were
painting is clearly evident. He was not simplicity of style and symmetry are made adjoining the Buddhist temples
so aspiring to spend time and resources its unique features. following the Buddhist rock-cut style.
ART AND CULTURE 1-311

Initially these temples were mainly carved out of rock faces and the ˜
The foundation of Jaipur, the fabled ‘pink city’, in
use of bricks was almost negligible, however, in later years, the 1727 AD, represents the final phase of the Rajput
Jains started building temple cities on hills based on the concept of architecture. Built by Jai Singh, Jaipur represents a
mountains of immortality. fusion of Eastern and Western ideas of town
planning.
MAIN FEATURES ˜
The city is enclosed by a wall and has bastions and
˜
Compared to the number of Hindu temples in India, Jain temples
towers at regular intervals. The City Palace is at the
are a few and spaced out. Surrounded by embattled walls, the
center of the walled city and is a spectacular
temples are divided into wards, guarded by massive bastions at its
synthesis of Rajput and Mughal architectural styles.
ends, with fortified gateways as the main entrances.
˜
The famous building Hawa Mahal, or A Palace of
˜
These temple cities were not built on a specific plan; instead they
Winds, (1799), which, has a five-storeyed
were the results of sporadic construction.
symmetrical facades composed of 953 small
˜
Natural levels of the hills, on which the ‘city’ was being builts, casements in a huge curve, each with a projecting
accommodated various levels so that as one goes higher the balcony and crowning arch.
architecture, the grandeur increases. ˜
The Jantar Mantar, the largest of five observatories
˜
The only variation in these temples was in the form of frequent built by Jai Singh II in the early 18th century, is
chamukhs or four-faced temples. In these, the image of a another interesting example of Rajput architecture.
Tirthankara faces the four sides, or four Tirthankars are placed
back to back to face the four cardinal points.
˜
Entry into this temple is also from the four doors. The Chamukh EUROPEAN INFLUENCE
temple of Adi Nath (1618 AD) is a characteristic example of the
four-door temple.
ON INDIAN ART
˜
The great Jain temples and sculptured monuments of Karnataka, The European colonists brought with them to India,
Maharashtra and Rajasthan are world-renowned. concepts of their World View and a whole baggage
˜
The most spectacular of all the Jain temples are found at of the history of European architecture—
Ranakpur and Mount Abu in Rajasthan; Deogarh (Lalitpur, Uttar Neo-Classical, Romanesque, Gothic and Renaissance.
Pradesh), Ellora, Badami and Aihole also have some of the The initial structures were utilitarian warehouses and
important specimens of Jain Art. walled trading posts, giving way to fortified towns
along the coastline.
RAJPUT ART THE PORTUGUESE INFLUENCE
The Rajputs were great patrons of art and architecture, the finest
examples being their forts and palaces. The Rajput palaces are They adapted to India the climatically appropriate
complex compositions built as inner citadels surrounded by the city Iberian galleried patio house and the Baroque
and enclosed by a fortified wall as at Chittorgarh and Jaisalmer. churches of Goa. St Cathedral and the Arch of
Some forts, such as those at Bharatpur and Deeg, were protected Conception of Goa were built in the typical
by wide moats. Portuguese-Gothic style.
˜
The St Francis Church at Cochin, built by the
MAIN FEATURES Portuguese in 1510, is believed to be the first church
˜
The oldest surviving palaces date from the mid 15th century and built by the Europeans in India.
are found at Chittor and Gwalior. ˜
The Portuguese also built the Fort of Castella de
˜
The Man Mandir, the largest palace in Gwalior, was built by Raja Aguanda near Mumbai and added fortifications to
Man Singh Tomar (1486-1516). The Man Mandir has two storeys the Bassein fort built by Bahadur Shah, the Sultan
above and two below, ground level overhanging a sandstone cliff. of Gujarat, in 1532 AD. The Bassein fort is famous
This gigantic cliff is punctuated by five massive round towers, for the Matriz (Cathedral of St Joseph), the
crowned by domed Cupolas and linked by delicately carved Corinthian pillared hall and the Porte da Mer (sea
parapets. The whole facade is enriched with brilliant blue tiles. gate).
˜
The palaces of Jaisalmer, Bikaner, Jodhpur, Udaipur and Kota
represent the maturity of the Rajput style. All of these palaces THE DANISH INFLUENCE
were built predominantly in the 17th and early 18th centuries. It is evident in Nagapattnam, which was laid out in
˜
The huge fortified city of Jaisalmer is situated far out in the Thar squares and canals and also in Tranquebar and
desert. The buildings are constructed with the local yellow-brown Serampore. The French gave a distinct urban design to
stone and they have been remarkably preserved owing to their its settlement in Pondicherry by applying the
remote locations. Cartesian grid plans and classical architectural
˜
The city of Bikaner is encircled by 5.63 km long stone wall in rich patterns. The Church of Sacred Heart of Jesus (Eglise
pink sandstone. There are five gates and three sally ports. The de Sacre Coeur de Jesus), the Eglise de Notre Dame
Jodhpur Fort dominates the city, which is surrounded by a huge des Anges and the Eglise de Notre Dame de Lourdes at
wall with 101 bastions, nearly 9.5 km long. Pondicherry have a distinct French influence.
1-312 GSP General Studies (Paper 1)

THE BRITISH INFLUENCE THE INDO-SARACENIC STYLE


It were the British who left a lasting impact on India’s architecture. ˜
The Chepauk Palace in Chennai designed by Paul
They saw themselves as the successors to the Mughals and used Benfield is said to be the first Indo-Saracenic
architecture as a symbol of power. The British followed various building in India.
architectural styles–Gothic, Imperial, Christian, English Renaissance ˜
Other outstanding examples of this style of
and Victorian being the essentials. architecture include the Law Courts, Victoria
˜
The first buildings were factories but later courts, schools, municipal Memorial Hall, Presidency College and Senate
halls and dak bungalows came up later, which were ordinary House of Chennai, Muir College at Allahabad,
structures built by garrison engineers. Napier Museum at Thiruvananthapuram, the Post
˜
The Church of St John at Calcutta was built in 1787 inspired by St Office, Prince of Wales Museum and the Gateway
Stephens Church at Walbrooks. of India in Mumbai, the Maharaja’s Palace at
˜
The Government House in Calcutta was built by Captain Charles Mysore and MS University and Lakshmi Villas
Wyatt modelled on the Kedleston hall of Derbyshire. Palace at Baroda.
˜
The Indian Government Mint in Calcutta is a half-scale replica of
˜
The architecture of New Delhi was the crowning
the Temple of Minerva at Athens and the Pachaiyappa’s Hall in glory of the British Raj. Robert Byron described
Chennai was modelled on the Athenium Temple of Theseus. New Delhi as The Rome of Hindustan.
˜
The Secretariat, University Library, Rajabai Tower, Telegraph
˜
The British built New Delhi as a systematically
Office and the Victoria Terminus all followed the Victorian Gothic planned city after it was made the capital in 1911.
style. ˜
The British Viceroy made Sir Edward Lutyens
˜
In Varanasi, one of the true Gothic monuments is the Queen’s responsible for the overall plan of Delhi. He was
College, built in a perpendicular style by Major Kitoe from 1847 to specifically directed to “harmonise externally with
1852. the traditions of Indian art”. Thus, the Western
architecture with Oriental motif was realised with
˜
In Calcutta, a High Court was constructed following the Gothic style.
chajjas, jalis and chhattris, as stylistic devices in
˜
The Howrah Bridge (1943), with its red brick facade surrounded by the Viceroy’s House (now the Rashtrapati
eight square towers represents a combination of the Oriental and Bhawan).
Roman styles. ˜
Herbert Baker added the imposing buildings of the
˜
Fort William, the stronghold of the British in mid 19th century that South Block and the North Block, which flank the
took 13 years to construct at a cost of more than $3.5 million and the Rashtrapati Bhawan.
Victoria Memorial in Calcutta (1921), designed by Sir William ˜
Another Englishman called Robert Tor Tussell
Emerson, are probably the most imposing of all the British
built the Connaught Place and the Eastern and
structures in India.
Western Courts.
UNESCO’s Cultural World Heritage Sites In India
Name of Cultural Year of Name of Cultural Year of
No. State No. State
World Heritage Site Notification World Heritage Site Notification
1. Agra Fort Uttar Pradesh 1983 15. Humayun’s Tomb, Delhi Delhi 1993
2. Ajanta Caves Maharashtra 1983 16. Khajuraho Group of Monuments Madhya Pradesh 1986
3. Buddhist Monuments at Sanchi Madhya Pradesh 1989 17. Mahabodhi Temple Complex at Bihar 2002
Bodhgaya
4. Champaner-Pavagadh Gujarat 2004 18. Mountain Railways of India Tamil Nadu 1999
Archaeological Park
5. Chhatrapati Shivaji Terminus Maharashtra 2004 19. Qutub Minar and its Monuments, Delhi 1993
(Formerly Victoria Terminus) Delhi
6. Churches and Convents of Goa Goa 1986 20. Rani-Ki-Vav (the Queen’s Stepwell) Gujarat 2014
at Patan, Gujarat
7. Elephanta Caves Maharashtra 1987 21. Red Fort Complex Delhi 2007
8. Ellora Caves Maharashtra 1983 22. Rock Shelters of Bhimbetka Madhya Pradesh 2003
9. Fatehpur Sikri Uttar Pradesh 1986 23. Sun Temple, Konarak Orissha 1984
10. Great Living Chola Temples Tamil Nadu 1987 24. Taj Mahal Uttar Pradesh 1983
11. Group of Monuments at Hampi Karnataka 1986 25. The Jantar Mantar, Jaipur Rajasthan 2010
12. Group of Monuments at Tamil Nadu 1984 26. Archaeological Site of Nalanda Bihar 2016
Mahablipuram Mahavihara (Nalanda University) at
Nalanda
13. Group of Monuments at Karnataka 1987 27. The Architectural Work of Corbusier, Chandigarh 2016
Pattadakal an Outstanding Contribution to the
Modern Movement
14. Hill Forts of Rajashtan Rajasthan (Chittorgarh, Kumbhalgarh, 2013
Ranthambhore, Amber Sub-Cluster,
Jaisalmer, Gagron)
ART AND CULTURE 1-313

Indian Arts in the UNESCO’s Representative List of the Intangible There are many types of traditional Indian
Cultural Heritage of Humanity folk paintings such as Madhubani
1. 2008 : Ramlila, the traditional performance of the Ramayan paintings from Bihar, Patachitra
2. 2008 : Tradition of Vedic chanting paintings from Orissa, Pithora paintings
from Gujarat and Madhya Pradesh, Nirmal
3. 2008 : Kutiyattam, Sanskrit theatre of Kerala
paintings from Andhra Pradesh, Warli
4. 2009 : Ramman, religious festival and ritual theatre of the Garhwal paintings of Maharashtra, Phad paintings
Himalayas, India originating in Rajasthan and so on.
5. 2010 : Mudiyettu, ritual theatre and dance drama of Kerala
What makes these paintings special is that
6. 2010 : Kalbelia, folk songs and dances of Rajasthan each of these types of paintings uniquely
7. 2010 : Chhau dance, a classical Indian (martial arts) reflects the cultural and socio-economic
dance originated in the eastern Indian states. milieu in that particular territory of India,
8. 2012 : Buddhist chanting of Ladakh recitation of sacred Buddhist giving us a clear idea about the life and
texts in the trans-Himalayan Ladakh region, Jammu and work in that particular region. The
Kashmir treatment of each type of painting vastly
9. 2013 : Sankirtana, ritual singing, drumming and dancing of Mainpur differed in the different states of India, so
did the colours and combinations used and
10. 2014 : Traditional brass and copper craft of utensil making among the
even the general approach to the art itself.
Thatheras of Jandiala Guru, Punjab, India
For instance, Warli paintings used clever
stick figures to depict everyday life in rural
Maharashtra. Madhubani paintings, on the
other hand, mostly depicted mythological
figures, the different seasons and major
INDIAN PAINTING events such as marriages and so on. Indian
folk paintings are as vast and diverse as the
Indian culture itself. A detailed study of this
The tradition of painting has been carried on in the Indian subject is very interesting and can take a
sub-continent since the ancient times. Standing as a testimony to researcher a whole lifetime to understand
this fact are the exquisite murals of Ajanta and Ellora, Buddhist and work on. Some of the most important of
the Indian folk paintings and arts will now
palm leaf manuscripts, Mughal and Kangra schools of miniature be dealt with.
Indian paintings etc. Some traditional Indian paintings, like those CAVE PAINTINGS
of Ajanta, Bagh and Sittanavasal, depict a love for nature and its The origin of cave paintings of India date
forces. Even the folk painting of India has become quite popular back to the Prehistoric times. Earliest
amongst art lovers. evidence of paintings in India are rock
paintings in Bhimbetka which are believed
to be made around 5000 BC. Paintings of
PAINTINGS Mirzapur and Panchmarhi are other such
examples.
The roots of the Indian painting can be traced back to the days of the Indus
Valley Civilisation. Paintings on pottery reflected a keen sense of painting The finest examples of these paintings
among the Indus valley people. The paintings of the Ajanta and Ellora caves comprise of the murals of Ajanta, Ellora,
exhibit the creative genius of the artists of that period. Bagh, Sittanavasal etc which reflect an
emphasis on naturalism. Ancient cave
Enduring tough weather conditions, these paintings have surprisingly,
paintings of India serve as a window to our
survived for such a long period of time. A better perspective to study the
ancestors, who used to inhabit these caves.
painting forms of the whole of the nation is to divide it into two heads such as
the Mural paintings and Miniature paintings. AJANTA PAINTINGS
Indian folk paintings are veritably India’s pride. They truly reflect its ancient Most of the paintings seen in the Ajanta
tradition and heritage. Since time immemorial, these paintings that caves date back to the period of the
ornamented the walls, homes and courtyards of the villagers and tribals, were Mahayana sect of Buddhism. The themes of
generally ignored as primitive forms of expression. These paintings lacked most of these paintings revolve around the
refinement and richness, so elitist groups who supposedly associated with life and teachings of the Lord Buddha. This
sophisticated pure art did not bother to give them much exposure. Only in the includes the Jataka stories related to the
start of the 20th century scholars began their research on the subject and various lives and incarnations of Buddha.
begin to realise the true value of Indian folk paintings.
1-314 GSP General Studies (Paper 1)

The Ajanta Caves lie in the gorge of are based on the Pandyan period of Malwa and also the Hindu Rajas of
Waghora river in Aurangabad, the 9th century. Rajasthan were the patrons of this great
Maharashtra. This World Heritage Site These paintings include animals, tradition. In fact, the Mughals were even
is famous for Buddhist monuments and fish, ducks, people collecting lotuses responsible for introducing the unique
paintings. from a pond, two dancing figures etc. Persian flavour in the miniature paintings
Calligraphic lines characterise these Apart from these, one can also find in of India.
paintings, which can be classified into them, inscriptions dating back to the The concept of illustrated palm leaf
portraits, narrative illustrations and 9th and the 10th centuries. The manuscripts came into being during the
ornamental decoration. There are 29 ceiling of the Ardhamandapam is 11th and 12th centuries. Since the
caves in Ajanta. Mural and fresco adorned with murals from the 7th manuscripts were diminutive in size, the
paintings are made on walls of caves. century. paintings done on it had to be much
In these expressions of emotions is smaller. This in itself could have given
done through hand postures, hairstyle MINIATURE PAINTINGS rise to the concept of miniature painting.
is different for each female and even Miniature paintings are special Miniature paintings are also done on
birds and animals show emotions. The works of art because they are wooden tables, ivory panels, paper,
technique employed is fresco painting. delicate handmade paintings, much marble, leather, on walls and even on
Tempra style i.e. use of pigment is used smaller in size than a normal cloth. Some of the special miniature
in these paintings. painting. paintings show the flourishing of the
ELLORA PAINTINGS This is indeed the hallmark of these Mughal period, illustrated manuscripts of
paintings, the intricate and delicate Jains and Buddhists, scenes from the
Ellora caves are nestled amidst the
brushwork, which lends them their Rajput history and also Deccan
Chamadari hills, lying approximately
unique identity, requires much miniatures. Additionally, miniature
18 miles to the North-East of
patience and skill on the part of the paintings also included themes from
Aurangabad city. The paintings can be
artist making them. Indian epics such as Ramayana,
found in five caves. However, all of
Mahabharata, Bhagvata Purana,
them are today preserved only in the It is indeed difficult even to create a
Rasikpriya, Rasamanjari and the ragas of
Kailasa temple. The rock paintings of normal painting or a portrait, so one
Indian classical music.
Ellora were painted in two different can imagine just how much more
series. difficult it would be to create a SCHOOLS OF MINIATURE PAINTING
The first series, which were done when miniature painting. The schools of Indian Miniature Painting
the caves were carved, revolve around FEATURES OF MINIATURE include the Orissa school, the Pala school,
Lord Vishnu and Goddess Lakshmi. the Mughal school, the Jaina school, the
PAINTINGS
The second series, painted centuries Rajasthani school and the Nepali school.
The colours used in miniature
later, illustrate procession of Shaiva
holymen, Apsaras etc.
painting are handmade, acquired PALA SCHOOL
from vegetables, minerals, indigo, The earliest specimens of the Indian
BAGH PAINTINGS conch shells, precious stones, pure Miniature painting are those attached to
The Bagh caves, situated on the banks gold and silver. The most common the Pala school and date back as early as
of the Bagh river, have been excavated theme of Indian miniature painting the 11th century. This school was
on the rock face of a lofty hill. The wall comprises the Ragas i.e. the characterised by its symbolic use of colour
paintings of these caves date back to melodies of Indian classical music. in the paintings, which took its inspiration
the period between the 5th and the 7th Miniature painting is an ancient art from the age-old tantric rituals in vogue
century. These paintings represent the in India and there were many then. Yet other characteristics of the Pala
most exquisite traditions of Indian art schools for the same, including those school included the use of graceful lines,
forms and predate the Ajanta frescoes. of the Rajputs, Deccans and the modelling forms with the delicate
There are nine sandstone Buddhist Mughals. manipulation of variation of pressure, use
caves with beautiful frescoes and of natural colours and so on.
sculptured stonework. EVOLUTION OF MINIATURE
PAINTING IN INDIA JAIN SCHOOL
SITTANAVASAL PAINTINGS The art of Indian Miniature painting The Jain School of Miniature Paintings
Sittanavasal is the site of an ancient started in the Western Himalayas, mainly concentrated on style. This school
Jaina Monastery, located at a distance round about the 17th century. These used strong pure colours, heavy gold
of around 58 km from Trichy. The paintings were very much outlines, stylish figures of ladies,
monastery is known for housing some influenced by the mural paintings diminution of dress to angular segments,
of the most exquisite frescoes in a rock that originated during the later half square-shaped hands and enlarged eyes.
cave. These were initiated by of the 8th century. This art further One can get to see the influence of Jain
Mahendra Verman and Narsimha flourished during the Mughal rule. miniature paintings on Rajasthani and
Verman. Most of these cave paintings Muslim kings of the Deccan and Mughal paintings as well.
ART AND CULTURE 1-315
MUGHAL SCHOOL DURING AKBAR’S REIGN Prominent features of paintings of this
The Mughal School of Miniature Akbar established a different period include too much use of gold,
Paintings mostly depicts amorous department for paintings and silver, and bright colours in paintings;
scenes, Mughal Royal courts and the Karkhanas were developed. He looked reduced liveliness (Artificial and
battlefields, using gold and stone upon paintings as a means of study Unnaturalness due to increasing
colours. European influence) pencil sketching
and amusement, hence he regularly
was also used during this period.
Mughal paintings feature stylised gave awards to the painters.
imagery in rich draped figures with a The participation of Indian painters DURING AURANGZEB’S REIGN
blend of Indian and Persian styles. started through their invitation to Aurangzeb was a puritan and therefore,
˜
The primary difference between the work in Karkhana of Akbar, thus did not encourage art. Painting
Rajput and the Mughal miniature Indian influence started in Mughal declined during his period and lost
paintings lay in the use of colours. The painting. much of its earlier quality. A large
Mughal school used muted colours, number of court painters migrated to
Paintings of this era had features like
giving its paintings a shadow and the provincial courts.
use of foresightening, 3D figures, use
depth, while the Rajasthan school
uses bold primary colours, which of calligraphy in the paintings, most DURING THE PERIOD OF BAHADUR
ended up giving the painting a more common themes are fairs and festivals, SHAH
abstract look. combined efforts of painters began There was a revival of the Mughal
here. Similarly transformation from painting after the neglect shown by
Mughal paintings reflect an exclusive
popular art to court art can be seen Aurangzeb. The style so employed
combination of Indian, Persian and
here. Daswant, Basavan and Kesu showed an improvement in quality.
Islamic styles. As the name suggests,
these paintings evolved as well as were famous painters of this time. After 1712 AD, the Mughal painting
developed during the rule of Mughal DURING JAHANGIR’S REIGN again started deteriorating under the
emperors in India, between the 16th later Mughals. Though retaining the
During Jahangir’s rule Mughal
century and 19th century. outer form, it became lifeless and lost
paintings reached its zenith. Jahangir
The origin of the Mughal school of the inherent quality of the earlier
was a naturalist (Lover of Nature)
painting is considered to be a landmark Mughal art.
hence preferred the pictures of flora
in the history of painting in India. and fauna i.e. birds, animals, trees, FEATURES OF THE MUGHAL
With the establishment of the Mughal flowers etc. PAINTINGS
empire, the Mughal School of He also emphasised on portrait The Mughal style evolved as a result of
Painting originated in the reign of paintings. Painting of this period had a synthesis of the indigenous Indian
Akbar in 1560 AD. decorated margins, and most of the style of painting and the Safavid
Emperor Akbar was keenly interested paintings are miniature paintings. School of Persian painting. The
in the art of painting and architecture. Jahangir was himself an artist and Mughal style is marked by supple
While he was a boy he had taken had his own workshop. When he was a naturalism based on close observation
lessons in drawing. young prince, he established a of nature and on fine and delicate
workshop under supervision of Aka drawing. It is of an high aesthetic
In the beginning of his rule an atelier of
Riza. Famous painter of this period merit. It is primarily aristocratic and
painting was established under the
were Ustad Mansur and Abul Hasan. secular.
supervision of the two Persian masters,
Mir Sayyed Ali and Abdul Samad The portrait of Jahangir illustrated is TUTINAMA
Khan, who were originally employed by a typical example of miniature The manuscripts at the time of Mughal
his father Humayun. executed during the period of were illustrated a lot about the
A large number of Indian artists from Jahangir. This miniature is in the paintings. Some important manuscripts
all over India were recruited to work collection of the National Museum, are given below An illustrated
under the Persian masters. New Delhi. manuscript of the Tutinama in the
It shows Jahangir holding a picture of Cleveland Museum of Art (USA)
The list of Akbar’s court painters appears to be the first work of the
includes a large number of names. the Virgin Mary in his right hand.
The portrait is remarkable for its Mughal school.
Some of the famous painters, other than
superb drawing and fine modelling The style of painting in this manuscript
the two Persian masters already
and realism. shows the Mughal style in its formative
mentioned, are Dasvanth, Miskina,
stage. Shortly after that, between
Nanha, Knha, Basawan. DURING SHAHJAHAN’S REIGN 1564-69 AD, was completed a very
The well-known artists of Shahjahan’s Shanjahan’s rule is marked by ambitious project in the form of the
period are Bichiter, Chaitaraman, Anup outstanding achievements in the field Hamzanama illustrations on cloth,
Chattar, Mohammed Nadir of of architecture though paintings also originally consisting of 1400 leaves in
Samarquand, Inayat and Makr. flourished during his rule. 17 volumes.
1-316 GSP General Studies (Paper 1)

HAMZANAMA and on large pieces of cloth. Different states and continued till the middle of
The Hamzanama illustrations are in a parts of the state stick to their own the 18th century.
private collection in Switzerland. It style and are thus, recognised as GULER
shows Mihrdukht shooting arrows at different schools of paintings. A The last phase of the Basohli style was
the bird on a multi-staged minaret, number of famous schools of Rajasthani closely followed by the Jammu group of
from the upper storey of a pavilion. In painting are in Mewar, Hadoti, paintings mainly consisting of portraits
this miniature, one can observe that Marwar, Kishangarh, Alwar and
of Raja Balwant Singh of Jasrota (a
the architecture is Indo-Persian, the Dhundhar. Rajasthani paintings have
small place near Jammu) by Nainsukh,
tree types are mainly derived from the clear influences of Mughal paintings,
an artist who originally belonged to
Deccani painting and female types are though they are quite distinct in own
Guler but had settled at Jasrota. He
adapted from the earlier Rajasthani ways.
worked both at Jasrota and at Guler.
paintings. Women are wearing four These paintings are in a new naturalistic
cornered pointed skirts and Bani Thani
Bani Thani is an Indian miniature and delicate style marking a change from
transparent muslim veils. Turbans the earlier traditions of the Basohli art.
painting painted by Nihal Chand from
worn by men are small and tight, The colours used are soft and cool. The
the Marwar school of Kishangarh. It
typical of the Akbar period. The style appears to have been inspired by
portrays a woman who is elegant and
Mughal style was further influenced by graceful. The painting’s subject, Bani the naturalistic style of the Mughal
the European paintings, which came in Thani, was a singer and poet in painting of the Muhammad period.
the Mughal court and absorbed some of Kishangarh in the time of king Sawant
the Western techniques like shading KANGRA
Singh (1748-1764). She has been
and perspective. The Guler style was followed by another
compared to the Mona Lisa. Inspired by
style of painting termed as the Kangra
Radha, Bani Thani is characterised by
RAJPUT PAINTINGS idealised distinct features such as style, representing the third phase of
arched eyebrows, lotus-like elongated the Pahari paintings in the last quarter
Rajput paintings originated in the royal
eyes and pointed chin. This painting of the 18th century. The Kangra style
states of Rajasthan, somewhere around
was featured in an Indian stamp issued developed out of the Guler style. It
the late 16th and early 17th century.
in 1973. possessed the main characteristics of
The Mughals ruled almost all the
the Guler style, like the delicacy of
Princely States of Rajasthan at that
drawing and quality of naturalism. The
time and because of this most of the PAHARI PAINTINGS name Kangra style is given to this group
schools of Rajput painting in India Pahari painting is the name given to
reflect strong Mughal influence. In of painting for the very reason that they
the Rajput paintings, made in are identical in style to the portraits of
1916, the famous ceylonese historian Himachal Pradesh and Jammu and
Coomaraswamy did the first Raja Sansar Chand of Kangra. In these
Kashmir. These paintings developed paintings, the faces of women in profile
remarkable scholarly work on the as well as flourished during the period
classification and nomenclature of have the nose almost in line with the
of the 17th to the 19th centuries. forehead, the eyes are long and narrow
Indian paintings. He concluded that Indian Pahari paintings have been
the theme of the Rajput paintings is and the chin is sharp.
done mostly in miniature forms.
related to the Rajputana as well as the
hill state of Punjab (of that time). Thus, CENTRES OF THE PAHARI MYSORE PAINTINGS
SCHOOL OF PAINTING These paintings are a form of classical
he divided the Rajput paintings in two
parts i.e. Rajasthani (concerning to the BASOHLI South Indian painting, which evolved in
Rajputana) and Pahari (concerned to the Mysore city of Karnataka. During
The earliest centre of painting in the
Jammu, Kangra, Garhwal, Basholi and that time, Mysore was under the reign
Pahari region was Basohli, where
Chamba). of the Wodeyars and it was under their
under the patronage of Raja Kripal
patronage that this school of painting
RAJASTHAN SCHOOL Pal, an artist named Devidasa reached its zenith. Centres of the
The very essence of miniature executed miniatures in the form of the Mysore school of paintings exist in
paintings is found in Rajasthan and Rasamanjari illustrations in 1694 AD. Mysore, Bangalore, Narasipura,
can be traced back to its schools of There is one more series of the Tumkur, Shravanabelagola and
Jodhpur, Jaipur, Mughal, Kangra and Rasamanjari miniatures painted in the Nanjangud.
Mewar. The Jodhpur School of same style and almost of the same
Miniature Paintings depict legendary period, but appears to be in a different FEATURES OF MYSORE PAINTINGS
lovers such as Dholu and Maru on hand. The illustrations of the two Mysore paintings are characterised by
camel back. They also depict hunting Rasamanjari series are scattered in a delicate lines, intricate brush strokes,
scenes with elephants and horses. This number of Indian and foreign graceful delineation of figures and the
school of miniature painting mostly museums. The Basohli style of painting discreet use of bright vegetable colours
uses gold and stone colours. These is characterised by vigorous strong and lustrous gold leaf. The paintings are
paintings are miniature paintings of glowing colours. The Basohli style designed to inspire feelings of devotion
the finest quality, made both on paper spread to the various neighbouring and humility in the viewer.
ART AND CULTURE 1-317
of Maharashtra, Phad paintings This seems one of the plausible
TANJORE PAINTINGS originating in Rajasthan and so on.
˜

reasons why their paintings mostly


These are one of the most popular forms revealed the blissful creative side of
of classical South Indian painting. It is What makes these paintings special is
that each of these types of paintings nature, including flora and fauna,
the native art form of Thanjavur (also animals, fishes, birds, the natural
known as Tanjore) city of Tamil Nadu. uniquely reflects the cultural and
cycle of life and death and many other
socio-economic milieu in that
The dense composition, surface richness aspects of creation, even geometrical
particular territory of India, giving us
and vibrant colours of Indian figures. Using bright, vibrant colours,
a clear idea about the life and work in
Thanjavur Paintings distinguish them Madhubani paintings verily pulsate
that particular region.
from the other types of paintings. with life and verve.
The treatment of each type of painting ˜
Though these women carried on with
FEATURES OF TANJORE PAINTINGS vastly differed in the different states of their paintings for centuries, this art
These paintings are notable for their India, so did the colours and was known and acknowledged only as
adornment in the form of semi-precious combinations used and even the late as the 1960s, when the area was
stones, pearls, glass pieces and gold. general approach to the art itself. affected by severe drought.
The rich vibrant colours, dashes of gold, For instance, Warli paintings used ˜
Looking for a different,
semi-precious stones and fine artistic clever stick figures to depict everyday non-agricultural way of earning
work are characteristics of these life in rural Maharashtra. Madhubani money, they began to bring their
paintings. They add beauty and culture paintings, on the other hand, mostly paintings out into the open in a bid to
to a variety of surroundings and decor. depicted mythological figures, the sell them and earn a livelihood.
The paintings so made are mostly of different seasons and major events ˜
Originally depicted on freshly
Gods and Goddesses because this art of such as marriages and so on. Indian plastered mud walls of huts and
painting flourished at a time when fine folk paintings are as vast and diverse displayed during family functions,
looking and striking temples were being as the Indian culture itself. sacred rituals, ceremonies and
constructed by the rulers of several marriages, the same gradually began
dynasties. It uses gesso work in
MADHUBANI PAINTINGS to be done on handmade paper, fabric
traditional paintings which has The Madhubani painting originated and canvas.
embossing material and covered with from the Mithila region in the state of ˜
Incidentally, there is one version that
gold foil. Bihar, India and is thus also known as talks about Madhubani paintings
Mithila Painting. being as ancient as the
The term madhu literally means Mahabharata itself. It relates how
FOLK PAINTINGS honey and bani refers to forest. So, king Janaka hired artists to paint on
those living in the hilly wooded the walls of the palace at the time of
Indian folk paintings are veritably
terrains of Bihar gave birth to this his daughter, Sita’s marriage to Lord
India’s pride. They truly reflect its
wonderful art. Rama.
ancient tradition and heritage. Since ˜
Madhubani paintings use
time immemorial, these paintings that FEATURES OF MADHUBANI two-dimensional imagery and uses
ornamented the walls, homes and
PAINTINGS natural colours derived from plants.
courtyards of the villagers and tribals, Ochre and lampblack are also used
were generally ignored as primitive These paintings were originally made
by the women of the village, on the to get reddish-brown and black hues
forms of expression. respectively. Because the art has been
walls of their homes. Influenced by the
These paintings lacked refinement tradition and culture of those times, confined to a single geographical
and richness, so elitist groups who they painted popular mythological region and has been passed down
supposedly associated with figures such as Ram, Sita, Krishna, through generations of families, the
sophisticated pure art did not bother to Radha, Durga, Shiva, Lakshmi and style and content of the painting has
give them much exposure. Only in the Saraswati; pictures of nature such as mostly remained the same.
start of the 20th century scholars began the sun, moon, starry nights and CONTRIBUTORS TO THE EVOLUTION
their research on the subject and begin religious plants such as the Tulsi OF
to realise the true value of Indian folk (sage); seasonal festivals and so on. MADHUBANI PAINTINGS
paintings. ˜
The techniques used in Madhubani Sita Devi of Jitwarpur village, Ganga
There are many types of traditional painting were guarded by the women Devi and Baua Devi were the ones
Indian folk paintings such as in the family and were passed on who were responsible to transform the
Madhubani paintings from Bihar, from generation to generation and Mithila paintings from village walls
Patachitra paintings from Orissa, from mothers to their daughters. onto canvas and paper. The todays’
Pithora paintings from Gujarat and ˜
The artisans who created artists such as Pushpa Kumari, Karpuri
Madhya Pradesh, Nirmal paintings Madhubani paintings would invoke Devi, Jamuna Devi, Lalita Devi, Sarita
from Andhra Pradesh, Warli paintings holy spirits, divine blessings and Devi, Godawari Dutta and
mother nature herself, before Mahasundari Devi are among the
proceeding to create a work of art. foremost contributors of the art.
1-318 GSP General Studies (Paper 1)

FEATURES OF KALAMKARI ART bright shades of purple, pink and


KALAMKARI ART brown with a touch of gold and silver
Kalamkari or Qalamkari art, literally The fabric used in Kalamkari acquires a
certain gloss due to being treated by colours. This colour scheme is what
means ‘pen-art’. The Persian term makes the Patachitra stand out from
various natural mixtures and dyes. It is
kalam means pen, and kari means the rest of the folk art.
first immersed in a mixture of resin
craftsmanship.
(myrabalam) and cow’s milk for about an
This is a form of hand-painted or hour. How the Pata Paintings
block-printed painting on cotton fabric ˜
Then contours are marked on it with a came into Existence
and is done in many different parts of bamboo soaked in a mixture of Every year, the painted wooden
India. Kalamkari work developed and fermented jaggery and water. Then images of Jagannath, Balabhadra and
evolved in Machilipatnam, Andhra vegetable dyes are used on the same. Subhadra, the deities of the Puri
Pradesh, largely due to the patronage The fabric is washed after each colour Jagannath temple, are offered the
of the Golconda sultans during the treatment, so each fabric undergoes traditional Abhisheka (holy bath
Mughal era. It had many business ties about 15-20 washings. Tints and shades ritual). This invariably leads to the
with Persia, due to which the are obtained with the help of plants, discoloration of the images, due to
Kalamkari industry came up in a huge seeds, crushed flowers and cow dung. which they have to be removed from
way in India. the Garbhagriha (sanctum
˜
Even today, Kalamkari craft is practiced sanctorum) and taken for repainting.
It is said that in the olden days, there in many families of Andhra Pradesh and It is at this time that the temple
used to be large groups of painters, forms a major part of their livelihood. substitutes three paintings for the
singers and musicians (popularly ˜
Though Kalamkari art witnessed a images. In Sanskrit, the word pata
known as chitrakathi), who moved slight, temporary decline in between, it means cloth and chitra means
around each village, telling people was revived in India and abroad and has painting.
great stories from Hindu mythology. been given tremendous worldwide
With time, they started illustrating exposure since the 18th century, FEATURES OF THE PATACHITRA
these stories with impromptu paintings especially by the English during the PAINTINGS
made on canvas sheets. This is how British Raj in India. ˜
Pata paintings are very popular
Kalamkari was born. Then it went on with tourists, especially foreign
to evolve steadily through the PATACHITRA PAINTINGS visitors, who take them back as a
centuries. There are predominantly two Pata or Patachitra is a form of painting souvenir.
types of Kalamkari art in India. The that is very distinct and is set apart from ˜
Patachitras depicting the utsava
first is the Srikalahasti style and the the rest. Originated in Orissa, the themes image of Puri Jagannath are
other, the Machilipatnam style. of these paintings mainly revolve around considered to be extremely special.
The Srikalahasti style of this art Lord Jagannath, the prime deity at the These can also be found at several
involves using the pen or kalam for Jagannath temple of Puri, a beach town in ethnic stores and handicraft expos
freehand drawing and then filling the the East Indian state of Orissa. across the country.
colours inside. The piece here is Patachitras are painted by artists of ˜
Patachitra art is traditionally
developed entirely by hand. This style Orissa, called Chitrakars. They use bold, practised by a certain family of
of Kalamkari is woven around temples vibrant and very Indian colours to depict artists living around the
and deities and depicts all religious their images. Jagannath temple area at Puri.
objects, such as temple hangings, It originated in the 8th century and is Starting off as a ritual, Patachitra
scrolls, chariot banners and so on and supposed to have been the earliest form of art is now considered to be one of
mythological scenes based on great indigenous art. It thrived on the the most cherished collectors
epics like Ramayana, Mahabharata, patronage of the royalty and their kins. items.
Puranas and so on.
When the popular Bhakti Movement ˜
Along with these cloth paintings,
Kalamkari paintings incorporate only evolved in the 16th century, the paintings the Chitrakars also often etched
natural dyes taken from plants and of Radha and Krishna were included in a images of the deities on dried palm
require the artisan to undertake 17 riot of colours and shades of orange, leaves. This art is known as
complex procedures to complete one yellow and red. Talapatachitra. Rectangular
work of art. Smt Kamaladevi pieces of palm leaves are dried and
Important instances from Krishna’s life
Chattopadhyay is mainly responsible to stitched together by means of thin
and Krishna Leela, especially, Raas Leela,
maintain and develop the art of black thread. The desired designs
were depicted through the medium of
Kalamkari, as the First Chairperson are then etched on the leaves with
these paintings.
of the All India Handicrafts Board. a needle.
At present, the JJ School of Art in Additionally, aspects of nature, such as
birds and animals, flora and fauna, were
˜
The art of Patachitra thrives even
Mumbai is experimenting with the
also included in the paintings. Krishna today and has attained
potential of Kalamkari art on Silk Ikat
(a form of tie-and-dye textile popular in would invariably be depicted in blue, international status as one of the
Pochampally, Andhra Pradesh. while the gopis would be shown attired in finest art forms.
ART AND CULTURE 1-319

Kalighat paintings got hugely popular in the 20th century, when


KALIGHAT PAINTINGS Parsi based families lived with each other to acquire them. The main
Kalighat paintings first came into being during the aim of Kalighat paintings was to create a dynamic picture that
19th century at Kolkata, Bengal, in the market area would intuitively apprehend the subject rather than simply
close to the Kalighat temple there. It first appeared represent it. The general treatment of figures and planes forged the
as a souvenir associated with the Kali temple at two dimensional quality of the pictorial space. The broad planes, the
Kalighat. In the beginning, Kalighat temples only bold lines, the linear tensions, the vibrant colours used and the
treated Hindu mythological themes, but gradually symmetrical curves smoothly blended in together to create a visual
initiated including everyday themes, secular, social poetry of sorts. Sadly, since the painters left their portraits
and the current political themes as well. unsigned, vital information about them is not available.
So, the art that had till then been within the
boundary of religion alone, stepped outside it and BATIK PAINTINGS
commented on contemporary issues affecting the Batik is a wax-resistive dyeing technique used on fabric. Batik is an
social, cultural and societal milieu. important aspect of Indonesia and is actually considered as a
These paintings started reflecting the new values and national art there. Interestingly, similar patterns of the Batik art is
new ethics of a modern society. Some artists even also found in several countries such as Ghana, Cameroon, Nigeria,
painted Imam Hussain’s horse, Duldul and certain Mali, the Philippines, Thailand, Malaysia, Sri Lanka, Iran,
other aspects of Islam, in order to please their Bangladesh and India. The commonest Batik fabrics used to
Muslim clients. With time, Hindu artisans from excellent Batik prints are poplin, cambric, voiles and pure silk.
several parts of Bengal migrated into Kalighat and Natural colours derived from barks of trees, leaves, flowers and
established their profession there. That entire area minerals were used for Batik printing. Batik art in India is actively
around the temple, soon came to be known as propagated and promoted mainly by the University of Shantiniketan
Pata-Para or the artists’ locality. The artists who in Calcutta and Chola Mandalam in Chennai.
created Kalighat paintings came from several areas
of rural Bengal. LEPAKSHI PAINTINGS
Their paintings, hence, were an impression of their Another type of Indian painting is the Lepakshi painting; a wall
own perception of urban life and society in their painting made on the walls of the temple of Lepakshi, a small village
times. They would, for instance, show their derision in Anantpur, district of Andhra Pradesh. The theme behind there
towards the Babu culture which raised its hood then, paintings was secular despite being depicted on temple walls.
the rise of feminism and liberal attitudes, religious
hypocrisy, social debauchery and so on. The paintings
did not stop with being mute works of art. They
actually portrayed a dynamic society, evolving each
and every day.
FEATURES OF THE KALIGHAT PAINTINGS
MUSIC, DANCE AND
The Kalighat paintings actually reflected the
painters’ own orthodoxy and fear of the rapid social
DRAMA
changes taking place at the time.
India, the land of a rich culture and heritage, since its
˜
Popular themes portrayed then would be of the Rani
of Jhansi on horseback, babus wooing their inception, has music, dance and drama as an integral aspect
concubines, cats bearing Hindu holy marks on the of its culture. Earlier used as a medium of propagating
forehead as allusion to debauchery,
good-for-nothing, dandies and so on.
religion and social reforms, these performing arts have
˜
There was a general wave of resentment against the immensely contributed in educating the masses as well.
British Raj, so they would try to portray the victory These arts now also serve as a means of entertainment.
of good over evil, symbolising the ultimate victory of
India over the British rule.
˜
The most important and attractive aspect of the DEVELOPMENT OF MUSIC IN INDIA
Kalighat paintings was that they were simple The melodies of Samaveda can be considered to be the earliest
drawings and paintings, which could easily be examples of Indian music. The science of music called
reproduced by lithography. Such prints were Gandharvaveda, is an Upaveda of Samaveda. Bharata’s
hand-coloured, giving the entire picture a beautiful, Natyashastra is the earliest known treatise on dance, drama and
raw look. This trend continued up to the early part music. Even earlier, the statues found from the Indus Valley
of the 20th century and these paintings ended up in Civilisation period depict elements of music. Some of the statues
museums and private collections. show a performer in a dance form, while some others show a type of
˜
Kalighat paintings varily captured the essence of musical instrument, which looks like a flute. Even the Rigveda
daily life and they influenced modern artists like the contains description of musical instruments like veena, mridangam
late Jamini Roy. and bansuri.
1-320 GSP General Studies (Paper 1)

Some date the advent of the system of ˜


Sa, Re, Ga, Ma, Pa, Dha, Ni, Sa (in the The most popular ragas are Bahar
classical Indian music to Amir ascending order). Bhairavi, Sindhu Bhairavi, Bhim
Khusrow. Muslim rulers and noble ˜
Sa, Ni, Dha, Pa, Ma, Ga, Re, Sa (in the Palasi, Darbart, Desh, Hamsadhwani,
men freely extended their patronage to descending order). Jai Jayanti, Megha Malhar, Todi,
music. In the courts of the Mughal Musicians can elaborate ragas on the Yaman, Pilu, Shyam Kalyan and
emperors, music is said to have basis of this plan and create more tonal Khambaj.
flourished and Tansen was one of the patterns pleasing to the ears of the Dhrupad This style of singing is
Navratnas (nine jewels) of Akbar’s listeners. Ragas in the South mostly traditionally performed by men with
court. For the first time in 13th have Sanskrit names, unlike in the a Tanpura and Pakhawaj. The lyrics
century, the division between North North, where their names often come sung in Dhrupad are in a medieval
Indian (Hindustani) and South Indian from specific regions or from the form of Hindi and typically heroic in
(Carnatic) music was also being more dialects of regions. theme or in praise of a particular
sharply delineated by Haripaladeva in
deity. Dhrupads are sung in four
his text sangeet sudakara. Classical SOME FAMOUS RAGAS styles called Bans Gaurhar, Dagur,
music, both Hindustani and Carnatic, There are six principal ragas, which
Khandhar and Nauhar, initially
may be either instrumental or vocal. have their off shoots known as
named after the language or dialect
Ragapatnis and Ragaputras; which
INDIAN CLASSICAL MUSIC in which the verse was written.
vary from 84 to 108. Some of the famous
Pandit Uday Bhawalkar, Pandit
Indian Classical Music originated from ragas are as follow
Ritwik Sanyal and the Umakant
Vedic chants or Sama music. This ˜
Raga Bhairavi It is a morning raga.
Gundecha and Ramakant Gundecha
music chiefly consisted of chanting of It is also considered as the Lord of
(Gundecha Brothers) are some
hymns in praise of the Vedic Gods. The Ragas. It is thought to have originated
famous Dhrupad vocalists.
musical structure of the chants was from the throat of Shiva.
characterised by descending order of ˜
Raga Megha It is a melody of clouds Khayal It consists of about 4-8 lines of
nodes, initially two to five, which later and rains. lyrics set to a tune. The performer
was increased to seven nodes. The uses these few lines as the base for
˜
Raga Durga It is a very popular late
Raga (structure of melody) and Tal improvisation. Generally the theme
evening raga.
(structure of rhythm) are the two major of Khayal is romantic in nature. They
˜
Raga Bilaval It is one of the ragas
characteristics of Indian classical sing about love, even if they are
that appears in the Sikh tradition and
music. The melody deals with the rise related to the divine creatures. One of
is part of the Sikh holy scripture Sri
and fall of sounds and the latter deals the most important/unique features
Guru Granth Sahib. It is a morning
with the pattern of time beats of Ragas. of Khayal is the use of taan in the
raga and is sung with a feeling of deep
Tala is the pulse of Indian music. The devotion and repose and is performed compositions. The Khayal form of
term tal is derived from the Sanskrit during the hot months. Hindustani classical music is
word, tala which means ‘to strike with ascribed to Hussain Shah Sharqui,
palms’. Early musicians may have HINDUSTANI MUSIC the 15th century ruler of the Sharqui
employed claps or palm strokes to dynasty. It was made popular by the
The origin of Hindustani classical music
mark time in dance and music which 18th century Mughal ruler
may be traced back to the period of the
later developed into a complicated Mohammed Shah. Some of the
Delhi Sultanate and to Amir Khusrow
system of 108 tals of classical music. It (1253-1325 AD), who encouraged the modern day vocalists are late
is a time cycle that remains fixed practice of musical performance with Bhimsen Joshi, Nagraj Havaldar,
throughout a particular rendering tal, particular instruments. He is believed Kishori Amonkar, Ulhas and
binds music together and offers a to have invented the sitar and the Kashalkar, Prabhakar Karekar,
regularity that calms the mind. The tabla and is said to have introduced Pandit Jasraj etc.
tala and raga varies in each new ragas. Most of the Hindustani Hori It is a genre of semi-classical
composition. Sometimes, tala is more musicians trace their descent to singing, which is popular in Uttar
active and controls the other or Tansen. Pradesh and Bihar. It comes under
vice-versa. This depends on the person It is said that Tansen’s music had a the category of seasonal songs.
performing the music. magical effect. He could stop the rising Dhamar These compositions are
Raga is the basic scale or note pattern waves of the Yamuna and by the force similar to Dhrupad, but are chiefly
of a melody formed by selecting notes of his Megha Raga, he could cause the associated with the festival of Holi.
from the thirteen tonal intervals rain to fall. There are 10 main styles of Here, the compositions are
conventionally established in the singing and composition in Hindustani specifically in praise of Lord Krishna.
octave space. The nodes are selected music. Dhrupad, Dhamar, Hori, Khayal, This music, sung in the Dhamar Tal
from ascending as well as descending Tappa, Chaturang, Rayasagar, Tarana,
is chiefly used in festivals like
progressions. Sargam and Thumri.
Janmashtami, Ramnavami and Holi.
They are as follows
ART AND CULTURE 1-321

Chaturang It denotes four colours or a Exponents Bal Krishna Ealchal Exponents Pandit Jasraj, Moti Ram,
composition of a song in four parts; Karanjikar, Vishnu Digambar Mani Ram, Sanjeev Abhyankar and
Fast Khayal, Tarana, Sargam and a Paluskar, Pandit Omkarnath Thakur, others.
Paran of Tabla or Pakhawaj. Veena Sahasrabuddhe and Malini
Tarana Another vocal form of the Rajurkar. CARNATIC MUSIC
Hindustani music is Tarana. Tarana AGRA GHARANA Carnatic music is a system of music
are songs that are used to convey a commonly associated with the Southern
The Agra Gharana places great
feel of joy and are usually performed part of the Indian sub-continent, with
importance on developing forcefulness
towards the end of a concert. its area roughly confined to four
and depth in the voice, so that the nodes
Thumri It is an informal vocal form of states–Andhra Pradesh, Karnataka,
are powerful and resonant.
Hindustani classical music and is Kerala and Tamil Nadu.
Founders Haji Sujan Khan, Ustad
said to have begun with the court of In contrast to Hindustani music, the
Ghagghe Khuda Baksh.
Nawab Wajid Ali Shah, the Nawab of main emphasis in Carnatic music is on
Oudh. The compositions are either Exponents The important singers of vocal music; most compositions are
this gharana are Faiyaz Khan, Latafat written to be sung and even when
devotional or romantic in nature. It
Hussain Khan and Dinkar Kaikini. played on instruments, they are meant
was inspired by Bhakti movement. It
used mainly Hindi dialects like KIRANA GHARANA to be performed in gayaki (singing)
Awadhi or Braj Bhasha. It derives its name from the birth place style. Also the Carnatic music is
Raga Sagara It consists of different of Abdul Kharim Khan of Kirana, near precomposed with the scope for
parts of musical passages in different Kurukshetra. In the Kirana style of improvisation being very little
ragas as one song composition. These singing, the swara is used to create an compared to Hindustani music.
compositions have 8 to 12 different emotional mood by means of elongation Purandardas is considered the
ragas and the lyrics indicate the and use of Kanas. Father of Carnatic Music. He is
change of the ragas. The peculiarity Founders Abdul Karim Khan and credited for the codification of Carnatic
of this style depends on how smoothly Abdul Wahid Khan. music. The system for classifying
the musical passages change South Indian ragas, Melankara was
Exponents Hirabhai Barodekar,
alongwith the change of ragas. developed by Venkat Mukhi Swami.
Begum Akhtar, Bhimsen Joshi,
Sargam It is a type of composition that The present form of Carnatic music
Gangubai Hangal and Prabha Atre.
has swaras in their lyrics as well. was acquired in 18th century, when
Sargams are composed in different
JAIPUR GHARANA the trio of Thyagaraja, Shamashastri
ragas and tals. These are also The most distinctive feature of the and Muthuswami Dikshitar made
prominent in initial learning stage of Jaipur gharana can be best described their compositions.
Hindustani classical music. through its complex and melodic form, Saint Tyagaraj was one of the greatest
which arises out of the involutedly and composers of Carnatic music, a form of
Ghazal It is a poetic form consisting of undulating phrases that comprise the
rhyming couplets and a refrain, with Indian classical music. Tyagaraj was
piece. an ardent devotee of Lord Rama. In his
each line sharing the same metre. It
is an ancient form originating in 6th Founder Ustad Alladiya Khan. praise and honour he wrote numerous
century Arabic verse. It spread into Exponents Alladiya Khan, kritis. Tyagaraj composed many new
South Asia during the 12th century, Mallikarjun Mansur, Kesarbhai janya ragas as well as melakarta raga.
due to the influence of Sufi mystics. Kerkar, Kishori Amonkar, Shruti He is said to have invented 66 new
Sadolikar, Padma Talwalkar and ragas in his time.
Originally, Ghazal was part of Dari and
Ashwini Bhide Deshpande.
Urdu, but gradually it has been COMPOSITION OF
written in many languages of the RAMPUR SAHASWAN GHARANA CARNATIC MUSIC
Indian sub-continent. Some of the In the Rampur Sahaswan Gharana, Most compositions in Carnatic music
important poets of Urdu Ghazal are there is a stress on the clarity of swara have three parts to their body. The
Faiz Ahmed Faiz, Mirza Ghalib and and the development and elaboration of first two lines of the song (sometimes
Firaq Gorakhpuri. the raga is done through a stepwise just one) are called Pallavi. They occur
progression. over and over, especially after each
SOME FAMOUS GHARANAS stanza. Usually, the Pallavi is followed
GWALIOR GHARANA MEWATI GHARANA by two more lines or sometimes just
This is the oldest among all the Khayal The Mewati Gharana gives importance one more. This portion is called Anu
Gayaki (vocal) styles. The distinctive to developing the mood of the raga Pallavi. This is sung at the beginning
feature of this style of singing has been through the notes forming it and its for sure, but sometimes even during
noted for its lucidity and simplicity. style is Bhava Pradhan. It also gives the end of the song, but not necessarily
equal importance to the meaning of after each stanza. The stanzas of a
Founders Ustad Hassu Khan, Ustad
the text. song are called Charanam.
Haddu Khan, Ustad Nathu Khan.
Founder Ghagge Nazir Khan.
1-322 GSP General Studies (Paper 1)

Varnam It is a composition usually sung or played at the There is no formal apprenticeship, where the student
beginning of a recital and reveals the general form of the raga. learns the music. It is learnt by absorbing through
The two halves are almost equal in length. regular contact.
The Varnam is made up of two parts as follow Folk music is an essential component of weddings,
1. The Purvanga or first-half. 2. The Uttaranga or second-half. engagements and births. Musical instruments used are
Kriti It is a highly evolved musical song set to a certain raga and also different from classical music.
fixed tala or rhythmic cycle. They are not as refined and are often in a crude form.
Ragam It is a melodic improvisation in free rhythm played Folk instruments are usually crafted by the musicians
without mridangam accompaniment. themselves and from commonly available materials, such
Tanam It is another style of melodic improvisation in free as bamboo, coconut shells, pots etc.
rhythm. Bhavageet It literally means emotional poetry and is a
form of expressionist poetry and light music.Most of the
Pallavi This is a short pre-composed melodic theme with words
poetry sung in this genre pertain to subjects like love,
and set to one cycle of tal. Here, the soloist improvises new
nature, philosophy etc and the genre itself is not much
melodies built around the word Pallavi.
different from ghazals.
Trikalam It is the section where the Pallavi is played in three
This genre is quite popular in many parts of India,
tempos keeping the tal constant.
notably in Karnataka.
Swara-Kalpana It is the improvised section performed with
Bihugeet It is a traditional folk music of Assam
the drummer in medium and fast speeds.
performed through Bihu dance in the festival of Bihu.
Ragamalika This is the final part of the Pallavi, where the The songs have themes of romance, love, nature and
soloist improvises freely and comes back to the original theme at incidents.
the end.
Lavani It is a popular folk form of Maharashtra
Difference between Hindustani and Carnatic music styles traditionally, the songs are sung by female artists, but
male artists may occasionally sing Lavanis. The dance
No. Hindustani Music Carnatic Music
format associated with Lavani is known as Tamasha.
1. Mainly practised in North India. In South India
2. It is influenced by Arabic and Quite indigenous in nature
Dandiya It is a dance oriented folk music from Gujarat
Persian music styles. that has also been adapted for pop music worldwide,
3. Artists are provided with the scope No such freedom is enjoyed by popular in Western India, especially during Navratri.
of improvisation. these artists The present musical style is derived from the traditional
4. It has strict adherence to time and No strict time adherence and musical accompaniment to the folk dance of Dandiya
performed on 6 major ragas performed on 72 major ragas. called by the same name.
5. Various sub-styles emerged due to No sub-styles are there.
Pandavani It is a folk singing style of musical narration
the practise of improvisation,
which are named as gharana. of tales from ancient epic Mahabharata with musical
6. Instruments and vocals are given Vocals are given more importance accompaniment and Bhima as hero. This form of folk
equal importance. are used. theatre is popular in the State of Chhattisgarh and in
7. Tabla, Sarangi, Sitar etc are major Veena, Violen and Mrindangum are the neighbouring tribal areas of Odisha and Andhra
instruments used. used. Pradesh.
Naatupura Patu It is Tamil folk music. It consists of
Modern Indian Music Gramathisai (village folk music) and Gana (city folk
With the British rule came Western music. Indians adopted some of music).
their instruments such as violin and clarinet to suit the demands of Garba It is sung in honour of Hindu Gods and
Indian music. Orchestration of music on stage is a new development. Goddesses during Navratri. They are sung in the honour
Use of cassettes replaced oral transmission of tunes and ragas. of Lord Krishna, Hanuman, Rama etc.
Performances, which were earlier limited to a privileged few, have now
been thrown open to the public and can be viewed by thousands of Tappa The Tappa is said to have developed in the late
music lovers throughout the country. Music education no longer 18th century AD in Rajasthan from the folk songs of
depends on the master-disciple system but can be imparted through camel riders. The credit for its development goes to
institutions teaching music. Shorey Mian or Ghulam Nabi of Multan. Tappa
literally means ‘jump’ in Persian. They are essentially
folk– lores of love and passion and are written in Punjabi.
FOLK MUSIC Its beauty lies in the quick and intricate display of
India has a rich legacy of folk or popular music. This music is a various permutations and combinations of nodes.
rustic reflection of the larger Indian society. Due to the extreme The compositions are very short and are based on
cultural diversity across India, there are a variety of folk styles. Shringara Rasa. It is rather strange that even though
Folk music is not taught in the same way as classical music. the Tappa lyrics are in Punjabi, Tappa is not sung in
Punjab.
ART AND CULTURE 1-323

FAMOUS CLASSICAL Famous Personalities of Instrumental Music

SINGERS OF INDIA Sarod Veena


Ustad Alauddin Khan Dr Balamuralikrishna
USTAD BADE GHULAM ALI KHAN
Ali Akbar Khan Shehnai
One name that truly epitomises the 20th
Ahmad Ali Khan Bismillah Khan
century Hindustani classical music tradition
Amjad Ali Khan Ali Bor ‘Vilayati’
is that of Ustad Bade Ghulam Ali Khan.
Tabla Flute
DR BALAMURALIKRISHNA Ustad Allah Rakha Hariprasad Chaurasia
One of the most famous names associated Lal Ahmad
with classical Carnatic music is that of
Mian Kader Baksh Pandit Bholanath
Dr Balamuralikrishna. This legendary
Ashiq Ali Khan Pannalal Ghosh
musician and Padma Bhushan Award is an
Ustad Zakir Hussain Sarangi
able composer, singer, poet and
instrumentalist, who can sing perfectly in Sitar Bundu Khan
three octaves. Pandit Ravi Shankar Pandit Ram Narayan
Sital Nawaz Abdul Halim Jaffar Khan Ustad Mehboob Khan
BHIMSEN JOSHI
Akshay Kumar Ghosh Pandit Udayalal
One of the most prominent and legendary
Ustad Vilayat Khan Pandit Madhav Prasad
Hindustani vocalist was Pandit Bhimsen
Mandolin Ustad Abdul Wahid Khan
Joshi. He can very easily be called as
someone, who has not just earned true fans, Upalappu Srinivas Sabri Khan
but has also won over critics. Subbaraju Mridangam
Violin TV Gopala Krishna
PANDIT KUMAR GANDHARVA Dr Balamuralikrishna Dr Balamuralikrishna
One of the most famous classical musicians in
VG Jog Khanjira
India was Pandit Kumar Gandharva. His real
Lal Gudie Jay Araman Dr Balamuralikrishna
name was Shivaputra Siddaramaiah
Komkali. Pandit Kumar Gandharva was born Ariyakkudi Sri Ramanya lyengar Santoor
on 8th April, 1924 in a place called Dharwad Semmangudi Sri Srinivasa lyer Shiv Kumar Sharma
located in Karnataka. Sri GN Balasubramaniam Drum
Karaikudi Sambasiva lyer TH Vinayakaram
Yehudi Menuhin
Pandit Shivkumar Sharma
Dr L Subramaniam
He is a very famous classical musician, who has
acquired international fame by playing the
classical instrument, Santoor. The Santoor is a
trapezoid shaped hammered dulcimer often EMERGENCE OF DANCE IN INDIA
made of walnut wood and has 72 strings, which In India, dance is considered to be divine in origin. The Gods and Goddesses
are struck with two delicate caved wooden not only take great delight in dance, drama and mime, but many are great
mallets.
dancers themselves. Shiva’s cosmic dance, Tandava is believed to encompass
Ustad Zakir Hussain creation, preservation and destruction and this idea has been embedded in
Popularly known as Tabla Maestro, Ustad Zakir Hindu thought and ritual, since the dawn of civilisation. The dance of Kali,
Hussain is the son of the renowned tabla player the dark and fierce Goddess of destruction are very significant. Krishna is
Ustad Allah Rakha. He was born on 9th March, one of the most popular dancing divinities of the Hindus.
1951 and showed much interest in tabla since a According to the Natyashastra, Brahma the creator had created drama. He
very young age.
then took pathya (words) from the Rigveda, Abhinaya (gesture) from the
Ustad Amjad Ali Khan Yajurveda, geet (music and chant) from Samaveda and rasa (sentiment and
A distinguished maestro in the field of playing emotional element) from Atharvaveda to form the fifth Veda, Natyaveda.
Sarod, Ustad Amjad Ali Khan is popularly known The first and the oldest of evidences to date are the discovery of the bronze
as the Sarod Samrat. He is the 6th generation figurine of a dancer from the Indus Valley Civilisation excavations at
Sarod player in his renowned family. Mohenjo-daro and Harappa.Dancing figures are also commonly seen in
many primitive cave paintings and sculptures at temples and stupas.
1-324 GSP General Studies (Paper 1)

BASIC ASPECTS OF DANCE BHAVA


Bhavas are the imitations of emotions that the actors perform. The
RASA AND BHAVA
artists experience an emotion and is so overwhelmed by it that he
According to Bharata’s Natyashastra, there are
seeks a medium with it to express those feelings.
eight fundamental feelings or mental states referred
to as Sthayi Bhavas which can be experienced by BHAVA OF DANCES
human beings. Corresponding to these mental There are two fundamental bhava of classical dances –Tandava and
states, there are eight Rasas also. Lasya. It is considered that these bhava had originated from the
Sthayi Bhavas Rasas
dance of Lord Shiva and Goddess Parvati.
Delight (Rati) The Erotic (Sringara) Tandava is the exotic dance of Lord Shiva and shows the construction,
Laughter (Hasya) The Comic (Hasya) destruction and sensation of life.
Sorrow (Soka) The Pathetic (Karuna) Lasya Every dance form in India finds its root in Lasya. Goddess
Anger (Krodha) The Furious (Rudra) Parvati is creator of Lasya. The steps of the dance (Mudra) in Lasya is
Heroism (Utsaha) The Heroic (Vira)
very gentle, natural and affectionate. ‘Taal’ word is derived from two
words ‘Ta’ and ‘La’. ‘Ta’ stands for Tandava dance and ‘La’ for Lasya.
Fear (Bhaya) The Terrible (Bhayanaka)
Tandava is furious (Raudra) dance of Lord Shiva, whereas Lasya
Disgust (Jugupsa) The Odious (Bhibhatsa)
represent love and affection. All the classical dances of India is based
Wonder (Vismaya) The Marvellous (Adbhuta) upon these two bhavas. Bharatnatyam, Manipuri, Kuchipudi, Odissi,
and Kathak are related to Lasya bhava and Kathakali is with
MARGI AND DESI Tandava.
The Natyashastra divides dance into various
categories. On one hand, we have the classical dance Tandava Dance
forms such as Bharatnatyam, Kathakali, Kathak
There are two dance forms of Lord Shiva — Lasya (soft and affectionate)
and so on and on the other hand, we have dances and Tandava (anger or furious). The demon beneath the feet of Lord
like Garba, Bhangra, Chhau and so on. Although, Nataraja signifies the darkness of knowledge. Tandava dance signifies the
these are all dances, they fall into two distinct creation, maintenance and destruction of the world. Lord Shiva is therefore
categories i.e. Margi (classical) and Desi (folk or called as Nataraja means king of dancers.
regional). Tandu is an agent of Lord Shiva and the name of Tandava signifies him.
Tandava dance is source of eternal energy and Rupak (dramatic
composition). There are five dramatic compositions––Shrishti, Sthithi,
CLASSICAL DANCES Samhara, Tirobhava, Anugraha. Hence, Tandava reflects the universal
The literal meaning of shastra is– holy books pathway of life and death.
incorporated to principles, laws and regulations. In
this dance form there is a certain order and balance NRITYA MUDRA (POSTURES OF DANCES)
between body and music. The essential theme of
Abhinaya Darpana, the book written by Nandikeshwara describes
classical dances is devotion to the God. The impulses
about the postures of dance. According to him there may be 28
of soul are reflected through the dances. ‘Sangita
postures from uncombined hand (Asamyuta Hasta) and 24 postures
Ratnakara’ written by Sharangdeva divided
from combined hand (Sanmyukta Hasta). Physical expressions may
classical dances into three forms
be of different types e.g. Tribhanga Mudra, in which neck, waist and
(i) Nritta It corresponds to pure dance steps knee are so bended, the body shape seems to be ‘S’ letter of alphabet.
performed rhythmically. Here the movements
of the body do not convey any mood and its CLASSICAL DANCES OF INDIA
purpose is just creating beauty by making
According to tradition, Indian classical dance has 4 famous styles:
various patterns.
Bharatnatyam, Kathakali, Kathak and Manipuri, some other dances
(ii) Nritya It corresponds to the song. In other got Classical dance position later.
words it is the interpretative dance and is
essentially expressional. At present, in Sangeet Natak Academy, Indian government has given
8 dances the position of classical dance. These are Bharatnatyam,
(iii) Natya It means dramatic representation with
Kuchipudi, Kathakali, Manipuri, Sattriya, Kathak and Mohiniattam.
speech, music and dance.
In classical dances there is very importance of rasa 1. BHARATNATYAM
(pleasure) and bhava (emotion) and create joy, Bharata Muni is propounder of this dance. It is very traditional dance
vigour and happiness. and is considered to be over 2000 years old. The development of this
dance form can be seen in Madras and Thanjavur in Tamil Nadu.
RASA
It is explained as quality of art between the dancer There are three fundamental elements of this dance. These are
and spectators. Any art without a rasa is incomplete. ––Bhava (Imitations of emotions), Raga (Music) and Taal (Rhythm
Dance without rasa will be purely a mechanical time). In this form of dance the three fundamental essence of life is
exercise of body motions and physical movements. present ––Philosophy, Religion and Science.
ART AND CULTURE 1-325

It is believed that this dance form SOME FACTS ABOUT The theme involves mythological
was evolved from ‘Sadir dance’ KUCHIPUDI DANCE presentation on eternal conflict between
which was performed by Devdasis ˜
Kuchipudi dance is related to a Telugu the good and evil in a good manner.
(temple dancer). The teacher of this short drama known as Atabhagavatam. SOME FACTS ABOUT
dance art was called ‘Nattuvanar’. based on religious narrative story or KATHAKALI DANCE
There are many elements of conversation. ˜
It is an exciting dance and dancers
Bharatnatyam like ˜
‘Bhamakalapam’ and ‘Gollakalapam’ are have a perfect control and
Alaripu An invocation to the Gods to related to Kuchipudi dance. coordination with their sensation and
bless the performance. ˜
The dance form was initially performed in mechanical movements.
Jatiswaram An abstract dance where groups by male artists moving from ˜
Kathakali dance starts with Kelikott
the drums set the beat and village to village. These male artists were which attracts spectators. Further it
dancer elaborate foot work and called ‘Bhagavatalu’. It was under the proceeds with Todyam with an
graceful movement of the body. impact of Vaishnavism that the themes invocation to the Gods to bless the
Shabdam The dancing is began to be based on Bhagavata Purana. perfomance. Purapada is pure dance
accompanied by a poem or song ˜
In this dance each principle character form after this Melappadam plays.
with a devotional or amorous introduces itself on the stage with a daarn ˜
The story of Kathakali is based on
themes. which is a small composition of dance and epics and puranas. The drama was
Varnam The centre piece of the song especially designed to help him or expanded with episodes from the
performance. It is the longest her to reveal own identity or skills. Mahabharata and Shiva Purana.
section of the dance punctuated ˜
In Kuchipudi, there are parts of Lasya ˜
With the breakdown of the old feudal
with the most complex and and Tandava dance or Folk dance and set up the dance drama began to
difficult movements. Positions of Classical dance. From there some in parts decline due to lack of patronage.
the hands and body tell a story, paintings are made like— ‘Manduka
usually of love and the longing
˜
The great Malayalam poet Vallathol
shabdam’ the story of the frog. ‘Bala
for the lover. Narayana Menon made it his mission
gopala tarangam’ involving dancing with
to revive the ancient dance form.
Tillana The final section is a pure the feet on the edges of a brass plate. ‘Tala
dance (Nritta) when the virtuosity chitra nritya’ dancers drawing pictures on
˜
Kathakali also called as the ‘Ballet of
of the music is reflected in the the floor with their dancing toes. East’. No other dance in India is
complex footwork and captivating similar to the Kathakali. The reason
˜
The most popular form of Kuchipudi is
poses of the dancer. behind is that in this dance, fingers,
Matka in which the dancer dances on a
eyes, pupils all have movement. The
˜
It is female dominated dance. bronze plate with earthen pot (matka)on
expression of this dance comprises a
They form their postures like his head.
rasa and 64 postures.
idols and goddess. Emphasis is on ˜
Dashavatar Shabdam, Manduka
hand movement to convey Shabdam, Prahlada Shabdam, Sri Rama 4. ODISSI
different kinds of emotions or Pattabhishekam are some dancing styles It presents the earliest evidence of
rasa. In this artists visualise his of Kuchipudi. dance in India. It derives its name from
or her body as made up of ˜
Lakshminarayana Sastry’s honoured Odra Nritya which is mentioned in
triangles. Kuchipudi in 20th century. The most Natyashastra of Bharata Muni. Initially
˜
Rukmini Devi Arundale nurtured prominent teacher of this dance is Chenna this dance was performed by Devdasi or
this dance. T Balasaraswati is an Satyam. Maharish. The Jain King Kharvela
excellent dancer. Yamini ˜
Famous exponents of Kuchipudi are patronised this dance.
Krishnamurthy, Sonal Vempati Chenna Satyam, Radha Reddy, SOME FACTS ABOUT ODISSI DANCE
Mansingh, Padma Yamini Krishnamurthy, Swapnasundari,
Subrahmanyam, Leela Samson
˜
The themes of Odissi include Radha -
Chinta Krishnamurthy, Vijaya Krishna’s love, Dashavatar and
etc are some excellent artists. Prasad etc. Jagannath’s philosophy. The extract
2. KUCHIPUDI 3. KATHAKALI of Gita Govinda is played by dancers.
This is a famous classical dance of This is a famous classical dance of Kerala.
˜
The dancing of Maharis was an
Andhra Pradesh. The name of this It is a combination of dance and theatre. It indispensible part of religious rituals
dance is derived from a village is one of the oldest form of dances in the in temples.
name ‘Kuchelapuram’ of Andhra world. The only sanskrit theatre style, ˜
The ascetic followers of Vaishnavism
Pradesh. It is a male dominated called ‘Taiyam’ performed by Chakyar caste disapproving of the immoralities of
dance form. Chinta Krishnamurthy of Kerala. the devdasi system, introduced the
and Tadepalli Peraiah nurtured practice of getting young boys to
The main sources of Kathakali (‘katha’
this dance. In this dance there is dance on the ritual dances in temples.
means ‘story’ and ‘kali’ means ‘drama’)
special focus on movements of feet, The boy dancers were known as
were Kudiattam and Krishnattam folk
hand, neck and other body parts. Gotipuas.
drama traditions.
1-326 GSP General Studies (Paper 1)

˜
Thus over time three schools of SOME FACTS ABOUT principled and pure classical form. It
Odissi dance developed MANIPURI DANCE is interesting that Kathak dance
Mahari – Devdasi tradition ˜
There are 64 types of rasa present in the involves without bending or flexing
Naratala – In royal courts dance. of knees. Tandava and Lasya are
Gotipua – Young boys dressed up in ˜
The main musical instrument is the two important parts of this dance.
female attire and enacting female Pung or the Manipuri classical drum. ˜
Kathak dance starts with prayer of
roles Besides this Kartal Cholam and Dhol Lord Ganesha and ‘Salami’. Salami
˜
It is similar to Bharatnatyam in Cholam is another form of this dance. is a Mughal tradition. Further it
terms of mudras and facial ˜
Rabindranath Tagore introduced this proceeds with Aamad in which male
expressions. Odissi has been aptly dance in Shantiniketan in 19th century. and female dancers come on the
termed as ‘mobile sculptures’ stage. Thaat is characterised by slow
˜
The ‘pena’ a stringed instrument is used
because of its graceful, sensuous and dance while Toda is lyrical dance.
in Lai Haraoba and Pena singing.
lyrical dance style. Gaat Nikas introduces about the
˜
Women dancers use Lehanga, called context of dance.
˜
The concept of Tribhanga (three Kumin. It is made by a transparent silk
bends) expressed in Indian sculpture
˜
The most significant feature of this
cloth called ‘Pason’.
is innate to it. The dancer move dance is the single saying or dialogue
˜
The martial dancers of Manipur, the is performed by different postures
either in front or back or in a circular
Thang-ta have their origins in the days and mechanical movement of hands.
pattern creating intricate
when man’s survival depended on his Dancer uses more than 100 ankle bell
geometrical shapes.
ability to defend himself from wild and dances in circular path.
˜
This dance is characterised by animals. Sometimes they become faint during
various Bhangas (stances) which ˜
Today, Manipur has an evolved and this dance. These are some
involves stamping of the feet and
sophisticated repertoire of martial characteristic features of this dance.
striking various postures as seen in
dances, the dancers use swords, spears ˜
Some of the famous Kathak dancers
Indian sculputres. The common
and shields. are Birju Maharaj, Sitara Devi,
Bhanga are –Bhanga, Abhanga,
Atibhanga and Tribhanga.
˜
‘On Life of Vishnu’ is the theme of Damyanti Joshi, Kartik Ram, Gopi
Manipuri dance. Krishna etc.
˜
Hindustani music is popular in
˜
The famous Manipuri dancers are: Famous gharanas of Kathak dance
Odissi dance.
Jhaveri Sister, Guru Bipin Singh, Lucknow gharana Kathak achieved a
˜
Bhangi and Karan are two elements
Poushali Chatterjee, Sohini Roy, peak in the reign of Nawab Wajid
of this dance.
Suvarna, Ranjana, Darshana, Charu Ali Shah in Lucknow, who himself
˜
Some of the famous Odissi dancers Mathur etc. was a student of Kathak. This
are Guru Pankaj Charan Das,
gharana emphasised dramatic and
Kelucharan Mohapatra, Sonal 6. KATHAK sensuous expression.
Mansingh, Sanjukta Panigrahi etc. It is a dance form born in Uttar Pradesh.
Its origin can be traced in Rasleela of Jaipur gharana It was launched by
5. MANIPURI Braj Bhumi. It derives its name from Bhanuji. It emphasises fluency,
Manipuri dance, being indigenous to ‘kathika’ or story tellers, who presented speed and long rhythmic patterns as
Manipur is inextricably woven into the with gestures verses from the epics. The it emphasises technical mastery of
life pattern of the people. It is dance was originally associated with pure dance.
influenced by Vaishnavism. In this temples with the central concept of Varanasi gharana The gharana of
Radha Krishna’s love is played by Krishna as the divine dancer and Radha Varanasi is said to have evolved in
actors. It is one of the most beautiful as his partner. It was influenced by Rajasthan and it developed a style
dance of India. ‘King Bhagya Chandra’ Vaishnavism and accompained by of its own characterised by moderate
and ‘Kirti Chandra’ patronaged this keertans. Kathak branched off into the tempo, grace and precision.
dance. Lai Haraoba which is courtly stream under the Mughal rulers Rajgarh gharana It is not so well-known
pertaining to the universe is and was influenced by persian costumes and is said to have developed under
considered as primary phase of this and style of dancing. the patronage of Raja Chakradhar
dance.
SOME FACTS ABOUT KATHAK DANCE Singh.
The Manipuri style emphasises
Bhakti, not the sensuous aspect. Both
˜
In modern time Leela Sokhe (Menka) 7. SATTRIYA
Tandava and Lasya are involved in revived and honoured this dance and The Sattriya dance form was
this dance but Lasya predominates. established a Kathak training centre at introduced in the 15th century AD by
The most striking part of Manipuri Khandala. great Vaishnav saint and reformer of
dance is its colourful decoration, ˜
Kathak dance involves intricate Asom, Mahapurusha Shankardev for
lightness of dancing foot, delicacy footwork and pirouettes. The themes of propagation of Vaishnava faith. It was
of drama, lifting music and poetic song range from Dhrupads to Tarana, recognised as classical dance by
charm. Thumris and Ghazals. Kathak is very Government of India in year 2000.
ART AND CULTURE 1-327

SOME FACTS ABOUT CENTRAL INDIA This is basically a community dance for
SATTRIYA DANCE GAUR women and performed on auspicious
˜
Sattriya, is derived from the word occasions. In Haryana, the songs sung
It is a popular folk dance of Madhya
‘Satra.’ In Assamese ‘Satra’ means, for Ghoomar are high-pitched and rich
Pradesh and is popular in the Sing
a hermitage. Initially this dance in humour and satire. In Rajasthan,
Marias or Tallaguda Marias of South
was associated with ‘Ankia’ drama Ghoomar is performed to the songs of
Bastar. It involves men wearing
propounded and evolved by valour and victory.
head-dresses, with stringed cowries and
Shankardev. plumes of peacock feathers, making their DUMHAL
˜
The theme of Sattriya dance is story way to the dancing ground. The dance of the Kashmiri is called as
of puranas, which was composed Dumhal. It is performed by dancers
and expanded by Shankardev and MURIA
wearing long colourful robes and tall
his pupil Mahadev. It is called The Muria tribals of North Bastar area
conical caps, studded with beads and
‘Borgeet’. Traditionally this dance are known for performing the folk dances
shells. It is mainly the men folk of
was performed by ‘Bhogot’ (male of Muria. These dances start with an
Wattal, who perform this dance, that
hermit) only on religious invocation or prayer to the phallic deity
too on specific occasions.
ceremonies. Later women, also of their tribe and the founder of the
entered into it. Ghotul institution. One of the popular HIKAT
˜
Sattriya dance has many forms— Muria dances comprises of Har Endanna It is performed by women and comes
Apsara dance, Behar dance, Chali dance, performed by boys and girls across as a modification of a game
dance, Dashavatar dance. during marriages. played by children. Forming pairs, the
Toor Geet is based on classical ragas. participants extend their arms to the
˜
SAILA
front, gripping each other’s wrists.
˜
Famous Sattriya dancers are Jatin This dance is the dance form of With their body inclined back, they go
Goswami, Maniram Muktiyar, Chhattisgarh and is performed by round and round at the same spot.
Bapu Ram Muktiyar, Manik young boys in the post-harvest time. It is Namagen is another folk dance, that is
Barbian. basically a stick-dance, in which dancers specific to the State of Himachal
(each standing on one leg and supporting Pradesh.
8. MOHINIATTAM
himself by holding on to the man in front)
Mohiniattam from Kerala is a solo HURKA BAUL
form a circle. Then, all of them jump
female dance and is known for its It is performed during paddy and maize
together, going round and round.
rhythmic and unbroken flow of the cultivation, in different fields, by turns.
body movements. Mohiniattam is KARMA Its name has been derived from the
derived from the words-Mohini Popular among the Gonds and Baigas of terms Hurka, the drum which
(meaning beautiful women) and Chhattisgarh and the Oraons of constitutes the only musical
Attam (meaning dance). Madhya Pradesh, Karma dance is accompaniment to the dance and baul,
It developed in the tradition of associated with the fertility cult. These the song. The singer narrates the story
Devadasi system, which later grew dance forms represents the coming of of battles and heroic deeds, while the
and developed a classical status. green branches on trees, during the dancers enter from two opposite sides
spring season. It is related to the Karma and enact the stories, in a series of
Mohiniattam has the grace and
festival, which falls in the month of crisp movements.
elegance of Bharatnatyam and vigour
August.
of Kathakali. CHHOLIYA
Famous Exponents of KAKSAR A famous dance of Kumaon region,
Mohiniattam are Kalyani Amma, This dance is performed by people in Chholiya is mainly performed during
Vaijayanthimala, Bharati Shivaji and hope of reaping a rich harvest and is marriages. As the groom’s procession
Hema Malini. popular among the Abhujmarias of proceeds to the bride’s house, male
Bastar. It is mainly undertaken by dancers, armed with swords and
FOLK DANCES OF INDIA young boys and girls, in order to invoke shields, dance spiritedly.
the blessings of the deity.
Folk dance can be defined in the BHANGRA
simplest words as a form of dance, NORTHERN INDIA One of the most popular dances of
developed by group of people, which GHOOMAR North India, performed during the
reflects the traditional life of the
The performers carry earthen pots and festival of Baisakhi is the Bhangra.
people of a certain country or region.
dance with slick movements of the hands Among the most virile and captivating
This form of dancing involves a group
and feet. Although, this is essentially a dances of India, it is undertaken by
of happy people, following dance
group dance, sometimes performers show men and includes tricks and acrobatic
instructions given by an experienced
their skills by dancing independently. feats. It is performed in Punjab.
caller.
1-328 GSP General Studies (Paper 1)

GIDDHA NORTH-WEST INDIAN FOLK DANCES


The dance performed by the women folk of Punjab is called the KALBELIA DANCE
Giddha. In this dance form, a woman or a pair of women dances One of the most sensuous dance forms of Rajasthan,
at a time, while the others surround them and clap in rhythm. performed by the Kalbelia snakecharmers community.
The dance is mainly performed during the festival of Teeyan to The Kalbelia dancers wear long black skirts embroidered
welcome the rains. with silver ribbons. As they spin in circle, their body sways
DHAMYAL acrobatically, sway sinuously to the accompaniment of
The folk dance of Haryana is known as the Dhamyal or the pungi, dufli and plaintive notes of the been, the wooden
Duph. The dance can be performed by men alone as well as with instrument of the snake charmers. So that, it is impossible
women. The Duph, after which the dance form is named, is a to believe that they are made of anything other than
circular drum, played nimbly by the male dancers, as they rubber. As the beat increases to such high pitch, free
dance. flowing voice, while others join in the dance. The vigorous
and zestful display of their perfect movements to the
NORTH-EASTERN INDIA enchanting tune of musical instruments is a treat to the
eyes. In 2010, the Kalbelia folk songs and dances of
BIHU (ASSAM) Rajasthan were declared a part of its intangible heritage
The folk dance of Assam is called Bihu. Every Assamese, young list by the UNESCO.
or old, rich or poor, takes delight in the dance, which forms a
part of the Bihu festival. The festival comes in mid-April, DANDIYA (GUJARAT)
during the harvesting time (which lasts for a month). Dandiya is the folk dance of Gujarat, which shows the
great vigour and joy of the inhabitants of the North-West
HOJAGIRI (TRIPURA) states of India. Dressed in colourful costumes, the people
Hojagiri is the folk dance of Tripura, the land of a large tribal of the desert play Dandiya gracefully, by holding big sticks
population. The dance is performed by young girls, who in their hands. Apart from being a traditional dance form
demonstrate a series of balancing skills and uses instruments of the state, Dandiya also features in the Navratras, a
of its kind. 9-day festival that is observed with pomp and gaiety
The dances are a part of the ceremony to appease the Goddess across many parts of the country.
Lakshmi, to ensure a happy harvest, as cultivation forms a TARPHA NACH OR PAVRI NACH
main source of the tribe’s livelihood. It is the dance of the Kokna tribals native to the hilly
THANGTA AND DHOL-CHOLOM (MANIPUR) regions of the North-West India. These dances derive their
names from the wind instruments of Tharpa or Pavri,
The Thangta dance of Manipur was an evolution from the
which are made of dried gourd, played during the dance
martial arts exercises encouraged by the kings of Manipur. The
performance.
dance is exciting in nature and is performed by young men
holding swords and shields. One of the instruments that TERA TALI (RAJASTHAN)
dominate Manipuri dances is the drum. Dhol Cholom, a drum Native to the Kamar tribe of North-West India, Tera Tali
dance, is one of the dances performed during Holi. is performed by two or three women. The performers sit on
the ground, while performing the Tera Tali, which is an
NONGKREM (MEGHALAYA)
elaborate ritual followed in the region. Small metal
To celebrate the remembrance of the evolution of Khasis and
cymbals called Manjiras are tied to different parts of the
their indigenous democratic state called Hima, Nongkrem
body, mostly on the legs of the dance performer.
dance is performed in Meghalaya, during autumn. The Khasis
are a tribe of Meghalaya, who also celebrate the ripening of SOUTH INDIA
paddy for threshing by dances and songs. PADAYANI OR PADDENI (KERALA)
FOLK DANCE OF ARUNACHAL PRADESH One of the most colourful and enchanting dances of
In Arunachal Pradesh, an organised group of tribal performers Kerala, Padayani or Paddeni is associated with the
perform dances, plays, musical scripts and dance-dramas, festivals of certain temples. These temples are found in the
based on stories of Lord Buddha. The dancers wear masks of districts of Alleppey, Quilon, Pathanamthitta and
demons or animals, as described in the tales of Buddha, Kottayam districts. The literal meaning of Padayani in
alongwith splendid costumes. These dances are mostly folk art is military formations or rows of army.
performed in monasteries, during festivals.
KUMMI AND KOLATTAM (TAMIL NADU)
FOLK DANCES OF SIKKIM Kummi and Kolattam are the dances performed by the
In Sikkim, the men are attracted more towards the monastic tribal women of Tamil Nadu, during certain festivals.
style of dancing, while the women have their own folk dance Kummi is a very simple form of dance, where dancers form
style. The dances of Sikkim are different than those of Indian circles and clap as they dance. Kolattam is also quite
traditions. Masks used in dances are something close to Indian similar, the only difference being the use of small wooden
cultural dances. rods by the dancers.
ART AND CULTURE 1-329

KARGAM AND PULI VESHAM courts or villages, dance has moved into the auditorium of today,
(TAMIL NADU) bringing pleasure to many more people, in far-flung regions. This
One of the most essential parts of a ritual dedicated vast sub-continent has given birth to varied forms of dancing, each
to Mariamma, the Goddess of health and rain in the shaped by the influences of a particular period and environment.
state of Tamil Nadu is the Kargam dance. It is These pristine forms have been preserved through the centuries, to
performed by men, wherein they balance pots, filled become a part of our present culture, a living heritage, our pride and
with uncooked rice and surrounded by a tall conical delight.
bamboo frame covered with flowers, on their head.
Folk Dances and Tribal Dances in India
Mridangam and nathaswaram form the musical
instruments that accompany the dance. Puli Vesham State Dance (s)
is another very interesting dance of Tamil Nadu, Maharashtra Kathakeertan, Lezin, Dandaniya, Tamasha, Gafa, Dahikala,
which is performed by men during a specific festival. Lavani, Mauni, Dasavtar
In this dance form, the men disguise themselves in Karnataka Huttari, Suggi Kunitha, Yakshagana
tiger costumes and move around the streets. Kerala Kaikottikali, Kaliyattam, Tappatikkali
Tamil Nadu Kolattam, Pinnal Kolattam, Kummi, Kavadi, Karagam
SOUTH-WEST INDIA DOLLU KUNITHA Andhra Pradesh Ghanta Mardala, Veedhi Natakam, Burrakatha
(KARNATAKA) Odisha Ghumara Sanchar, Chadya Dandanata, Chhau
It is a popular drum dance of Karnataka in which, West Bengal Kathi, Chhau, Baul, Kirtan, Jatra, Lama
large drums are decorated with coloured cloth and Assam Bihu, Khel Gopal, Rasalila, Tabal Chongli, Canoe
slung around the necks of men. The tribes of Punjab Giddha (women), Bhangra (men)
Karnataka, basically comprising of hunters and food Jammu and Kashmir Rauf, Hikat
gatherers, stocked with a regular précis of songs and Himachal Pradesh Jhora, Jhali, Dangli, Mahasu, Jadda, Jhainta, Chharhi
dances related to hunting, food gathering and burial
Haryana Jhumar, Rasalila, Phag dance, Daph, Dhamal, Loor, Gugga,
funeral rites. Khoria, Gagor, Swang
Gujarat Garba, Dandiya Raas, Tippani, Gomph
RITUAL DANCES (KARNATAKA)
Rajasthan Ginad, Chakri, Gangore, Terah Taal, Khayal, Jhulan Leela,
Amongst the ritual dances of Karnataka, Kavadis Jhuma, Suisini
performed for the worship of Lord Subramanya.
Bihar Jata Jatin, Jadur, Chhau, Kathaputli, Bakho, Jhijhiya,
Then, at harvest time, the Dodavas of Karnataka Samochakwa, Karma, Jatra, Natna
perform the Balakat dance. Apart from that, we have Uttar Pradesh Nautanki, Thora, Chappeli, Rasalila, Kajri
Devare Thatte Kunitha, Yell-Ammana Kunitha and Madhya Pradesh Karma
Suggikunitha, which are dances related to the name
Meghalaya Laho
of a deity or instrument balanced on the head or held
Goa Mando
in the hand.
Mizoram Khantumm
GHODE MODNI (GOA) Uttarakhand Garhwali
Goa was ruled by the Portuguese for many years.
Hence, the European influence is quite evident in the
annual carnival and the folk dance performed
DRAMA AND THEATRE
therein, known as Ghode Modni (dummy horse Drama has also been practised since times immemorial. The word,
presentation). The dance brings forth the brave deeds Drama, could spring from a child’s play. Theatre in India started as
of the Goan warriors, where the attractively dressed a narrative form, with recitation, singing and dancing becoming
dancers perform armed with swords. integral elements of the theatre. This emphasis on narrative
elements made our theatre essentially theatrical right from the
TARANGMEL (GOA) beginning. While drama is a broad term consisting of both written
While performing Tarangmel, the energetic young plays and performances, theatre is a specific form which involves
girls and boys crowd the streets in colourful group, live performances in front of an audience.
with flags and streamers or tarang in their hands.
This group of young dancers invites everyone to join HISTORY OF THEATRE IN INDIA
the festive spirit.
India has a long and rich tradition in theatre going back to 5000
LAVA DANCE OF MINICOY (LAKSHADWEEP) years. The origin of Indian theatre is closely related to ancient
Minicoy is renowned for its tradition of the Lava rituals and seasonal festivities of the country. Bharata’s
dance, performed during the festive occasions on the Natyashastra (2000 BC to 4th century AD) was the earliest and
island. Lava dance is a very colourful and energetic most elaborate treatise on dramaturgy written anywhere in the
dance of Lakshadweep, in which the dancers are world. The traditional account in Bharata’s Natyashastra gives a
dressed in multi-hued costumes and headgears. They divine origin to Indian theatre, attributing it to the Natyaveda, the
also carry a drum while dancing. In this form of holy book of dramaturgy created by Lord Brahma.
dance, the participants perform to the rhythmic beats In Natyashastra, Bharata Muni consolidated and codified various
of drums and songs. Nurtured in temples, princely traditions in dance, mime and drama.
1-330 GSP General Studies (Paper 1)

Natyashastra describes ten classifications Sanskrit theatre was characterised FORMS OF TRADITIONAL
of dramas ranging from one act to ten by its high degree of refinement in
acts. No book of ancient times in the performance technique. It followed THEATRE
world contains such an exhaustive study well-atriculated, aesthetic principles, BHAND PATHER
on dramaturgy as the Natyashastra. It is usually those laid out in the ancient It is the traditional theatre form of
addressed to the playwright, the director dramatics texts. It depended on a Kashmir and a unique combination of
and the actor because, to Bharata Muni high degree of audience knowledge dance, music and acting. Satire, wit
these three were inseparable in the and expertise i.e. only the refined and parody are preferred for inducing
creation of a drama. The Sanskrit word sensibility could appreciate it. laughter. In this theatre form, music is
for drama nataka, derives from the word Religion played an important role in provided with Surnai, Nagaara and
meaning ‘dance’. In traditional Hindu drama as certain rituals accompanied Dhol.
drama, expression was achieved through most plays and even the stage was KRISHNATTAM
music and dancing as well as through consecrated before a performance. It is the folk theatre of Kerala which
acting, so that a play was a combination Thus, the Sanskrit drama could be came into existence in the middle of
of opera, ballet and drama. called an Amalgamation of the 17th century AD under the patronage
religious, deductional and of King Manavada of Calicut.
TYPES OF THEATRE IN INDIA entertaining elements.
The theatre in India has encompassed all MUDIYETTU
the other forms of literature and fine arts TRADITIONAL THEATRE It is the traditional folk theatre form of
into its physical presentation. Literature, Kerala and is celebrated in the month
In different regions of India, there
mime, music, dance, movement, painting, of Vrischikam (November-December).
are religious festivals, fairs,
sculpture and architecture, all mixed into It is usually performed only in the
gatherings, ritual offerings, prayers,
one and being called Natya or Theatre in Khali temples of Kerala.
almost throughout the year.
English. Roughly the Indian theatre can
be divided into three distinctive kinds: During these occasions, traditional SWANG
the Classical or the Sanskrit theatre, theatre forms are presented. They Traditionally, this form was mainly
the Traditional or the Folk theatre and reflect the common man’s social music-based. Gradually, prose too,
the Modern theatre. attitudes and perceptions. In this played its role in the dialogues. The
social portrayal, there is also the softness of emotions, accomplishment
SANSKRIT PLAYS individual’s role, which is given due of rasa alongwith the development of
importance. character can be seen in this theatre
It is difficult to determine the precise
form.
origins of the Sanskrit drama. Fragments In traditional theatre forms, there
of the earliest known plays have been are special styles of dance portraying NAUTANKI
traced from the 1st century AD. However, the entry on to the stage or platform, It is usually associated with Uttar
scholars believe that a living theatre narrative and descriptive roles. The Pradesh. The most popular centres of
tradition must have existed in India best example of descriptive acting is this traditional theatre form are
much earlier. The earliest phase of the Vidyapati naach. Kanpur, Lucknow and Hathras. Gulab
Sanskrit theatre includes the writing and In this traditional theatre form, Bai of Kanpur gave a new dimension
practice of theatre up to about 1000 AD emphasis is not on beauty, but on to this old theatre form.
based almost entirely on the rules, acting itself and on narrative and RASALILA
regulations and modifications laid down descriptive skills. Dance as a
It is based exclusively on Lord
in the Natyashastra. narrative art is the base of theatre
Krishna’s legends. It is believed that
Bhasa was the earliest dramatist while form,which can be seen in the
Nand Das wrote the initial plays based
the climax of Sanskrit drama reached in traditional theatre form of Bhavai of
or the life of Krishna.In this theatre
Abhigyan Shakuntalam of Kalidasa and Gujarat. In this form, quick or slow
form, the dialogues in prose combine
Mrichchhkatikam of Shudrak. The foot movement is a means of
beautifully with songs and scenes from
Sanskrit plays were limited by certain narration. The art of making the
Krishna’s pranks.
conventions. Tragedy was taboo and the entry by dancing has been perfected
end was always happy. There was no in the traditional Kashmiri theatre BHAVAI
place for plays that raised controversies form, Bhand Jashn. It is the traditional theatre form of
(although Bhasa has shown death on the The way each character walks and Gujarat. The centres of this form are
stage in one of his plays). enters the platform, identifies him. Kutch and Kathiawar.
In Koodiyaattam and Ankia Naat, JATRA
The basic plot in most Sanskrit plays
the entry by dancing itself is
centre around the hero who struggles for This form was born and nurtured in
complicated and artistic. In the
(and finally obtains) the object of his Bengal. It is a musical play which was
forms, the tempo and basic posture
desire. The realisation of this goal is part of fairs in honour of Gods or
and gesture identifies the role of the
closely entwined with the three ends of religious rituals and ceremonies.
character.
Hindu life–duty, pleasure and wealth.
ART AND CULTURE 1-331

Krishna Jatra became popular due to


Chaitanya’s influence. Later,
MODERN INDIAN THEATRE PUPPETRY
Modern Indian theatre, as we know it The early puppet shows in India dealt
however, worldly love stories too,
today, has a legacy that is influenced mostly with histories of great kings,
found a place in Jatra.
by and draws inspiration from various princes and heroes and also political
MAACH sources. Modern theatre or satire in rural areas. Slowly, this art form
It is the traditional theatre form of historically, what can be clearly emerged from the precincts of the temples
Madhya Pradesh. The term Maach identified as the Western and villages to reach out to the outside
is used for the stage itself as also for Proscenium Style of theatre, was world performing on various social and
the play. In this theatre form, songs not introduced in India before the late contemporary themes in Indian towns and
are given prominence in between the 18th century at time of the cities.
dialogues. consolidation of the British empire in
TYPES OF TRADITIONAL
various parts of India.
BHAONA INDIAN PUPPETS
It was through the British that The different traditional forms are glove,
It is a presentation of the Ankia Naat Western Proscenium Style Theatre
of Assam. In Bhaona, cultural rod, string and shadow puppets. The local
reached Indian shores. In the 1830s, name given to puppetry varies from state
glimpses of Assam, Bengal, Odisha, under the patronage of the rich native
Mathura and Brindavan can be seen. to state within India.
families, we had the first Bengali
The Sutradhaar or narrator begins language theatre, which was outside GLOVE
the story, first in Sanskrit and then the traditional format of indigenous The glove puppet, also known as Hand
in either Brajboli or Assamese. folk performance genres. Puppets, is a small figure having head
TAMAASHA and arms with a long skirt as part of its
NATIONAL SCHOOL OF costume. The puppeteer wears the puppet
It is a traditional folk theatre form of DRAMA (NSD) like a glove and fits the index finger into
Maharashtra. It has evolved from It is a theatre training institute the hollow head of the puppet.
the folk forms such as Gondhal, situated at New Delhi, India. It is an
Jagran and Kirtan. autonomous organisation under ROD
Unlike other theatre forms, in Ministry of Culture, Government of The rod puppets, often larger than glove
Tamaasha, the female actress is the India. It was set up in 1959 by the puppets are supported and manipulated
chief exponent of dance movements in Sangeet Natak Akademi and became by rods of various types and sizes. In rod
the play. She is known as Murki. an independent school in 1975. In puppets, the action of the rods are
2005, it was granted deemed concealed in many ways. The main
DASHAVATAR university status, but in 2011, it was holding rod that supports the puppet may
It is the most developed theatre form revoked on the institute’s request. be hidden by a robe or costume of the
of the Konkan and Goa regions. In a bid to decentralise its activities, puppet. The action rods are usually
The performers personify the 10 NSD opened Regional Resource Centres connected to the hands of the puppet and
incarnations of Lord Vishnu the God (RRC) across India, the first of which manipulated by the puppeteer to show
of Preservation and Creativity. was opened at Bengaluru in 1994. action.

THEYYAM SANSKAAR RANG TOLI SHADOW


It is a traditional and extremely In 1989, NSD established the The shadow puppets are flat puppets, that
popular folk theatre form of Kerala. Theatre-In- Education Company are operated against the rear of a tightly
The word Theyyam is derived from (TIE), also called Sanskaar Rang Toli. stretched white cloth screen with a light
the Sanskrit word, Daivam meaning Today, it has become India’s premier behind it, so that the audience looks at the
‘God’. educational resource centres and shadow of the puppet on the screen.
coaches children aged 8 to 16 years. Traditional shadow puppets are two
KOODIYATTAM dimensional and are made of animal skin.
It is one of the oldest traditional The puppet shapes or cutouts are
Theatre Festivals in India
theatre forms of Kerala. It is based perforated and split bamboo or cane sticks
on Sanskrit theatre traditions.
¢
Bharat Rang Mahotsav, New Delhi
are attached vertically to the puppet for
¢
International Theatre Festival of
handling and manipulation. The shadow
YAKSHAGAANA Kerala
Qadir Ali Baire Theatre Festival puppet performance is known as shadow
It is the traditional theatre form of ¢

(leather) play or shadow theatre.


Karnataka. It is based on mytholo-
¢
Kalidas Samaroh, Ujjain
gical stories and Puranas.
¢
Prithvi Theatre Festival, Mumbai STRING
¢
Nandikar Festival, Kolkata
The string puppet also known as
THERUKOOTHU ¢
Nehru Centre’s National Theatre
Marionettes has jointed body and limbs
It is the most popular form of folk Festival, Mumbai
that allow movement. String puppets are
drama of Tamil Nadu. It literally made of wood or wire or cloth stuffed with
means street play. cotton, rags or saw dust.
1-332 GSP General Studies (Paper 1)

The puppet is suspended from a hand held control strings, Road, New Delhi. In April 2015, Government of India took
that are attached to different parts of the puppet’s body. The over management control of Lalit Kala Akademi citing
puppet is manipulated by operating the control as well as by complaints regarding alleged administrative and financial
loosening or pulling the relevant string(s). Some of the irregularities in its functioning.
traditional string puppets are very heavy. SAHITYA AKADEMI
The Sahitya Akademi, India’s National Academy of
POPULAR EXAMPLES OF PUPPETRY Letters, is an organisation dedicated to the promotion of
literature in the languages of India. It was founded on 12th
FORMS March, 1954. The Sahitya Akademi organises national and
Togalu Gombeyatta, Karnataka These puppets are mostly regional workshops and seminars; provides research and
small in size. The puppets, however differ in size, according to travel grants to authors; publishes books and journals,
their social status. including the Encyclopaedia of Indian Literature; and
Tholu Bommalata, Andhra Pradesh This Shadow presents the annual Sahitya Akademi Award of ` 100,000
theatre has the richest and strongest tradition. The puppets (approx. USD 1.500 (as in year 2013)) in each of the 24
are large in size and have a jointed waist, shoulders, elbows languages it supports, as well as the Sahitya Akademi
and knees. They are coloured on both sides. Fellowship for lifetime achievement.

Ravanachhaya, Odisha The most theatrically exciting is the Sahitya Akademi is the central institution in India for
Ravanachhaya of Odisha. The puppets are in one piece and
˜
literary dialogue, publication and promotion in the country
have no joints. They are not coloured, hence, throw opaque ˜
promoting Indian literature throughout the world
shadows on the screen. ˜
keeping alive the intimate dialogue among the various
Putul Nautch, West Bengal The traditional rod puppet form linguistic and literary zones and groups thought
of West Bengal is known as Putul Nautch. They are carved
˜
seminars, lectures, symposia, discussions, readings and
from wood and follow the various artistic styles of a particular performances
region. The puppets of Odisha are smaller than those from ˜
providing research and travel grants to authors
Bengal or Andhra Pradesh. Rod puppet shows of Odisha are ˜
publishes books and journals, including the Encyclopaedia
more operatic and prose dialogues are seldom used. of Indian Literature
Yampuri, Bihar The traditional rod puppet of Bihar is Other than its headquarters of New Delhi, the Akademi has
known as Yampuri. These puppets are made of wood. Unlike four regional offices in Kolkata, Mumbai, Bengaluru and
the traditional rod puppets of West Bengal and Odisha, these Chennai. It has two Translation Centres at Bengaluru and
puppets are in one piece and have no joints. Kolkata, besides a Project.

Major Puppetry Traditions of Various States SANGEET NATAK AKADEMI


The Sangeet Natak Akademi was set up in 1953 for the
Andhra Tholu Bommalata (shadow) and Koyya Bommalata (string)
Pradesh promotion of performing arts. Setting up of Sangeet Natak
Akademi is one of the main recommendations of the
Assam Putul Nach (string, rod)
National conference on dance, drama, and music, held in
Karnataka Gombeyatta (string), Togalu (shadow)
New Delhi in 1951.
Kerala Tholpavakoothu (shadow)
Functions and Powers The Sangeet Natak Akademi is an
Maharashtra Kalasutri Bahulya (string), Chammadyache Bahulya
autonomous body under the Ministry of Culture. It acts at
(shadow)
the national level for the
Odisha Kandel Nach (glove), Rabana Chhaya (shadow), Kathi Kundi
(rod), Gopalila Kamdhei (string)
˜
promotion and growth of Indian music, dance and drama;
Rajasthan Kathputli (string)
˜
maintenance of standards of training in the performing
art;
Tamil Nadu Bommalatam (string), Bommalatam (shadow)
˜
revival, preservation, documentation and dissemination of
West Bengal Putul Nach (rod), Tarer or Sutor Putul (string), Beni Putul
(glove)
materials as well as instruments relating to various forms
of music, dance and drama recognition of outstanding
artists.
NATIONAL CULTURAL INSTITUTIONS The Akademi is also responsible for the establishment and
LALIT KALA AKADEMI maintenance of institutions and projects of national
The Lalit Kala Akademi or National Academy of Art is importance in the field of performing arts. Important
India’s national academy of fine arts. It is an autonomous institutions established by the akademi are
organisation, established in New Delhi in 1954 by government The Jawaharlal Nehru Manipuri Dance Academy in
of India to promote and propagate understanding of Indian Imphal, the premier institution in Manipuri dance and
art, in and outside the country. Lalit Kala Akademi provides music, was established in 1954. The Sattriya Kendra,
scholarships and a fellow program, and sponsors and Guwahati was established in 2001. Projects of National
organises exhibitions in India and overseas. It publishes a importance supported by the Akademi are: Kuttiyattam the
bilingual journal. It is funded by the Union Ministry of age-old Sanskrit theatre of Kerala Chhau dances of Eastern
Culture. Its headquarters are at Ravindra Bhawan, Ferozshah India and Sattriya traditions of Assam.
ART AND CULTURE 1-333

RAM NAVAMI
FESTIVALS AND FAIRS The month of April brings with it the occasion of Baisakhi
and Ram Navami. The birth anniversary of Lord Rama is
in the Hindu month of Chaitra (March-April). This
Our country has a rich cultural heritage and is home household festival is celebrated with pomp in Northern
parts to celebrate the new crop season.
to people from diverse religions. It is a universally
acknowledged truth that every religion gets its own NAG PANCHAMI
unique expression from the fairs and festivals It is celebrated on the 5th day of the Full Moon fortnight
during the Hindu month of Shravana as per the Hindu
celebrated by its followers on various occasions. These calender. Nag Panchami is celebrated as the victory of
fairs and festivals reflect the traditional spirit and Krishna on Kaliya, the most dangerous snake.
culture of the people of India and are a unique symbol RAKSHA BANDHAN
of National Integrity. The festival is all about affection, fraternity and sublime
sentiments which brothers and sisters all over the country
share with each other. It is celebrated on the Full Moon of
the month of Shravana (Shravan Purnima), which falls in
FESTIVALS AND FAIRS the Gregorian month of August. Raksha Bandhan means
A festival is a time for celebration, marked by feasting, the ‘Bond of Protection’ or ‘Knot of Protection’, which all
ceremonies or other observances. However, fairs are periodic brothers vows to their respective sisters.
gatherings of people away from their place of residence, often
with entertainment. The fairs and festivals in India are a GANESH CHATURTHI
wonderful statement of the country’s diversity and multi-ethnic This festival is celebrated on the 4th day of the waxing
character. Moon of Hindu month-Bhadra (August-September). It is a
festival totally dedicated to Lord Ganesha. Popularised by
FESTIVALS OF HINDUS Tilak, it has achieved the status of the most important of
festival of Maharashtra.
MAKAR SANKRANTI
In the month of January, the festival of Sankranti or Makar JANMASHTAMI
Sankranti , is celebrated as Pongal in South India. According to The Janmashtami festival celebrates the birth of Lord
the solar calendar, the festival traditionally coincides with the Krishna, a revered Hindu deity. In Maharashtra, the
beginning of the Sun’s Northward journey (Uttarayan) and falls celebration is in the form of a two-day festival called
on the 14th of January. Govinda. During the festival, an Earthen pot is tied high
In some states, this festival is celebrated by special offering of up on different streets and groups of people create
pyramid-like structures to get their hands on this pot.
rice and pulses cooked together with and without jaggery and
clarified butter. DUSSEHRA/NAVRATRI
VASANT PANCHAMI According to the Hindu calendar, it is celebrated on the
10th day of the Shukla Paksha bright half of the month of
This festival is celebrated in the Northern parts of India to
Ashvin (Ashwayuja). The word dashhara comes from
celebrate the arrival of spring, whereas in Eastern India, this
‘Dussehra’ where ‘dash’ means Ten (headed) and ‘hara’
day is celebrated as Saraswati Puja, by offering prayers to the
means defeated. Dussehra celebrates the victory of good
Goddess of learning and wisdom. The 5th day of bright half of
over evil. It is celebrated in different ways. It is done
the month of Magha, (January-February) is reserved for the
through worshipping the Goddess Chamundeshwari
worship of Saraswati, the Goddess of learning.
(Durga). Chamunda (Chamundi or Charchika) got her
HOLI name as a combination of Chanda and Munda, the two
It is celebrated on the day of Full Moon in Falgun monsters she killed.
(February-March). It is an ancient festival of India and was DIWALI
originally known an ‘Holika’. Then on the eve of Holi, Holika Deepawali or Diwali is certainly the biggest and the
Dahan takes place, in which, an effigy of Holika, the brightest of all the Hindu festivals. It is the festival of
devil-minded sister of demon king Hiranyakashyap, is placed in lights (deep meaning, light and avail meaning, a row i.e. a
the woods and burnt. For Holika tried to kill Hiranyakashyap’s row of lights) that is marked by 4 days of celebration,
son Prahlad, an ardent devotee of Lord Narayana. which literally illumines the country with its brilliance
The ritual symbolises the victory of good over evil and also the and dazzles all with its joy. There are various legends
triumph of a true devotee. Holi is also associated with the pointing to the origin of Diwali or ‘Deepawali’. Some
divine love of Radha and Krishna and is celebrated with believe it to be the celebration of the marriage of Lakshmi
particular vigour in Mathura, the birth place of Krishna. with Lord Vishnu.
1-334 GSP General Studies (Paper 1)

Whereas in Bengal, the festival is During the celebration of Id-ul-Azha, the Baisakhi falls on the 1st day of
dedicated to the worship of Mother Muslims remember Hazrat- lbrahim’s Baisakh month (April-May), according
Kali, the Dark Goddess of Strength. trial by slaughtering an animal such as to Nanakshahi or Sikh calendar.
Lord Ganesha, the elephant headed sheep, camel, cow and goat. Baisakhi is a seasonal festival
God, the symbol of auspiciousness and People think that Allah has given them celebrated all over the state of Punjab
wisdom, is also worshipped in most power over the animal and allowed them and few other parts of India.
Hindu homes on this day. to eat its meat, but only if they GURPURAB
In Jainism, Deepawali has an added pronounce his name at the sole act of
Guru Nanak Jayanti or Gurpurab is
significance to the great event of Lord taking life.
the most auspicious festival of Sikh
Mahavira, attaining the eternal bliss of
MUHARRAM community in India. Nanakpanthi
Nirvana. Diwali also commemorates
The 10th of Muharram (the day of Hindus and other followers of Guru
the return of Lord Rama alongwith Sita
Ashura-Ashoora) is observed as an Nanak’s philosophy also celebrated
and Lakshman, from his 14-year
long exile and vanquishing the demon important day by both Sunni and Shia this festival. This festival is celebrated
king Ravana. Muslims, however, for different reasons. on Kartik Purnima, which is a very
Based on the Hadith of Prophet auspicious day for Hindus.
FESTIVALS OF MUSLIMS Mohammed, Sunni Muslims celebrate
Ashura as the day when Prophet Moses
FESTIVALS OF CHRISTIANS
SHAB-E-BARAT
Literally, Shab-e-Barat means the (Moosa) fasted, because Allah saved the GOOD FRIDAY
Night of Salvation or the Night of Israelites from their enemy in Egypt. The Good Friday is also referred as
Freedom from the Fire of Hell. It occurs Shia Muslim’s observance of Ashura is Holy Friday, Black Friday, Great
in Mid-Shaban between the 14th and different altogether. Friday, Long Friday and Silent
15th day of Shaban, as per the Islamic They observe Ashura as the day of Friday. Good Friday is a fast day
Calendar. martyrdom of Hussein Ibn Ali, the created by the Roman Catholic
grandson of Prophet Mohammed at the Church in the 4th century AD (long
This night, known as Shab-e-Barat or after Jesus died). Its purpose is to
Laylat-ul-Baraa, is called Laylatul Battle of Karbala.
commemorate the crucifixion and
Nisf min Shaban in Arabic. The Shia Muslims, therefore, consider this a death of Jesus.
blessed night starts at sunset on the day of sorrow and observe it as such by
14th and ends at dawn on the 15th. The refraining from music, listening to EASTER
Muslims observe Mid-Shaban as a sorrowful poetic recitations, wearing It is one of the most popular religious
night of worship and salvation. Some mourning attire and refraining from all celebrations in the World. Easter isn’t
spend the whole night awake and some joyous events (e.g. weddings), that in a Christian or directly Biblical term,
worship half of it. anyway distract them from the but comes from a form of the name
sorrowful remembrance of that day. Astarte, a Chaldean (Babylonian)
ID-UL-FITR
Goddess known as the Queen of
It is one of the major holidays of Islam. ID-E-MILAD NABI Heaven.
It comes at the end of the holy month of The Prophet’s birthday or Milad un Nabi
Ramzan and celebrates the end of the as it is commonly known in Muslim CHRISTMAS
fasting. The holiday comes on the 1st culture, is celebrated in most of the It is a holiday shared and celebrated
day of the 10th month in the Islamic Muslim countries and in India as well. by many religions. The date of 25th
Lunar Calendar. Because the month of The day is celebrated to commemorate December comes from Rome and was
Ramzan is all about fasting, Id-ul-Fitr the birth of the Prophet Mohammed. a celebration of the Italic God, Saturn
is all about celebration. On Id-ul-Fitr, It is celebrated in the 3rd month of the and the rebirth of the Sun God.
they celebrate the end of Ramazan with Islamic calendar. While the Shias This was done long before the birth of
a sweet snack and then get ready for a
celebrate it on the 17th of the month, the Jesus. To many people, it is a favourite
day of celebration.
Sunnis celebrate on the 12th of the time of the year involving gift giving,
ID-UL-AZHA (BAKR-ID) month, according to the Islamic parties and feasting. Christmas is a
Id-ul-Azha takes place on the 10th of calendar. The date of this festival varies holiday that unifies almost all those
Zil-Hajj to commemorate the great in the Gregorian calendar. professing Christendom.
incident in our history as well as the
event of Hajj itself. Ofcourse Id has a
FESTIVALS OF SIKHS FESTIVALS OF PARSIS
greater meaning for those who actually BAISAKHI The Parsi community, small in
perform Hajj, as they visit all those The origin of Baisakhi festival began number, is confined by and large, to
Holy sites while residing in Makkah with the martyrdom of Guru Tegh the city of Mumbai and to some extent
and also give sacrifice there. And it is Bahadur, the 9th Sikh guru who was in Gujarat. The Parsis have many
that very sacrifice, that connects publically beheaded by Aurangzeb, the festivals but no pilgrimage sites as
Muslims all over the world. Muslim ruler, on 11th November, 1675. such.
ART AND CULTURE 1-335

According to the Parsis, there are 6 MOPIN-ADI FESTIVAL ANDHRA PRADESH


seasons in a year and a significant It is the main festival of Gallong BRAHMOTSAVAM
festival occurs in each. The Community and is normally celebrated It is a Hindu festival celebrated
Gahambars, as the festivities are called in order to get rid of natural calamities annually at Tirumala Venkateswara
were originally agricultural in nature, and diseases, effects of evil spirits and temple at Tirupati in Andhra Pradesh.
but as Zoroastrianism spread far and for good harvest, health, wealth and The festival continues for 9 days in the
wide, they took on a religious prosperity. The festival lasts for 5 days months of September and October.
significance. from the 8th of April, before sowing of
paddy. SAMMAKKA JATARA
PATETI
It is celebrated once in 2 years on a very
The New Year day of the older school of MURUNG FESTIVAL
large scale for 3 days before Magha in
the Parsis, Pateti falls somewhere in It is a festival of abundance and Warangal district of Andhra Pradesh.
September. Wearing new clothes, the richness of wealth. It is celebrated in Animals are sacrificed and vows are
Parsis offer prayers at the temples, the month of January every year for 7 redeemed, intoxicants are widely used.
give alms and arrange sumptuous days. The special offering to the deity is
feasts at their homes. jaggery which is collected in huge piles.
BUDDHA MAHOTSAVA
KHORDAD SAL The festival celebrated across the Those who fulfil vows offer jaggery
The birth anniversary of Zoroaster, state, showcases the ideals of equal to their weight and jaggery is
Khordad Sal is celebrated on the 6th Buddhism, ushering in the much distributed as prasadam.
day of the Parsi month, Farvardin, by needed values of peace, harmony and NAGULA CHAVITI
offering prayers at the temples. tolerance in today’s world. The festival This is observed on the 4th day of the
highlights include monastic dances, bright fortnight in the month of either
ZARTHOST NO DEESO traditional tribal music, song and Shravana or Kartika. This is a
This day is observed as the death dances, religious procession, food pre-historic celebration of Naga or
anniversary of the Prophet Zoroaster, festival, handloom and handicraft Serpent worship observed throughout
who is believed to have died in a temple exhibition, flower/ horticulture shows Andhra. Every village in Andhra has
while praying. The day is an occasion of and adventure sports like trekking, some or other Naga idol carved in stone
mourning and discourses are held on rafting and hot air ballooning, cultural or wood.
the life and works of the prophet. nights and other special events.
DECCAN FESTIVAL
JAMSHED NAVROZ OTHER FESTIVALS
The best feature of this festival are the
Some Parsis follow the Fasli Calendar Other festivals that are celebrated in arts, crafts, dances and music.
and their New Year commences with the state are Losar, Khan, Nyokum,
the Vernal Equinox. The New Year Gomkum Gompa, Boori Boot, Mopin, LUMBINI FESTIVAL
celebration dates back to the times of Aran etc. The festival is observed to
the legendary King of Persia, commemorate the religion of Buddhism.
Jamshed. It is said, that King ASSAM The event has been named after
Jamshed introduced solar reckoning BIHU Lumbini, which is the birthplace of Lord
into the Persian calendar and also It is the chief festival celebrated on Buddha.
determined the date when the Sun three occasions in Assam, namely
enters the constellation of Aries, as the Rongali Bihu, Bhogali Bihu or Kangali
TIRUPATI FESTIVAL
beginning of the year. This day came to Bihu. It is most popular festival of This 9 day festival is observed with
be known as Navroz or Jamshed Assam. grand celebration at the Tirumala
Navroz day. Venkateswara temple. Tirupati
RONGALI BIHU OR BHOGALI BIHU Brahmotsavam is celebrated in the
On this occasion, it was customary for It marks the advent of the cropping month of September and October.
the king to be weighed in gold and season and also ushers in the
silver and the money was then Assemese New Year. Bhogali Bihu or VISAKHA UTSAV
distributed to the poor. Magh Bihu is the harvest festival and This festival is the most exclusive event,
Kati Bihu or Kangali Bihu coming in which celebrates the traditions and
STATEWISE FAIRS AND autumn is a simple affair. culture in the form of arts, crafts, music
FESTIVALS OF INDIA DEVADDHVANI
and dance.

ARUNACHAL PRADESH It is held once a year at Kamakhya and UGADI FESTIVAL


SOLUNG-ADI is connected with Manasa or Maroi It marks the celebration of New Year,
puja. Manasa, the serpent Goddess, is as per the Hindi Lunar Calendar.
The Solung is the main festival and is a
worshipped by many people in several People celebrate the occasion by
manifestation of the Adis festival
districts of Assam by Hindu religion wearing new clothes and decorating
culture. Solung celebration continues
followers. their homes.
for 5 days.
1-336 GSP General Studies (Paper 1)

Ugadi Pachadi is special CHHATTISGARH SUI MATA FESTIVAL


chutney (condiment) that is BHORAMDEO FESTIVAL It is Chamba’s annual festival. This
made on this day and offered to festival is celebrated in the month of April
The architecturally brilliant Bhoramdeo
the deity. After the rituals, the for 4 days.
temples, built by king Ramachandra of the
chutney is given to everyone as a
Nag dynasty, in the Satpura hills on the OTHER FESTIVALS
Prasad. This festival generally
banks of river Sankari, form the backdrop
falls in the Hindu month of Other festivals that are celebrated in
for the annual Bhoramdeo festival.
Chaitra (March or April), on the Himachal Pradesh are Holi, Baisakhi,
first day of the bright half. POLA FESTIVAL Gugga, Dussehra, International
It is celebrated by worshipping bullocks. Himalayan festival, Haryali, Rakhi,
PONGAL FESTIVAL Navratra, Chrewal, Jagra, Sair, Judged,
Children play with idols of Nandi bull made
Each and every household in the Losar Shivratri, Halda, Nawala etc.
of clay and fitted with clay wheels. A bull
state observes this day festival
race is a major event of the festival. JAMMU AND KASHMIR
with merriment. Also known
as Harvest festival, Pongal TEEJA FESTIVAL PURMANDAL MELA (FEBRUARY-MARCH)
celebrates the beginning of This festival is celebrated by married Purmandal is 39 kms from Jammu city.
harvesting crops in the fields. women, in which they pray for the welfare On Shivratri, the town wears a festive
Bhogi festival, Surya Pongal, of their husbands. look and for 3 days, people continue,
Mattu Pongal and Kaanum celebrating the marriage of Lord Shiva to
Pongal are the names given to GUJARAT Goddess Parvati.
the celebrations of 4 days. MODHERA
It is an annual festival of Indian classical JHIRI MELA (OCTOBER-NOVEMBER)
RAYALASEEMA FOOD AND dances at Sun temple, where classical dance An annual fair is held in the name of Baba
DANCE FESTIVAL forms are presented in an atmosphere they Jitu, a simple and honest farmer who
One amongst the biggest were originally presented in. It is celebrated preferred to kill himself rather than
cultural festivals, Rayalaseema in January. submit to the unjust demands of the local
festival is celebrated to landlord to part with his crop. He killed
accentuate the art and cuisine of DHULETI himself in the village of Jhiri, Jammu.
the area as well as the state. The next day after Holi is Dhuleti or Dhuli
Another interesting fact about Padva festival. It is celebrated in March. JHARKHAND
the festival is that many famous SOHRAI
personalities come here to give HIMACHAL PRADESH It is associated with cattle and is
their performances. LOHRI celebrated during the festival of Diwali,
‘Rayalaseema food and dance At the end of the month of Pausha, the which falls in the month of November.
festival’ is held in the month of Lohri festival is celebrated with Lohadi. During the festival celebrations, the cattle
October every year. Lohri known as Lohkadiyan. The people are washed and worshipped. This is a time
welcome the singers and give them gifts of for great amusement and performances of
BIHAR food grains. On Lohri night, the boys sing cattle such as bullfights are organised to
CHHATTHA Harin (the deer) songs. multiply the fun and excitement.
It is a major festival in Bihar
GOTSI OR GOCHI SARHUL
and is celebrated a week after
This is the most popular festival in the This is the main festival of the tribal
Deepawali. Chhattha is the
valley, which is celebrated in the month of population of Jharkhand. The verbal
worship of the Sun God.
February in the houses of those, who have meaning of ‘Sarhul is worship of the Sal
RAMNAVAMI been blessed with a son in the past year. tree’. Sarhul can also be redefined as
It is a Hindu festival celebrated People gather in those houses and drink worship of nature in which local people
in all parts of the country. This chhang wine. worship Sita, the wife of Lord Rama as
is the auspicious day when Lord PHULAICH Dhartimata.
Rama was born. In the monsoon month of Bhadrapada KARNATAKA
NAVARATRI comes the festival of flower-watching
UGADI
(ukhyang) in the Kinnaur valley. This
It is one of the major festivals It is believed that the creator of the Hindu
festival is also known as Phulaich and it
connected with the autumnal pantheon, Lord Brahma, started the
commemorates the dead. The fair opens
equinox, beginning on the first work of creation on Chaitra Suddha
with animal sacrifices and soon the entire
and ending on the 10th day of Padhyami or Ugadi day. Ugadi is
village collects on a hill top, and looks for
Navaratri. After the idol of celebrated with festival fervour in
the Ladra flower. People serve rice wine
Durga has been worshipped for Maharashtra, Karnataka and Andhra
and food to the dear departed ones on a
nine preceding days, it is taken Pradesh.
mound of bricks.
to the river and casted into it.
ART AND CULTURE 1-337

GOWRI FESTIVAL MADHYA PRADESH ODISHA


This festival is celebrated a day GHAILA AND BIDRI GHAILA NUAKHAI
before Ganesh Chaturthi. It is a This is the Earthen pot ceremony of the Gonds The festival is intended for eating
significant festival in parts of and is identical to the Akshya Tritiya of the new rice of the year, it is observed as
Karnataka, Andhra Pradesh and Hindus. It is performed in the month of a general festival.
Tamil Nadu. Jyaistha (May-June). This festival corresponds On this occasion, the new rice is
KERALA to the feast of transplantation of paddy cooked with milk and sugar (Kshiri)
seedlings celebrated by the Mundas and most and then offered as Bhog to Goddess
ONAM
other settled agricultural tribes of Lakshmi.
It is a special festival, celebrated
Chhotanagpur.
at the time of cropping. Festival of BADA OSHA
Mahashivaratri is celebrated at KAJRI NAVAMI It is essentially a major occasion of
the bank of the Periyar river. It is On the 9th day of the waxing moon fortnight of fasting. In coastal Odisha,
like Kumbha. Onam, associated the month of Shravana, falls the Kajri festival. thousands of people gather at
with the legend of Mahabali is a Only those women, who are blessed with sons, Dhabaleswar, temple of Lord
state festival. It comes in the observe this festival. Their worship ritual Mahadeva, in rocky islands in the
month of Chingam, according to continues till the Full Moon day of the same midst of the river Mahanadi near
Malayalam calendar. month. Cuttack for fulfilment of boons.
VISHNU It is celebrated on the 14th of
MIZORAM Kartika (September-October). This
It is a festival held in the state of MINI KUT
Kerala (and in the adjoining areas fast is also observed by men.
It is celebrated with great gaiety and
of Tamil Nadu) in India around PUNJAB
merriment expressed through singing, dancing,
the 1st day in the Malayalam
feasting and drinking of homemade rice beerzu. CHAPPAR MELA
month of Medam (March-April).
It is dedicated to the memory of dead relatives. The popular Chappar mela is
MANIPUR organised every year at Chappar,
PAWL KUT Ludhiana. People mainly worship
YAOSANG It is a harvest festival celebrated during snake embodiment of Guga in this
It is celebrated during the months December to January. Mothers with their fair. The fair falls on the 4th day of
of February, March on a Full children sit on memorial platform and feed one the month of Bhadas every year.
Moon night. During this festival, another. This custom is also performed during Other festivals that are celebrated
the boys and girls come together Chapchar Kut and is known as in the state are: Lohri, Baisakhi,
and dance on the Thobal Chongba. Chawnghnawt. Drinking of zu is a part of this Hola Mohalla etc.
KANG festival.
PONDICHERRY
This is the Rath Yatra of Manipur; NAGALAND
it is quite popular among the MASCARADE FESTIVAL
NAZU FESTIVAL Mascarade or Mask festival is
Manipuri Hindus. This festival is It is one of the most merry-making festivals of
celebrated for 10 years. celebrated sometime during
the Pochury tribe of Nagaland. This gala event
March-April in Pondicherry. It
CHUMPHA is celebrated for 10 days in the month of
reflects one more fact of the French
(FESTIVAL OF TANGKHUL NAGAS) February, prior to the sowing of seeds for the
sense of humour.
It is celebrated for 7 days in the year. Songs and dances are performed by the
Pochury tribe during this occasion. With unique masks and fancy
month of December. The costumes, revelers go round the
Chumpha festival is a great TULUNI FESTIVAL streets in joyous merriment
festival of the Tanghkhul Nagas. The most important festival of the Sumi Nagas indulging in a kind of gimmick
The festival is held after harvest. in Nagaland, it is celebrated during the 2nd dance, to the accompaniment of
NINGOL CHAKOUBA (THE week of July. Tuluni is marked with feasts as beating accordion and trumpets
SOCIAL FESTIVAL OF MANIPURIS) the occasion occurs in the fruitful season of the during the festival.
It is a remarkable social festival of year. This festival is celebrated with great
pomp by the Sumi tribe of Nagaland.
MASI MAGAM FESTIVAL
the meiteis. Married women of the It is celebrated on the Full Moon day
family, who were married to YEMSHE FESTIVAL of the Magam asterism in the Tamil
distant places, come to the The Pochury tribe of Nagaland celebrates month of Masi (February-March).
parental house along with their arrival of the new harvest during Yemshe Thousands of people in good faith go
children and enjoy sumptuous festival. All the Pochuries ceremonalise this to the seashore in the early morning
feasts. festival with great pomp and show expecting a to have a holy dip in the sea for
good harvest and no one is allowed to harvest, ablution of their sins.
until the whole festival is over.
1-338 GSP General Studies (Paper 1)

Deities from as many as 50 temples KHARCHI PUJA UTTAR PRADESH


in Pondicherry (Puducherry) and This is one of the most important festivals BHARAT MILAP
from the outlying areas of Tamil of Tripura and is basically celebrated by It is celebrated during the months of
Nadu are brought in ceremonial the tribal and non-tribal groups. The term October/ November and is carried out
procession to the seashore with khya signifies Earth, so the festival of at Nati Imli in Varanasi on the day
accompaniment of Nadhaswaram Kharchi Puja means the worship of the following Vijayadashmi or Dussehra. It
near Kuruchi-Kuppam in Earth, which sustains life with all her pertains to the episode of the return of
Pondicherry, municipal area for the resources. It is observed in the month of
Lord Rama to Ayodhya after 14 years of
symbolic immersion. July in the region of Agartala.
exile and his reunion with his brother
SIKKIM TAMIL NADU Bharat.
SAGA DAWA PANGUNI UTTIRAM
WEST BENGAL
It is the holiest of the holy Buddhist The marriage of the God and Goddess is
JALPESH MELA
festivals to commemorate different the annual event in all temples in Tamil
Nadu in the month of Panguni. For On the occasion of Shivaratri
years of his life; Lord Buddha took
villagers, this is a very important festival (February-March), a month long fair is
his birth, achieved enlightenment
and as it comes in the off season, they held at Jalpesh near Main Guri in the
and passed away attaining Nirvana.
participate in it fully and give offerings of district of Jalpaiguri. The fair centres
This three fold auspicious day falls round the age old Shiva temple
paddy to the temples.
on the full moon day of the 4th dedicated to Lord Jalpeswara.
month of Buddhist Lunar Calendar PONGAL
around the end of May and early It is celebrated on the 1st day of the Tamil POUSH MELA
June. month Thai. The pot in which Pongal is It comes between the 7th and 9th
cooked is called Pongal Panai. The word of Poush. Poush Mela is held at
PHANG LHABSOL Pongal means rice cooked in milk and Shantiniketan in the district of
This festival is unique to Sikkim in jaggery. Pongal is a celebration of spring Birbhum.
which the snowy range of on the occasion of the ascent of the Sun to
Khanchenjunga is worshipped for its the North. It is the celebration of the SOME FAMOUS FAIRS
unifying powers. harvest season. It is celebrated for 4 days KUMBH MELA
This festival also marks the signing in Tamil Nadu. It is unique in the respect that it does
of the treaty of blood brotherhood NADAVAVI not exhibit the features associated with
between the Lepchas and Bhutias by a traditional Indian fair. It is basically
Vavi means a well. This is a local festival
Khye Bumsa and Tetong Tek, a religious congregation which is held
of great importance held on the Full Moon
when the local deities were invoked day in April-May, owing its origin to one once every 12 years (Maha Kumbha) at
to witness the occasion. Kodi Kannikadanam Thatchachariar and one of the 4 holy places (Allahabad,
to the money he collected for Varadharaja Ujjain, Nasik, Haridwar) in turn. An
LOSOONG ardha or half Kumbha occurs every 6
Perumal temple.
It marks the end of the harvest years.
season and also the end of the 10th UTTARAKHAND According to mythology, when the
month of the Tibetan Year. Taking BHITAULI Devas or Gods and the Asuras or
eve from a good harvest and praying
This festival of Uttarakhand is feted in demons together churned the waters of
for even better prospects for the next
the month of Chaitra, according to the the primeval ocean, many priceless
crop, the festival is marked by
Hindu calendar and it falls on the very 1st things floated up from the ocean.
Chaam dancing at the monasteries
day of Shravana and is celebrated with Among these was a pot (Kumbha) of
at palace (Tsu-La-Khang), Phodong
much pomp and show all over the state. It Amrit, the nectar of immortality.
and Rumtek. is a grand festival of sharing gifts from During the struggle for the possession
TRIPURA the brothers to their sisters. The religious
of the amrit between the demons
rites and rituals during this festival are
KHATARUA and the Gods, some drops of the
celebrated with much veneration.
This is essentially one of the most precious nectar got spilt. These fell at
special festivals of Tripura and is CHHIPLA JAAT 12 places including another world
observed by the pastoral agricultural This is one of the main festivals of called Patala.
society with much fun and vigour in Chhiplakot situated in the heartland of Four of these 12 places, namely
the month of Ashwin, which falls in Kali and Gori rivers to express different Haridwar, Ujjain, Allahabad and
mid September. This grand occasion aspects of human faith. The religious rites Nasik, are in India and a great fair is
calls for unrestrained merriment and rituals include barefoot journey, bath, held at each of these places in a 12
and on this special day people light worship, traditional songs and dances, years cycle. Ujjain is also visited by
collective food and the possession of the many pilgrims during eclipses when a
bonfires around which children
body of Bonia, by the folk God are the holy bath is considered meritorious.
dance displaying multi-hued flags.
essential parts of this festival.
ART AND CULTURE 1-339

PUSHKAR MELA (RAJASTHAN)


It is held on the day of Kartik Purnima (in
October-November). Devotees gather around Pushkar
lake and take a ceremonious bath in it. Pushkar, one of
CRAFTS
the holy pilgrimage places of India, is said to possess the Indian crafts, quite diverse are rich in history and
only temple where Brahma is still worshipped. religion and have been embedded as a culture and
Pushkar fair, characterised among other things by its
cattle market, has now become very well-known to the
tradition with in the rural communities. Not only have
tourists, both domestic as well as foreign. Traditionally, religious beliefs and continuing heritage lead to their
a religious occasion, it is accompanied by an exchange of production, but also the foreign and domestic trade of
camels.
these items to a great extent, contributed in their
Saliana (Palampur) among the non-religious fairs,
Saliana, (Palampur) is noted for its wrestling matches, increased production and also in the evolution of
while at Sonepur mela (in Bihar), sale of elephants adds different craft forms in the country.
extra colour to the occasion.
GARH MUKTESHWAR FAIR CRAFTS OF INDIA
This fair is held every year at Garh Mukteshwar in Uttar
There are numerous examples of handicrafts amongst what has
Pradesh where besides ritual bathing in the Ganges,
been procured from the Indus Valley Civilisation. The tradition
brisk economic activities also take place.
of crafts in India has grown around religious values, needs of the
URS common people and also the needs of the ruling elites. In addition
This is the birth or death anniversary celebrations at a to this foreign and domestic trade have also played an important
dargah. Urs are held annually at the dargah of famous role in the evolution of different craft forms in India. The craft
sufi saints. At the dargah of the sufi saint Qutubuddin traditions of India have withstood the depredation of time and
Bakhtiar Kaki, another unique festival, sair-i several foreign invasions and continue to flourish till date. It is
gulfaroshan or phoolwalon ki sair is held every year. mainly due to the open-mindedness of the Indian handicraftsmen
to accept and assimilate new ideas.
This unique festival of flowers, known as the procession
of flowers and flower sellers, dates back to the Mughal
period. Hindus, Muslims and others participate
CRAFTS IN VARIOUS PHASES
whole-heartedly in it. INDUS VALLEY CIVILISATION
During the period of Indus Valley Civilisation we find a rich craft
SURAJKUND HANDICRAFT MELA
tradition and a high degree of technical excellence in the field of
Surajkund is a small ancient kund (pond) in Haryana on pottery, sculpture (metal, stone and terracotta), jewellery,
the outskirts of Delhi. This place is the venue for the weaving etc. The Harappan craftsmen not only catered to all the
annual handicrafts fair. local needs but traded with the outside world.
MUKTSAR FAIR VEDIC AGE
It is one of the largest Sikh fairs held in Punjab and is In the Vedic age (BC 1500), we find numerous references in the
held on the Makar Sankranti day. The festival is in Vedas of artisans involved in pottery-making, weaving, woodcraft
commemoration of a battle fought in 1705-1706 by Guru etc. The Rigveda refers to a variety of pottery made from clay,
Gobind Singh, against the pursuing imperial forces wood and metal. There is a reference to weavers and weaving.
which overtook him here and cut his followers to pieces. During Vedic age and after types of crafts increased manifold
MINJAR FAIR which include metal, wood, ivory, clay, terracotta, clothes, crafts.
It is the most popular fair of Chamba, which is attended MAURYAN PERIOD
by a large number of people from every nook and corner During the Mauryan age, we find great developments in the field
of the district. This mela is held on the second Sunday of of sculpture. In this period, more than 84000 stupas are said to be
the Shravana month or in the month of July-August. built in India, including the famous Sanchi stupa, which has
Minjar fair has been declared as one of the state fairs of beautiful stone-carving and relief work done on it. Numerous
Himachal Pradesh. sculptures from Bharhut, Mathura, Amravati, Vaishali, Sanchi etc
The fair is announced by distribution of Minjar, a silk show female figures adorned with a display of jewellery, which
tassel worn on some parts of the dress by men and continues to inspire contemporary jewellery making. The period
women alike. This tassel symbolises the shoots of paddy between 1st century BC and 1st century AD was a period of
and maize, which appear around this time of the year. political confusion as a result of foreign invasions. The impact of
This week-long fair begins, when the Minjar flag is this turmoil is visible in the amazing Buddhist sculptures from
hoisted is historical Chaugan. Taxila, Begram, Bamiyan, Swat valley etc.
1-340 GSP General Studies (Paper 1)

KUSHANA PERIOD Folk embroideries done by the village CANE HANDICRAFTS


During the Kushana period, jewellery, women folk of India are a good Cane products, a famous form of Indian
sculpture, textile making, leather example. There are craftsmen, who handicraft, include utilitarian objects
products, metal working etc were the are specialised in creating textiles or like trays, baskets, stylish furniture
main handicrafts that assimilated jewellery, which meet the criteria set etc. The district of Tamil Nadu is
foreign influences and used them in up by a particular group of people, famous for Cane handicrafts in India.
accordance with the Indian setting. The usually a particular caste or
Gupta age saw rapid advancement in community. They have their own BONE AND HORN HANDICRAFTS
the field of handicrafts and art forms. distinctive designs and styles. Originating in the state of Odisha, the
The murals at Ajanta and Ellora bear bone and horn handicrafts are famous
2. RELIGIOUS CRAFTS
testimony to it. for creating birds or animal figures,
They develop around religious centers
which seem alive e.g. a bird seems like
MEDIEVAL PERIOD and themes. These craft items are
connected with religious institutions twittering.
In the Medieval period, the
handicraftsmen flourished in the field of and relevant ceremonies. Various Besides this, goods like pen stands,
pottery, weaving, wood carving, metal religious places in India specialise in ornaments, cigarette case, table lamps,
working, jewellery etc. The contribution particular craft items. e.g. Varanasi pepper and salt sets, chess sets, napkin
of the Cholas and the Vijayanagara and Kanchipuram in Tamil Nadu rings, laughing buddha etc are
empire in the field of bronze sculpture, specialise in weaving clothes for prepared in Odisha, Karnataka, Kerala
silk weaving, jewellery, temple carving religious ceremonies, particularly silk and Uttar Pradesh.
etc is simply unparalleled. The Mughal materials. Similarly, Puri in Odisha,
period was the Golden period in the which is a big pilgrim center, is CLAY HANDICRAFTS
history of Indian art, craft and culture. connected with crafts like patacharita With its origination during the Indus
The Mughals brought with them a rich a painting on cloth and wood and stone Valley Civilisation, clay craft or pottery
heritage. The Mughals introduced carvings. is said to be one of the most primitive
methods like inlay work, glass form of handicrats in India. People
engraving, carpet weaving, brocades,
3. COMMERCIAL CRAFTS engaged in pottery are called
enameling etc. Specialised craftsmen of a particular Kumhaars. Besides its world famous
group who are specialised in a terracotta form, pottery has got
BRITISH PERIOD particular skill and who can different forms like Red Ware, Grey
England flooded the Indian market with completely master the craft engage in Ware and Black Ware. Uttar Pradesh
its cheap machine-made items, which commercial crafts. They even have is known for its painted Black Wares.
ousted the homemade crafts. A number sub-groups which work for particular
of craftsmen were turned destitutes groups and their tools and techniques BELL METAL HANDICRAFTS
overnight. Those who continued with may vary. The weavers, the dyers, the The hard form of bronze, which is
their craft had to compete with the printers, the goldsmiths and the usually used to make bells, is referred
machine-made goods. Gandhiji’s carpenters are some of the commercial as bell metal. This kind of hard alloy
Swadeshi Movement focussed on the craftsmen. is used to make crafts like vermilion
plight of the Indian craftsmen and on boxes, bowls, candle stands, donari
the need for maintaining the ancient KINDS OF HANDICRAFTS IN (pendants) and many more. This bell
craft traditions. INDIA metal crafts are mostly prevalent in
Madhya Pradesh, Bihar, Assam and
AFTER INDEPENDENCE Varied kinds of handicrafts are
manufactured in India, which have Manipur.
After Independence, the Handicrafts
Board was set up to look into the plight gained popularity even in BRASS HANDICRAFTS
of the dying crafts. Slowly demand grew International markets. The most
Durability of brass items added on to
for these items both at home and also known forms of handicrafts in India,
the fame of brassware. Items made of
abroad. Recent export figures show that are discussed below
brass like crawling Krishna, Lord
India is lagging behind in many Ganesha’s figure in different postures,
handicraft commodities except in the
BAMBOO HANDICRAFTS
Being a producer of Bamboo, vases, table tops, perforated lamps,
case of gems and jewellery items.
handicrafts made from Bamboo are ornament boxes, hukkas, toys, wine
glasses, plates, fruit bowls and many
CLASSIFICATION one of the eco-friendly crafts in India.
more are extensively used in many
The varied items made from bamboo
OF CRAFTS are baskets, dolls, toys, furniture, Indian houses till now.
There are three main classifications of mats, wall-hangings, umberella These artisans are famously known as
crafts which are as follows handles, crossbows, khorahi, kula, Kansaris. The manufacturing of
dukula, kathi, jewellery boxes and brassware is mainly done in
1. FOLK CRAFTS many more. Bamboo handicrafts are Rajasthan.
People, for their personal use or for a mostly made in West Bengal, Assam
limited client base, create folk crafts. and Tripura.
ART AND CULTURE 1-341

DHOKRA HANDICRAFTS ROCK HANDICRAFTS


Dhokra, the oldest form of handicraft is known Prevalence of rock carving, one of the primitive rock art can be seen in the
for its traditional simplicity. This tribal states of Rajasthan, Odisha and Nagpur. Rajasthan Jaipur and Madhya
handicraft originated in Madhya Pradesh. The Pradesh are famous for marble stone carvings. Green-coloured stone art is
other states involved in this are West Bengal, the speciality of Madhya Pradesh, whereas, Patharkatti is the unique rock
Bihar and Odisha. craft of Gaya. Age-old temples of Odisha are the world famous examples of
rock-craft in India. Numerous utensils, decorative pieces, stone jewellery and
Dhokra is famous for its unique items portraying
statues are made from rocks.
folk characters. Dhokra jewellery, candle stands,
pen stands, ash trays and varied kinds of SHELL HANDICRAFTS
showpieces are available at every handicraft From time immemorial, shell handicrafts are one of the demandable crafts in
shops. India. Shell handicraft can be made out of three types of shells like conch
shell, tortoise shell and sea shell. Different kinds of goods like bangles,
JUTE HANDICRAFTS
forks, decorative bowls, lockets, spoons, buttons, curtains, chandeliers, mirror
Jute craftsmen have created a worldwide niche in frames, table mats etc are the products of shell crafting. Generally, the places
the field of jute handicrafts. The huge range of located on the sea shore like Gulf of Mannar, Goa, Odisha.
jute crafts includes bags, office stationeries,
bangles and other jewellery footwear, wall- WEAVING OR EMBROIDERY HANDICRAFTS
hangings and many more. West Bengal, Assam Weaving mainly refers to the process of cloth production by two thread sets
and Bihar, being the leading jute producers, lead known as Weft and Warp, crossed with each other. This traditional form of
the Jute handicrafts market in India. handicraft is mostly found in the states of Gujarat, Madhya Pradesh and
Rajasthan.
PAPER HANDICRAFTS
Vibrant coloured papers are combined together to
form varied crafts like kites, masks, decorative
flowers, lamp shades, puppets, hand-fans etc.
Papier mache developed in the Mughal era is even
a famous form of paper handicraft in India.
This craft industry is mainly located in Delhi,
INDIAN CINEMA
Rajgir, Patna, Gaya, Avadh, Ahmedabad and
Allahabad.
The Indian film industry is considered as the biggest film
industry in the world. So much so that, India’s film industry
Geographical Indications (GI) has been popularly dubbed as Bollywood. Although the main
Geographical indications of goods are defined as that language in which the movies are produced in Hindi, almost
aspect of industrial property which refer to the
Geographical Indication referring to a country or to a
every regional language of India has its own mini film industry
place situated therein as being the country or place catering for their respective areas.
of origin of that product. Typically, such a name
conveys an assurance of quality and distinctiveness
which is essentially attributable to the fact of its
origin in that defined geographical locality, region or
HISTORY OF INDIAN CINEMA
country. When Lumiere brothers invented cinema in the last decade of the 19th
Under Articles 1 (2) and 10 of the Paris Convention century, they did not quite realise the fact that their invention would, in
for the Protection of Industrial Property, years come to entertain millions across the world in an unprecedented
Geographical Indications are covered as an element manner. Only 7 months after its inauguration (premiere show) in France,
of Intellectual Property Rights (IPRs). They are also Lumiere brothers’ films were shown in Bombay for the first time on 7th July,
covered under Articles 22 to 24 of the Trade Related 1896. In 1899, Harishchandra Sakharam Bhatavdekar made a film on a
Aspects of Intellectual Property Rights (TRIPS) wrestling match in Bombay. The honour of making the first feature film goes
Agreement, which was part of the Agreements to Dadasaheb Phalke (Dhundiraj Govind), who made the first silent film
concluding the Uruguay Round of General Raja Harishchandra in 1913. The history of Indian cinema can be broadly
Agreement on Tariffs and Trade (GATT) negotiations. divided into three phases
Some Indian Products under GI
Darjeeling Tea (West Bengal); Chanderi Saree (Guna, 1. THE ERA OF SILENT FILMS (1913-31)
Madhya Pradesh); Kotpad Handloom Fabric (Koraput, A kaleidoscopic view of the History of India includes the pioneering efforts
Odisha); Kota Doria (Kota, Rajasthan); Kanchipuram of Save Dada (Harishchandra Sakharam Bhatavdekar). He made two
Silk (Tamil Nadu); Mysore Agarbathi (Mysore, short films as early as 1897. The first short films in India were directed by
Karnataka); Salem Fabric (Tamil Nadu); Solapur Terry Hiralal Sen, starting with Flower of Persia (1898). In 1900, the entire
Towel (Maharashtra); Kangra Tea (Himachal Indian entertainment sector underwent huge changes and the coming of
Pradesh); Coorg Orange (Karnataka); Mysore Betel Dadasaheb Phalke took Indian cinema to new heights. Thus, the
Leaf (Karnataka); Banaras Brocades and Sarees (Uttar
pathbreaking film of the silent era, Raja Harishchandra, was released in
Pradesh); Tirupathi Laddu (Andhra Pradesh) etc.
1913. During this time and during the era of the talkies, the main sources for
the Indian films were the mythological texts.
1-342 GSP General Studies (Paper 1)

Interesting Facts about Indian Cinema entertainment and instead started Phir Milenge, Diksha, Taare Zameen
India’s First Alam Ara (Light of the Universe) was serving as potent instruments to educate Par, Peepli Live etc are intended to
Talking released by Ardeshir Irani of Imperial the masses as well. educate the masses. Films such as
Film Movie tone. 14th March, 1931 was a Sarkar, Rajneeti, Page 3 and Fashion
historic day for Indian cinema. The film
was based on a successful Passi play
3. POST-INDEPENDENCE reflect the political and social scenarios of
which was written by Joseph David. FILMS (1947) (TILL TODAY) the contemporary society. As years fly
Film with Indra Sabha, with 71 songs is the film away, Indian cinema betters itself with
The Golden period in the history of more number of films, making it to the
the Most with most number of songs. The film
Number of was made in 1932 by Madan Theatres Indian cinema is attributed to the 1950s. golden pages of its history.
Songs and the director of the film was Guru Dutt, Mehboob Khan, Raj Kapoor,
JJ Madan. Balraj Sahni, Nargis, Bimal Roy, Meena VARIOUS TYPES
Longest LOC Kargil, at 4 hrs 25 min, is the Kumari, Madhubala, Dilip Kumar graced OF INDIAN CINEMA
Indian longest Indian movie made so far. The the screens. In South India, esteemed
Movie story is of the Indian soldiers fighting in
actors like Rajkumar, Gemini Ganesan, PARALLEL CINEMA
Kargil. Mera Naam Joker at 4 hrs 14
NT Rama Rao and several other actors The Parallel or New Cinema movement in
min is a close second.
and actresses entertained the audiences. India began in the regional cinema first,
First Colour Kisan Kanya is the first colour film in
Film in India. It was a 1937 Hindi feature film towards the end of the 60s, with the
In Bombay, while the magic of Guru
India which was directed by Moti B Gidvani production of Marinal Sen’s ‘Bhuvan
Dutt’s and Bimal Roy’s were
and produced by Ardeshir Irani of Shome’ (1996) and Uski Roti, both
imperial pictures. preponderant, Indian cinema moved one
financed by the Film Finance
First Indian Bhanu Athaiya was the first Indian to
step further with the release of K Asif’s
Corporation (FFC), initiating the new
to get an get an Oscar. She won the award for Mughal-e-Azam in 1960. A trail of
wave in Indian cinema. The New Indian
Oscar the best costume designer for Richard romantic movies followed all over India.
Attenborough’s film Gandhi in 1982. cinema was the cinema of social
While the Indian commercial cinema
Longest The song Ab Tumhare Hawale Watan significance and artistic sincerity,
enjoyed popularity amidst the movie-
Hindi Film Saathiyo in the film by the same name presenting a modern, humanist
goers, Indian Art cinema (also known as
Song is the longest Hindi film song. The perspective in contrast to the fantasy
length of the song is 20 minutes and Parallel cinema or New Wave
world of the popular cinema.
the song is featured in three cinema) did not go unnoticed. Adoor
instalments in the film. Gopalakrishnan, Ritwik Ghatak, This phenomenon was very important
Aravindan, Satyajit Ray, Mrinal Sen, because it gave rise to a cinema
Shyam Benegal, Sai Paranjpe, Shaji characterised by significant themes and
2. PRE-INDEPENDENCE Karun and several other art film directors innovative treatment; essentially a
TALKIES (1931-47) were making movies that took India to cinema that sought truth, did not obey
International fame and glory. convention and did not become
The rapid growth of the Indian cinema
subservient to popular notions of what
led to the end of the silent era and By the 1970s, Indian cinema enjoyed the
was good and palatable.
ushered in the era of the talkies. The histrionics of superstars like Rajesh
latter introduced the Indian cinema in a Khanna, Sanjeev Kumar, Waheeda INDIAN COMMERCIAL CINEMA
completely new way to the audiences. Rehman, Asha Parekh, Tanuja and Commercial cinema is the most popular
Now, one could hear the actors and others.At one hand, the Hindi cinema form of cinema in India. Ever since its
actresses talk, laugh, sing and cry. was growing leaps and bounds and on the inception, the Commercial Indian movies
Initially films were primarily made in other, the regional films were making have seen a huge following.
Hindi, Tamil, Bengali and Telugu and their presence felt too.
Commercial or popular cinema is made
proved to be phenomenal successes. The A number of well-established Hindi not only in Hindi, but also in many other
1930s and the 1940s witnessed the rise filmstars, who became a part of the star regional languages of East and South
of film personalities, such as Debaki system in India, actually began their India. Commercial films, in whatever
Bose, Chetan Anand, SS Vasan, Nitin career with the Indian regional films. languages they are made, tend to be quite
Bose and others. The 1980s saw the rise of several woman long (approx three hours), with an
Their contributions helped the Indian directors, such as Aparna Sen, Prema interval. Another important feature of
cinema to grow further. By this time Karnath, Meera Nair and others. The Commercial Cinema in India is music.
apart from Bombay (Mumbai), the film regional films in Malayalam, Kannada,
industry shaped up well in the down Telugu, Bengali and other languages REGIONAL CINEMA IN INDIA
south as well. The Tamil, Telugu and produced a number of romantic films. India is home to one of the largest film
Kannada film industries were making Renowned film personalities like industries in the world. Every year,
indigenous films as well. By the late Balachander IV, Sasi, Balu Mahendra, thousands of movies are produced in
1940s, films in India were made in Mani Ratnam, Ram Gopal Verma and India. Indian film industry comprises of
various languages, but the religious others made their marks. Stars like Hindi films, Regional movies and Art
influence was predominant. With the Shahrukh Khan, Rajnikant, Madhuri cinema. The Indian film industry is
struggle for independence, the entire Dixit, Aamir Khan, Chiranjeevi, Juhi supported mainly by a vast film going
scenario altered. Indian cinema now saw Chawla, Hrithik Roshan and others Indian public, though Indian films have
films based on the then contemporary explored all possible techniques to enrich been gaining increasing popularity in the
social issues. Movies no longer were Indian cinema with their performances. rest of the world, especially in countries
limited to the periphery of Films such as Gandhi, Terrorist, Amu, with a large number of emigrant Indians.
ART AND CULTURE 1-343

ART AND CULTURE


PRACTICE QUESTIONS

SET 1 SECTIONWISE/TOPICWISE QUESTIONS


1. The term Apabhramsa was used in 6. There are only two known examples of Which of the statement(s) given above
Medieval Sanskrit texts to denote cave paintings of the Gupta period in are correct?
(a) Outcastes among the Rajputs Ancient India. One of these is paintings (a) Only 1 (b) Only 2
(b) Deviation from Vedic Rituals of Ajanta caves. Where is the other (c) Both 1 and 2 (d) Neither 1 nor 2
(c) Early forms of some of the modern surviving example of Gupta paintings?
(a) Bagh Caves (b) Ellora Caves
11. Which one of the following temples
Indian languages
figured with the Institution of
(d) Non-Sanskrit verse metres (c) Lomas Rishi Caves
Devdasis?
(d) Nasik Caves
2. Which of the following texts of Ancient (a) Jagannath temple, Puri
India allows divorce to a wife deserted 7. Consider the following statements (b) Pasupathinath temple, Kathmandu
by her husband? about Kathakali? (c) Kandariya Mahadeva temple, Khajuraho
(a) Kamasutra 1. It is a dance drama of Kerala. (d) Chausath Yogini temple, Bhedaghat
(b) Manavadharam Shastra 2. Actors of Kathakali depict characters
12. Consider the following statements.
(c) Sukra Nitisara from the epics Ramayana and
Mahabharata and from the Puranas 1. Harikeli is a Sanskrit drama written by
(d) Arthashastra
(ancient scriptures). Chauhan ruler Vigrahraja.
3. Which of the following pairs is 2. Adhai din ka Jhopada which is a
3. Ritual traditions like Theyyams
correctly matched? Mudiyetlu and the Martial Arts of Kerala mosque in Ajmer contains lines of
Folk Dance State played a major role in shaping the Harikeli on its walls.
(a) Naqqual : Bihar dance into its present form. Which of the statement(s) given above
(b) Tamasha : Odisha is/are correct?
(c) Ankia Nat : Assam
Which of the statements given above are
correct? (a) Only 1
(d) Baha : Punjab
(a) 1 and 2 (b) 2 and 3 (b) Only 2
4. Consider the following statements (c) Both 1 and 2
(c) 1 and 3 (d) All of these
about Kathak, the North Indian classical (d) Neither 1 nor 2
dance. 8. The Mathura School of Art was
1. The word ‘Kathak’ derived from ‘Katha’ influenced by. 13. Which of the following statements is
literally means story teller. (a) Amaravati School of Art true about Tamasha?
2. Kathak originated in the North, but (b) Roman School of Art (a) Tamasha is a traditional folk theatre

ART AND CULTURE Practice Questions


Persian and Muslim influences later (c) Gandhara School of Art form of Maharashtra
altered the dance from a temple ritual to (d) Bodhgaya School of Art (b) It has evolved from the folk forms such
a courtly entitlement. as Gondhal, Jagran and Kirtan
9. Consider the following statements
Which of the statement(s) given above about Indian Classical Dances. (c) In this theatre form the female actress
is/are correct? is the chief exponent if dance
1. All the Classical Dances of India have
(a) Only 1 (b) Only 2 their origin in Bharata’s Natyashastra. movements in the pali
(c) Both 1 and 2 (d) Neither 1 nor 2 (d) She is known as Murki
2. They have three basic aspects Nritta,
5. Which one of the following statements Nrittya and Natya. 14. Which one of the following pairs of
is incorrect about Bharatnatyam? Which of the statement(s) given above composers in different languages and
(a) Bharatnatyam is one of the most is/are correct? their works on the Mahabharata theme
popular Indian dances of Tamil Nadu (a) Only 1 (b) Only 2 is correctly matched?
(b) The name is derived from the word (c) Both 1 and 2 (d) Neither 1 nor 2 (a) Sarladasa : Bengali
‘Bharatha’ and is associated with the (b) Kasirama : Oriya
Natyashastra 10. Consider the following statements (c) Tikkana : Marathi
(c) It skilfully embodies the three primary regarding Sanchi stupas (d) Pampa : Kannada
ingredients of dancing 1. Sanchi stupa is located in Sanchi,
Madhya Pradesh, 14 km from Vidisha. 15. Consider the following statements.
(d) The technique of Bharatnatyam
consists of 54 principles 8 of 2. It has three stupas all the gateway 1. Maach is the traditional theatre form of
coordinated hand, foot, face and body around them but the most famous is the Madhya Pradesh.
movements great stupa which was originally made 2. In this theatre form, dialogues are given
of brick in Ashoka’s time. prominence in between the songs.
1-344 GSP General Studies (Paper 1)

Which of the statement(s) given above Codes 26. Consider the following statements.
is/are correct? A B C D 1. The Elephanta cave is a
(a) Only 1 (b) Only 2 (a) 4 2 3 1 conglomeration of seven caves, out of
(b) 3 1 4 2 which the most important is the
(c) Both 1 and 2 (d) Neither 1 nor 2
(c) 4 1 3 2 Mahesha Murti cave.
16. Which of the following pairs of folk (d) 3 2 4 1 2. The cave complex has been given
dance form and states is not correctly the status of World Heritage by UNESCO.
matched? 21. Which of following was one of the main
effect of Vedic culture in Indian Which of the statement(s) given above
(a) Korku : Maharashtra is/are correct?
(b) Jhummar : Haryana history?
(a) Progress of Metaphysics (a) Only 1 (b) Only 2
(c) Thali : Himachal Pradesh
(d) Mukna : Manipur (b) Development of Sanskrit (c) Both 1 and 2 (d) Neither 1 nor 2

17. With reference to the Bagh Paintings, (c) Solidification of race 27. Consider the following statements
consider the following statements. (d) None of the above about Ajanta paintings.
1. Bagh Painting is on the same line of 1. These are frescoes paintings painted
22. Consider the following with reference to
Ajanta. on the rocks of its caves.
Jaina-miniature paintings (Apbhramsa
2. There are 9 caves at Bagh, the 4th cave School). 2. These belong to the period of
Rang Mahal has got the maximum 3rd century BC to 7th century BC.
1. Bulging fish-shaped eyes
number of paintings. 2. Pointed nose 3. The theme is concerned with the
Which of the statement(s) given above Buddha and Bodhisatva.
3. Double chin
is/are correct? Which of the statements given above are
Which of the statement(s) given above
(a) Only 1 (b) Only 2 correct?
is/are true?
(c) Both 1 and 2 (d) Neither 1 nor 2 (a) 1 and 2 (b) 2 and 3
(a) Only 1 (b) Only 2
(c) 1 and 3 (d) All of these
18. Match the following. (c) 2 and 3 (d) All of these
28. Consider the following statements
List I List II 23. Which one of the following statements
about Amir Khusrow.
A. Pandit Vishnu 1. Introduced raga is not correct? 1. He was disciple of Nizamuddin Auliya.
Digambar classification in (a) The statue of Gomateshwara at 2. He was the founder of both Hindustani
Paluskar Indian music Sravanabelagola represents the last classical music and qawwali.
B. Venkatamakhi 2. Proponent of the Tirthankara of Jains Which of the statement(s) given above
Carnatic music (b) India’s largest Buddhist monastery is is/are correct?
C. Syama Sastri 3. Proponent of the in Arunachal Pradesh
Khayal form of (a) Only 1 (b) Only 2
(c) The Khajuraho temples were built (c) Both 1 and 2 (d) Neither 1 nor 2
Hindustani music
under Chandela kings
D. Amir Khusrow 4. Composed the 29. Lord Buddha preached the following
music for song (d) The Hoysaleshwara temple is
dedicated to Shiva four Noble Truths. Put them in correct
Vande-Mataram
order.
Codes 24. Match the following. 1. There is suffering.
A B C D A B C D 2. There is cessation of suffering.
List I List II
(a) 4 1 3 2 (b) 4 1 2 3 3. There is a path leading to cessation of
(Artist) (Art)
(c) 1 4 3 2 (d) 1 4 2 3 suffering.
A. Hiren 1. Bharatnatyam
ART AND CULTURE Practice Questions

19. Consider the following statements 4. There is cause of suffering.


Bhattacharya dance
regarding Pala School of Paintings. B. Malini Rajurkar 2. Hindustani Select the correct answer using the codes
1. The main patrons of the school was vocal music given below.
Raja Dharmapala and Devapala. C. Pratibha Prahlad 3. Kuchipudi (a) 1, 4, 2 and 3 (b) 1, 4, 3 and 2
2. The main artist of this school were dance (c) 1, 3, 2 and 4 (d) 1, 2, 4 and 3
Dhimnan and Vidopala. D. Vempati Chinna 4. Puppetry
30. Consider the following statements.
Satyam
Which of the statement(s) given above 1. The art of miniature painting was
is/are correct? Codes introduced to India by the Mughals.
(a) Only 1 (b) Only 2 A B C D A B C D 2. In the 16th century, the Mughal ruler
(c) Both 1 and 2 (d) Neither 1 nor 2 (a) 4 2 1 3 (b) 3 1 2 4 Humayun brought artists from Persia,
(c) 4 1 2 3 (d) 3 2 1 4 who specialised in Miniature paintings.
20. Match the following.
25. Consider the following statements. Which of the statement(s) given is/are
List I List II correct?
1. The Dilwara Jain temple in Mount Abu
A. Dakshinaarka 1. Goalpara (Assam) constructed out of white marble. (a) Only 1 (b) Only 2
temple (c) Both 1 and 2 (d) Neither 1 nor 2
2. Dilwara Jain temple enshrines various
B. Surya Pahar 2. Arasavilli (Andhra Jain Tirthankars.
temple Pradesh) 31. The Sufi saint who maintained that
C. Suryanar 3. Kumbhkonam Which of the statement(s) given above devotional music was one way of
temple (Tamil Nadu) is/are correct? coming close to God was
D. Surya Narayan 4. Gaya (Bihar) (a) Only 1 (b) Only 2 (a) Mouinuddin Chishti
temple (c) Both 1 and 2 (d) Neither 1 nor 2
(b) Baba Farid
ART AND CULTURE 1-345

(c) Sayyid Muhammad Gesudaraz Which of the statement(s) given above 3. Women are mainly engaged in the
(d) Shah Alam Bukhari is/are correct? creation of warli folk paintings.

32. Nastaliq was (a) Only 1 (b) Only 2 Which of the statements given above are
(a) a Persian script used in Medieval India (c) Both 1 and 2 (d) Neither 1 nor 2 correct?
(b) a raga composed by Tansen 39. Consider the following statements. (a) 1 and 2 (b) 1 and 3
(c) a cess levied by the Mughal rulers (c) 2 and 3 (d) All of these
1. Malwa architecture is notable for the
(d) a type of puppetry excellent combination of arch and lintel 44. Consider the following statements.
construction of stairs of flight, the use of 1. The Brihadeeswara temple is situated
33. The Nagara, the Dravida and the Vesara coloured titles and lofty plinth for the
are at Tanjore, the ancient capital of the
buildings. Chola kings.
(a) the three main racial group of the 2. Malwa boasts of Hussain Shah’s Tomb,
Indian sub-continent 2. King Rajaraja Chola constructed this
Jama Masjid, Jahaz Mahal, Hindola temple in 10th century BC, designed by
(b) the three main linguistic division Mahal. the famous architect Sama Verma.
(c) the three main styles of Indian temple Which of the statement(s) given above
architecture
Which of the statement(s) given above
is/are correct? is/are correct?
(d) the three main musical gharanas (a) Only 1 (b) Only 2
prevalent in India (a) Only 1
(c) Both 1 and 2 (d) Neither 1 nor 2 (b) Only 2
34. Which of the following statements 40. Consider the following statements. (c) Both 1 and 2
is/are correct about Gupta architecture? (d) Neither 1 nor 2
1. Sittanavasal paintings is situated at
1. It widely used red sandstone Puddukota in Tanjore.
2. The background was made more 45. Consider the following statements.
2. These caves were built by Mahendra
refined and decorative 1. Pattachitra style of painting is one of the
Varman and his son Narasimha Varman.
oldest and most popular art forms of
3. Naked figures was mostly absent 3. These caves have Jain paintings. Odisha.
Select the correct answer using the codes Which of the statements given above are 2. Pattachitra is done on canvas and is
given below. correct? manifested by rich colourful
(a) Only 1 (b) 2 and 3 (a) 1 and 2 (b) 1 and 3 appreciation creative motifs and
(c) 1 and 2 (d) Only 2 (c) 2 and 3 (d) All of these design and portrayal of simple themes
mostly mythological in dipiction.
35. Which of the following were common 41. Consider the following statements
to both Buddhism and Jainism? Which of the statement(s) given above
regarding policy of Sulah-kul.
1. Avoidance of extremities in penance
is/are correct?
1. Sulah-kul means universal tolerance.
and enjoyment. (a) Only 1 (b) Only 2
2. The policy of Sulah-kul was state
2. Indifference to the authority of the Vedas. (c) Both 1 and 2 (d) Neither 1 nor 2
principle of Akbar.
3. Denial of efficacy of rituals. 46. Consider the following pairs.
3. Din-i-llahi was a result of the policy of
4. Non-injury to animal life. Sulah-kul policy. Tradition State
Codes 1. Gatka, a traditional : Kerala
Which of the statement(s) given above
(a) 1 and 2 (b) 2, 3 and 4 is/are correct? Martial Art
(c) 1, 3 and 4 (d) All of these (a) Only 1 (b) 1 and 2 2. Madhubani, a : Bihar
traditional painting
36. Which one of the following pairs is (c) 1 and 3 (d) All of these
3. Singhey Khabar : Jammu and
correctly matched? 42. Match the following.

ART AND CULTURE Practice Questions


Sindhu Darshan Kashmir
(a) Harappan : Painted Grey festival
List I List II
Civilisation Ware Which of the pair(s) given above is/are
(Famous Temple) (State)
(b) The Kushanas : Gandhara School correctly matched?
of Art A. Vidyashankara 1. Andhra
temple Pradesh (a) 1 and 2 (b) Only 3
(c) The Mughals : Ajanta Paintings
B. Rajarani temple 2. Karnataka (c) 2 and 3 (d) All of these
(d) The Marathas : Pahari School of
Painting C. Kandariya 3. Madhya 47. Match the following.
Mahadeva temple Pradesh
37. What does Budhayam theorem D. Bheemeshwara 4. Odisha List I List II
(Baudhayan Sulva Sutra) relate to? temple (Famous Person) (Known as)
(a) Lengths of sides of a right-angled A. Amrita Shergil 1. Dancer
triangle Codes
B. Bhimsen Joshi 2. Painter
(b) Calculation of the value of Pie A B C D A B C D
C. Rukmini Devi Arundale 3. Poet
(c) Logarithmic calculation (a) 2 4 3 1 (b) 2 3 4 1
D. Suryakant Tripathi 4. Singer
(d) Normal distribution curve (c) 1 3 2 4 (d) 3 2 1 4
Nirala
38. Consider the following statements. 43. Consider the following statements. Codes
1. Mohiniyattam is the female 1. Maharashtra is known for its warli folk A B C D
semi-classical dance form of Kerala. paintings. (a) 2 1 4 3
2. Warli is the name of the largest tribe (b) 2 4 1 3
2. Laterally, the dance of the enchantress,
found on the Northern outskirt of (c) 3 1 2 4
Mohiniyattam was mainly performed in
Mumbai. (d) 2 3 1 4
the temple premises of Kerala.
1-346 GSP General Studies (Paper 1)

48. Match the following. 3. Each part of the city was made up of 60. The tank of the Great Bath complex at
walled sections Mohenjo-daro had stairs on
List I List II
Select the correct answer using the codes 1. Northern side. 2. Southern side.
(Architect) (Monument)
given below. 3. Eastern side. 4. Western side.
A. Thomas 1. Gateway of India
(a) Only 3 (b) 1 and 2 Codes
Metcalfe
B. G Wittet 2. Victoria Hall (c) Only 2 (d) All of these (a) Only 1 (b) Only 3
C. William 3. Metcalfe House 53. With reference to Indian rock cut (c) 1 and 2 (d) 3 and 4
Emerson architecture, consider the following 61. Consider the following pairs.
D. Herbert 4. Secretariat and statements?
Baker Legislative Council Famous Work of
1. The caves at Badami are the oldest Reign
Building (Delhi) Architecture
rock-cut caves in India
1. Virupaksha temple Chalukyas
Codes 2. The kanheri caves marks the beginning of Badami
A B C D A B C D of a long tradition of colossal Buddha 2. Kailasnath temple of Ellora Mauryas
(a) 3 1 2 4 (b) 1 2 3 4 3. At Ellora, caves were made for different 3. Brihadeeshwar temple Pallavas
(c) 4 3 2 1 (d) 2 1 4 3 faiths
Select the correct answer using the codes Which of the pairs given above are
49. Match the following. correct?
given below.
List I List II (a) 1 and 3 (b) Only 3
(a) Only 1 (b) 2 and 3
(Dancer) (Dance) (c) Only 1 (d) 1 and 2
(c) Only 3 (d) All of these
A. Kalamandalam 1. Kathakali
54. The dance encouraged and performed 62. Consider the following statements
Kshemavathy
regarding Neo-Gothic style of
B. Kottakkal 2. Manipuri from the temple of Tanjore was
architecture.
Sivaraman (a) Bharatnatyam (b) Kathakali
1. Its characteristics included high
C. Lakshmi 3. Mohiniyattam (c) Odissi (d) Mohiniattam pitched roofs, pointed arches and
Viswanathan
55. Koodiyattam is a detailed decoration.
D. N Madhavi Devi 4. Bharatnatyam
(a) dress prepared in Goa 2. The Gothic style had its roots in
Codes (b) traditional dance of Kerala buildings especially, churches built in
A B C D A B C D (c) festival of Andhra Pradesh Northern Europe during the medieval
(a) 1 3 2 4 (b) 3 1 4 2 (d) folk dance of Tamil Nadu period.
(c) 1 3 2 4 (d) 4 3 1 2 Which of the statement(s) given above
56. Arrange the following monuments in a is/are correct?
50. Consider the following statements. chronological order (a) Only 1 (b) Only 2
1. The magnificent ‘Ratha’ cave temples of 1. Brihadeeswara temple, Tanjore
2. Draupadipath, Mamallapuram (c) Both 1 and 2 (d) Neither 1 nor 2
Mahabalipuram were built by the
Pallava king Narasimha in the 7th and 3. Kailasa temple, Ellora 63. Consider the following places.
8th centuries. 4. Meenakshi temple, Madurai 1. Ahiraura 2. Dhauli 3. Girnar
2. There are eight Rathas at Codes
At which of the above place/places do
Mahabalipuram out of which five are (a) 1, 4, 2, 3 (b) 2, 3, 1, 4
Ashoka's rock edicts occur?
named after the Pandavas of (c) 3, 1, 4, 2 (d) 4, 2, 3, 1 (a) Only 1 (b) 1 and 2
Mahabharata and one after Draupadi.
57. Amir Khusrow’s name is associated with (c) 2 and 3 (d) All of these
Which of the statement(s) given above the invention of which of the following
ART AND CULTURE Practice Questions

is/are correct? 64. Consider the following statements.


instrument?
(a) Only 1 (b) Only 2 1. The Hill Palace, the official residence of
(a) Sarod (b) Sitar
(c) Both 1 and 2 (d) Neither 1 nor 2 the Kochi Royal Family is the largest
(c) Veena (d) Shehnai
archaeological museum in Kerala.
51. Match the following. 58. Consider the following statements 2. It was built in 1835, the palace
List I List II regarding Kota paintings. complex consists of 69 buildings.
1. It excelled in portrayal of hunting
A. Tabla 1. Latif Khan Which of the statement(s) given above
scenes and elephant fight.
B. Sitar 2. Kartik Kumar is/are correct?
2. Scenes of worshipping darbar and (a) Only 1 (b) Only 2
C. Shehnai 3. Ali Hussain
processional scenes also formed the
D. Flute 4. Pannalal Ghosh subject matter of this style. (c) Both 1 and 2 (d) Neither 1 nor 2
Codes Which of the statement(s) given above 65. Which one of the following statements
A B C D A B C D is/are correct? is not correct?
(a) 2 3 1 4 (b) 4 3 1 2 (a) Only 1 (b) Only 2 (a) The role of Abul Fazal in the debates of
(c) 1 2 3 4 (d) 3 2 1 4 Ibadat Khana was important
(c) Both 1 and 2 (d) Neither 1 nor 2
52. Which of the following is/are correct (b) The religious debates of Ibadat Khana
about town planning during Harappa 59. With reference to the cultural history of did not play any role in the
civilisation? India, the term sarvatobhadra refers to development of religious view of Akbar
(a) A religious sect (c) The Ibadat Khana was opened for all
1. The streets were laid from east to west
in a grid pattern (b) A style of temple construction the religious leaders in the later period
(c) An assembly of Brahmins (d) The Ibadat Khana was established in
2. Buildings were made of standardised
baked bricks and stones (d) An administrative functionary 1575 AD
ART AND CULTURE 1-347

66. The Pashupati seal, discovered at 72. Which of the following are not part of 2. The Gateway of India built in the
Mohenjo-daro archaeological site by Qutub complex in Delhi? traditional Gujarati style is the most
Sir John Marshall, which has given rise 1. Alai Darwaza 2. Qutb Minar famous example of Indo-Saracenic
to the Proto-Shiva theory, has the style.
3. Mehrauli Pillar
following characteristics. Which of the statement(s) given above
4. Alauddin Khilji’s tomb
1. Yogic posture in possibly Hath Yoga. is/are correct?
2. Surrounded by animals. Select the correct answer using the codes (a) Only 1 (b) Only 2
3. Sitting on a lotus. given below?
(c) Both 1 and 2 (d) Neither 1 nor 2
(a) 1 and 4 (b) 1, 3 and 4
Select the correct answer using the codes 78. Consider the following statements with
given below. (c) 3 and 4 (d) None of these
reference to the Pallava architecture in
(a) Only 1 (b) 1 and 2 73. Consider the following statements. India.
(c) 2 and 3 (d) All of these 1. Sarod, a plucked instrument, it is a
1. It shows the transition from rock-cut
modification of rabeb.
67. One of the observatories established by architecture to stone built ones
Sawai Jai Singh was at 2. In Sarod, there are seven strings for 2. The sculptures of Dwarapalas at
(a) Agra (b) Indore playing the melody and ten strings for entrance of the mandapa was started
(c) Ujjain (d) Jodhpur resonating. by Pallavas
Which of the statement(s) given above 3. The tradition of direct patronisation of
68. Which of the following Harappan sites is/are correct?
are located in the Kutch region? temples began with Pallavas
(a) Only 1 (b) Only 2 Which of the statement(s) given above
1. Desalpur 2. Dholavira
3. Lothal 4. Rojdi (c) Both 1 and 2 (d) Neither 1 nor 2 is/are correct?
Select the correct answer using the codes 74. Consider the following statements. (a) Only 1 (b) 2 and 3
given below. 1. Raja Ravi Verma (of Travancore State), (c) 1 and 3 (d) All of these
(a) 1 and 3 (b) 3 and 4 a product of Madras Art School studied 79. Consider the following statements.
(c) 1, 2 and 3 (d) All of these oil paintings and specialised in
1. Abhigyan Shakuntalam which is
portraiture.
69. With reference to the Indo-Islamic considered as epitome of Sanskrit
2. He welded Royal Academy of
architecture, choose the correct drama by many scholars is based on
Naturalism with Indian myths and
the life of Sunga ruler Agnimitra.
statements. legends and created lively images of
1. The dome architecture was one of the Indian past. 2. Mrichchhkatikam clarifies the fact that
important characterstics of in ancient India the constraints of Varna
Which of the statement(s) given above system were not as rigid as is generally
Indo-Islamic architecture
is/are correct? perceived.
2. They introduced the art of inlay
(a) Only 1 (b) Only 2 Which of the statement(s) given is/are
decoration and challigraphy in India
3. Bricks were never used in the (c) Both 1 and 2 (d) Neither 1 nor 2 correct?
Indo-Islamic architecture 75. Which of the following classical works (a) Only 1 (b) Only 2
Select the correct answer using the codes of literature were written during the (c) Both 1 and 2 (d) Neither 1 nor 2
given below. Gupta era?
80. Consider the following, with reference
(a) 1 and 2 (b) Only 1 1. Amarkosh 2. Kamasutra
to new elements introduced by
(c) 1, 2 and 3 (d) 2 and 3 3. Maghduta 4. Mudrarakshasa
Mughals in Architecture.
Select the correct answer using the codes 1. Charbagh style by Babur.
70. Consider the following statements
regarding Suka-Saptati. given below. 2. Artificial sands and gardens.

ART AND CULTURE Practice Questions


1. Suka-Saptati is a book of folk tales, (a) 1 and 2 (b) 2 and 3
Which of the statement(s) given above
which has a collection of folk tales told (c) 1, 2 and 3 (d) All of these
is/are correct?
by a parrot. 76. With reference to the Ajanta caves, (a) Only 1 (b) Only 2
2. Its translation into Persian is called consider the following statements. (c) Both 1 and 2 (d) Neither 1 nor 2
Tutinamah.
1. At Ajanta, caves were made for 81. Consider the following statements.
Which of the statement(s) given above different faith
is/are correct? 1. Tughlaq influence on the Sharqi
2. It has both Chaityas and Viharas architecture is seen in the use of arch
(a) Only 1 (b) Only 2
3. Seated Buddha in and beam and battering walls.
(c) Both 1 and 2 (d) Neither 1 nor 2 Dharmachakrapravartana is a notable 2. Propylones have been used in Atala
71. Consider the following statements. sculpture here Masjid and Jama Masjid.
1. Dashavatar is the most developed Select the correct answer using the codes Which of the statement(s) given above
theatre form of the Konkan and Goa given below. is/are correct?
regions. (a) 1 and 3 (b) Only 2 (a) Only 1 (b) Only 2
2. The performers personify the ten (c) 2 and 3 (d) 1 and 2 (c) Both 1 and 2 (d) Neither 1 nor 2
incarnations of Lord Vishnu.
77. Consider the following statements. 82. Which of the following performing arts
Which of the statement(s) given above is associated with Sumyukta Panigrahi?
1. Indo-Saracenic style of architecture
is/are correct? (a) Painting (b) Violin
combined the Indian architecture with
(a) Only 1 (b) Only 2 European. (c) Dance (d) Sports
(c) Both 1 and 2 (d) Neither 1 nor 2
1-348 GSP General Studies (Paper 1)

83. Consider the following statements 3. Metallurgy had also made a wonderful Which of the statements given above are
about Indus Valley Civilisation. progress during the Gupta period. correct?
1. The layouts of Harappan cities were The craftsmen were efficient in the art (a) 1 and 2 (b) 2 and 3
much planned and the streets were of casting metal statues and pillars. (c) 1 and 3 (d) All of these
narrow. Which of the statement(s) given above
91. Consider the following statements about
2. The houses of the earlier settlements is/are correct?
North-Indian music forms.
were with an upper storey, made of mud (a) Only 1 (b) 1 and 3
and baked brick in standardised sizes. 1. Ghoomar is basically a community
(c) 1 and 2 (d) All of these dance for males and performed during
3. The granaries and the Great Bath at
87. Consider the following statements marriages.
Mohenjo-daro seem for special rituals.
about Hoysala architecture. 2. Dhumhal is mainly the men folk of Wattal,
Which of the statement(s) given above who perform this dance on specific
1. The most notable temples of this
is/are correct? occasions.
period are those constructed in Belur,
(a) 1 and 2 (b) 2 and 3 3. Dhamyal is the folk dance of Haryana
Halebidu and Somnathpur.
(c) All of these (d) None of these can be performed by men alone as well
2. The Kirtinarayan temple was built by
84. Consider the following statements as with women.
Hoysala King Vishnuvardhana.
about stupas. 3. Chennakesava temple is located in Which of the statements given above are
1. Buddha’s relics were placed in a casket Belur and was built by correct?
chamber in the one of the pillar of the Vishnuvardhana of the Hoysala (a) 1 and 2 (b) 2 and 3
Sanchi stupa. dynasty in 1117 AD to commemorate (c) 1 and 3 (d) All of these
2. It is presumed that the elaborately a victory won over the Chalukyas.
92. Consider the following statements about
carved Toranas were built by ivory or Which of the statement(s) given above
Religious Movement.
metal workers in the 1st century BC is/are correct?
during the reign of King Satakarni of the (a) Only 1 (b) 1 and 3 1. A sutra in Panini’s Ashtadhyayi refers to
Satavahana dynasty. the worshippers of Shiva.
(c) 1 and 2 (d) All of these
3. The last addition to the stupa was made 2. The Shaiva Movement in the South
88. Consider the following statements flourished at the beginning through the
during the reign of Harsha when four
regarding sankhya school. activities of many of 63 saints known in
images of Buddha sitting in the dhyana
1. It strongly propounds dualism Tamil as Alvars.
mudra or meditation were installed at
the four entrances. 2. Sankhya philosophy does not believe 3. Vaishnava poet saints known as alvars
in the existence of god preached single-minded devotion
Which of the statement(s) given above 3. Sankhya does not accept the theory (Ekatmika Bhakti) for Vishnu and their
is/are correct? of rebirth and transmigration of soul songs were collectively known as
(a) Only 1 (b) Only 2 Which of the statements given is/are Prabandhas.
(c) 2 and 3 (d) All of these correct? Which of the statement(s) given is/are
85. Consider the following statements (a) 1and 2 (b) 2 and 3 correct?
about Mauryan period. (c) 1 and 3 (d) All of these (a) 1 and 2 (b) Only 3
1. It is said that one of the caves in the 89. Consider the following statements (c) Only 2 (d) All of these
Barabar hills called the Sudama Cave about Indian folk music. 93. Consider the following statements about
was dedicated by Ashoka to the Ajivika 1. The female dance of Punjab region is religious tradition.
monks of the Ajivika sect. known as Giddha. 1. In Christianity, Trinity refers to the
2. The monolithic rail at Sarnath in grey 2. Lavani is a popular folk form of teaching that God comprises three
and polished Chunar sandstone has Maharashtra and the dance format distinct persons i.e. the Father, the Son
ART AND CULTURE Practice Questions

been erected under the patronage of associated with it known as Tamasha. and the Holy Spirit.
Emperor Chandragupta Mauryan 3. Naatupura Patu is a Tamil folk music 2. Sufism originated in India and found a
himself. consists of Gramathisai (village folk congenial atmosphere in Iran and
3. It has rightly been said that Ashoka music) and Gana (city folk music), Afghanistan.
inaugurated a style of architecture also sung in Rajasthan.
3. The key scriptures of Judaism are
which spread in different parts of the Which of the statement(s) given above Kitab-i-Aqdas and Kitab-i-lqan.
country and itself at its best in the is/are correct?
magnificent masterpiece of Karla, Which of the statement(s) given above
(a) Only 1 (b) 1 and 3
Ajanta, Ellora and Elephanta. is/are correct?
(c) 1 and 2 (d) All of these
Which of the statement(s) given above (a) Only 1 (b) Only 2
is/are correct? 90. Consider the following statements (c) 1 and 2 (d) All of these
(a) Only 1 (b) 1 and 3
about Indian Classical music.
1. Mohiniyattam has the grace and 94. Consider the following statements about
(c) 1 and 2 (d) All of these architect of Delhi.
elegance of Bharatnatyam and vigour
86. Consider the following statements of Kathakali. 1. The British Viceroy made Sir Edward
about Gupta Art. 2. In Odissi the tribhanga (three bends) Lutyens responsible for the overall plan
1. Both the Nagara and Dravidian styles of symbolise the means to escape the of Delhi. He was specifically directed to
art evolved during this period. limitations of the body. harmonise externally with the traditions
3. Sattriya dance is classified into two of Indian art.
2. The temple at Deogarh near Jhansi and
the sculptures in the temple at Garhwa style namely—Paurashik Bhangi i.e. 2. Another Englishman called Robert Tor
tandav or masculine style and Shtri Russell built the Connaught Place and
near Allahabad remain important
Bhangi i.e. lasya or feminine style. the Eastern and Western Courts.
specimen of the Gupta Art.
ART AND CULTURE 1-349

3. Herbert Baker added the imposing Which of the statements given above are Which of the statements given above
buildings of the South Block and the correct? is/are correct?
North Block, which flank the Rashtrapati (a) 1 and 3 (d) 2 and 3 (a) Only 1 (b) Only 2
Bhawan.
(c) 1 and 2 (d) All of these (c) Both 1 and 2 (d) Neither 1 nor 2
Which of the statement(s) given above
is/are correct? 98. Consider the following statements about 102. Consider the following statements
(a) Only 1
Nayaka Architecture. regarding the architectural heritage
1. Nayaka architectural style was of India.
(b) Only 2
characterised by elaborate hundred and 1. Vitthala temple with the 56 musical
(c) 1 and 2
thousand-pillared mandapas, the high pillars is located in Tamil Nadu.
(d) All of the above gopurams with painted stucco statues on 2. The Chola Age Shiva temple located
95. Consider the following statements about the surface and long corridors. in Tanjore, is granite building.
Indian architect. 2. The Nayaka dynasty is also known for its 3. The Sun temple at Konark was built
mural and wall paintings. Tanjore during Ashoka’s reign.
1. The Jantar Mantar, built by Jai Singh II in
paintings from this time are known for
the early 18th century, is an interesting Which of the statement(s) given
their surface richness, vivid colours and
example of Rajput architecture. above is/are correct?
compact composition.
2. The palaces of Jaisalmer, Bikaner, (a) 1 and 2 (b) Only 2
3. The Nayaka dynasties emerged in South
Jodhpur, Udaipur and Kota represent the
India after the downfall of the (c) 2 and 3 (d) Only 3
maturity of the Rajput style.
Vijayanagara empire in 1565.
3. The most spectacular of all Jain temples 103. Consider the following statements
Which of the statements given above are regarding some of the concepts in
are found at Ranakpur and Mount Abu in
Rajasthan, important specimen of Rajput correct? Indian Classical Music.
style. (a) 1 and 3 (b) 2 and 3 1. Swara refers to the notes on the
Which of the statement(s) given above (c) 1 and 2 (d) All of these musical scale.
is/are correct? 99. Consider the following statements about 2. Raga refers to the melody in the
(a) Only 1 (b) Only 2 Odishan architecture. music.
(c) 1 and 2 (d) All of these 1. The temples of Odisha are of the 3. Indian classical music is the same as
Indo-Aryan style, which is distinct from mode in Western music.
96. Consider the following statements about
the South Indian style. Which of the statement(s) given
Indian architect.
2. The temples consist of the main temple above is/are correct?
1. The Qutub Minar, built to commemorate
or shrine and the frontal porch. The main (a) Only 1 (b) 1 and 2
the entry of Islam, was essentially a
temple, known as Vimana or Deula, is (c) Only 2 (d) 1 and 3
victory tower, decorated with several
the sanctum enshrining the deity.
calligraphic inscriptions. 104. Which of the following is/are not the
3. The porch or Jagamohana is a
2. Well decorated Alai Darwaza, which characteristics of the Kashmir style
congregation place for the devotees.
served as an entrance gateway to the of architecture?
mosque at the Qutub complex, marks Which of the statements given above are
1. It primarily uses granite as building
the evolution of another innovative correct? material.
feature in the Indo-lslamic architecture, (a) 1 and 3 (b) 2 and 3 2. The Mughals tried introduce wood
built by Qutub-ud-din Aibak. (c) 1 and 2 (d) All of these based architecture in Kashmir.
3. One of the salient feature of Indo-lslamic 3. It has pyramidal roofs.
100. Consider the following statements about
style of architecture is arched gateways
Temple architecture. Select the correct answer using the
made beautiful with geometric,

ART AND CULTURE Practice Questions


arabesque designs and calligraphic 1. Nagara has a curvilinear tower while codes given below.
inscriptions. Dravidian has its pyramidal. (a) 1 and 2
Which of the statements given above are 2. Pillars play an important part in the (b) Only 2
correct? Nagara style and not so important role in (c) Only 3
the Dravidian style. (d) 2 and 3
(a) 1 and 3 (b) 2 and 3
3. Nagara temples do not have enclosures
(c) 1 and 2 (d) All of these 105. Consider the following statements
while Dravidian temples do have.
97. Consider the following statements about regarding the structure of the
Which of the statement(s) given above
Rajasthani architecture. Mauryan pillars.
is/are correct? 1. The shaft of the pillar is cylindrical.
1. Rajasthan architecture arouses from (a) Only 1 (b) 1 and 3
Rajput School of Architecture, a perfect 2. The crowning animal is always a lion
(c) Only 2 (d) Only 3 in different forms.
mixture of Hindu and Mughal structural
pattern. 101. With reference to Buddhism in 3. The pedestal of the crowning animal
ancient India, consider the following is always circular.
2. The artistic builders of Rajput designed
major architectural styles which are statements. Which of the statement(s) given
located in cities like Jaisalmer, Udaipur, 1. According to Buddhism, the world is above is/are correct?
Jaipur and Jodhpur. transient and soulless. (a) Only 1
3. The Rajasthan architecture significantly 2. Buddhists does not believe in the (b) 1 and 2
depends upon Rajput architecture concept of moksha but preached to (c) Only 3
school which was mixture of Deccan and attain nirvana (d) None of the above
Hindu structural design.
1-350 GSP General Studies (Paper 1)

106. With regard to the dance form of Indian 3. The relief work on these paintings gives 111. Consider the following statements
known as Sattriya, consider the them a three-dimensional effect. regarding the Manipuri dance
following statements. Which of the statement(s) given above 1. Unlike the classical dances, Manipuri
1. It is a classical dance form. is/are correct? does not emphasise on the sensuous at
2. This dance form is associated with the (a) Only 1 (b) Only 2 all.
Shaivite tradition. (c) 1 and 2 (d) 2 and 3 2. Rabindranath Tagore played an important
3. It was introduced by the saint role in popularising the dance.
Sankaradeva. 109. With reference to Jatra, a performing
3. The dancers wear ankle bells during the
Which of the statements given above are art form, consider the following
performance.
correct? statements.
1. It is an art form popular in Gujarat Which of the statement(s) given above
(a) 1 and 2 (b) 2 and 3 is/are correct?
2. It is performed in religious ceremonies
(c) 1 and 3 (d) All of these (a) 1 and 2
in honour of gods
107. How is the Mughal style of painting 3. Earlier form of Jatra has been musical (b) Only 2
different from the Rajput style of but dialogues were added at a later (c) 2 and 3
painting? stage (d) All of the above
1. Mughal painting was secular while Which of the statement(s) given above 112. Which of the following statements
Rajput painting was religious. is/are correct? regarding the Mughal architecture
2. Mughal painting depicted nature while
(a) Only 1 (b) Only 2 under various Mughal emperors is/are
Rajput painting showed royal pomp
and show. (c) Both 1 and 2 (d) Both 2 and 3 correct?
3. Rajput paintings depicted the theme of 110. Identify the classical dance form using 1. The architecture under Shah Jahan
love while Mughal paintings avoided it. was inferior in grandeur as compared
its characteristics that have been
to Akbar’s period.
Select the correct answer using the codes mentioned below.
given below. 2. The least number of buildings were
1. It has the tribhanga posture.
built during Aurangzeb’s period.
(a) Only 1 (b) 1 and 3 2. Kiran Sehgal was its popular dancer.
3. Akbar borrowed architectural elements
(c) Only 3 (d) All of these 3. It used to be danced by young boys from the Rajputs.
108. With regard to the Tanjore style of known as Gotipuas in temples.
4. It was Babur who brought Sinan from
painting in India, examine the 4. Indrani Rehman brought the dance to Persia to construct buildings in India.
following statements. prominence.
Select the correct answer using the codes
1. The paintings of Shiva and Parvati in Which of the following option is correct? given below.
different poses are the most popular (a) Kathakali (a) 1, 2 and 3
themes of this style of painting. (b) Odissi (b) 2 and 4
2. The paintings are decorated by (c) Mohiniattam (c) 3 and 4
semi-precious stones, pearls and glass (d) Kathak
pieces. (d) 1, 3 and 4

Answers
1. (c) 2. (d) 3. (c) 4. (c) 5. (d) 6. (a) 7. (d) 8. (c) 9. (c) 10. (c)
11. (a) 12. (c) 13. (a) 14. (d) 15. (a) 16. (a) 17. (c) 18. (b) 19. (c) 20. (c)
ART AND CULTURE Practice Questions

21. (b) 22. (d) 23. (a) 24. (a) 25. (c) 26. (c) 27. (d) 28. (a) 29. (a) 30. (b)
31. (b) 32. (a) 33. (c) 34. (b) 35. (b) 36. (b) 37. (a) 38. (c) 39. (c) 40. (d)
41. (d) 42. (a) 43. (d) 44. (c) 45. (c) 46. (c) 47. (b) 48. (a) 49. (b) 50. (c)
51. (c) 52. (b) 53. (b) 54. (a) 55. (b) 56. (b) 57. (b) 58. (c) 59. (b) 60. (c)
61. (c) 62. (c) 63. (d) 64. (a) 65. (b) 66. (b) 67. (c) 68. (c) 69. (a) 70. (c)
71. (c) 72. (d) 73. (c) 74. (c) 75. (d) 76. (c) 77. (c) 78. (d) 79. (b) 80. (c)
81. (c) 82. (c) 83. (b) 84. (d) 85. (b) 86. (d) 87. (c) 88. (a) 89. (d) 90. (d)
91. (b) 92. (b) 93. (a) 94. (d) 95. (c) 96. (a) 97. (c) 98. (d) 99. (d) 100. (b)
101. (c) 102. (b) 103. (b) 104. (a) 105. (d) 106. (c) 107. (b) 108. (d) 109. (d) 110. (b)
111. (a) 112. (d)
ART AND CULTURE 1-351

SET 2 UPSC QUESTIONS (2017-2006)


2017 Select the correct answer using the (b) It is an ancient style bronze and
codes given below. brasswork still found in Southern part
1. The painting of Bodhisattva Padmapani (a) Only 1 (b) 2 and 3 of Coromandel area
is one of the most famous and oft- (c) 1 and 3 (d) 1, 2 and 3 (c) It is an ancient form of dance drama
illustrated paintings at and a living tradition in the Northern
7. Consider the following pairs.
(a) Ajanta (b) Badami part of Malabar
Place of
(c) Bagh (d) Ellora Location (d) It is an ancient martial art and a living
Pilgrimage
tradition in some parts of South India
2. Consider the following pairs. 1. Srisailam Nallamala Hills
2. Omkareshwar Satmala Hills 12. Consider the following pairs.
Traditions Communities
3. Pushkar Mahadeo Hills 1. Garba : Gujarat
1. Chaliha Sahib Sindhis
Festival 2. Mohiniattam : Odisha
Which of the above pairs is/are correctly
2. Nanda Raj Jaat Gonds matched? 3. Yakshagana : Karnataka
Yatra (a) Only 1 (b) 2 and 3 Which of the pair (s) given above is/are
3. Wari-Warkari Santhals correctly matched?
(c) 1 and 3 (d) 1, 2 and 3
Which of the pairs given above is/are (a) Only 1 (b) 2 and 3
8. With reference to the art and
correctly matched? archaeological history of India, which (c) 1 and 3 (d) 1, 2 and 3
(a) Only 1 (b) 2 and 3 one among the following was made 13. With reference to Buddhist history,
(c) 1 and 3 (d) None of these earliest? tradition and culture in India, consider
3. Which of the following is/are famous (a) Lingaraja Temple at Bhubaneswar of the following pairs.
for Sun temples? (b) Rock-cut Elephant at Dhauli Famous Shrine Location
1. Arasavalli 2. Amarakantak (c) Rock-cut Monuments at 1. Tabo monastery and : Spiti valley
3. Omkareshwar Mahabalipuram temple complex
Select the correct answer using the code (d) Varaha Image at Udayagiri 2. Lhotsava Lhakhang : Zanskar
given below. temple, Nako valley
9. In a particular region in India, the local
(a) Only 1 (b) 2 and 3 3. Alchi temple complex : Ladakh
people train the roots of living trees
(c) 1 and 3 (d) 1, 2 and 3 into robust bridges across the streams. Which of the pair(s) given above is/are
As the time passes, these bridges correctly matched?
become stronger. These unique ‘living
2016 root bridges’ are found in
(a) Only 1 (b) 2 and 3
(c) 1 and 3 (d) 1, 2 and 3
(a) Meghalaya
4. With reference to the cultural history 14. With reference to the Indian history of
of India, the memorising of chronicles, (b) Himachal Pradesh
art and culture, consider the following
dynastic histories and epic tales (c) Jharkhand
pairs.
was the profession of who of the (d) Tamil Nadu
Famous Work
following? Site
of Sculpture
(a) Shramana (b) Parivraajaka 2014 1. A grand image of : Ajanta

ART AND CULTURE Practice Questions


(c) Agrahaarika (d) Magadha Buddha’s
10. With reference to the famous Sattriya Mahaparinirvana with
dance, consider the following numerous celestial
2015 statements. musicians above and the
1. Sattriya is a combination of music, sorrowful figures of his
5. Kalamkari painting refers to followers below
dance and drama.
(a) a hand-painted cotton textile in South 2. A huge image of Varaha : Mount
India 2. It is centuries-old living tradition of
Avatar (boar incarnation) Abu
(b) a handmade drawing on bamboo Vaishnavites of Assam.
of Vishnu, as he rescues
handicrafts in North-East India 3. It is based on classical Ragas and Goddess Earth from the
(c) a block-painted woollen cloth in Talas of devotional songs composed deep and chaotic waters,
Western Himalayan region of India by Tulsidas, Kabir and Mirabai. sculpted on rock
(d) a hand-painted decorative silk cloth in Which of the statement(s) given above 3. ‘Arjuna’s Penance’/ : Mamalla-
North-Western India is/are correct? Descent of Ganga puram
(a) Only 1 (b) 1 and 2 sculpted on the surface of
6. Which of the following has/have been huge boulders
accorded ‘Geographical Indication’ (c) 2 and 3 (d) 1, 2 and 3
status? 11. With reference of India’s culture and Which of the pair(s) given above is/are
1. Banaras Brocades and Sarees tradition, what is ‘Kalaripayattu’? correctly matched?
2. Rajasthani Daal-Bati-Churma (a) It is an ancient Bhakti cult of Shaivism (a) 1 and 2 (b) Only 3
still prevalent in some parts of South (c) 1 and 3 (d) 1, 2 and 3
3. Tirupathi Laddu
India
1-352 GSP General Studies (Paper 1)

15. A community of people called Mangan (b) Chaitya is a place of worship, while Select the correct answer using the
iyars is well-known for their Vihara is the dwelling place of the codes given below.
(a) martial arts in North-East India monks (a) 1 and 2 (b) 2 and 3
(b) musical tradition in North-West India (c) Chaitya is the stupa at the far end of the (c) 1, 2 and 3 (d) None of these
(c) classical vocal music in South India cave, while Vihara is the hall axial to it
22. How do you distinguish between
(d) pietra dura tradition in Central India (d) There is no material difference
Kuchipudi and Bharatanatyam dances?
between the two
1. Dancers occasionally speaking
2013 19. Consider the following historical places dialogues is found in Kuchipudi dance
1. Ajanta Caves but not in Bharatanatyam.
16. With reference to the history of Indian 2. Lepakshi Temple 2. Dancing on the brass plate by keeping
rock-cut architecture, consider the 3. Sanchi Stupa the feet on its edges is a feature of
following statements Which of the above place(s) is/are also Bharatanatyam but Kuchipudi dance
1. The caves at Badami are the oldest known for mural paintings? does not have such a form of
surviving rock-cut caves in India. (a) Only 1 (b) 1 and 2 movements.
2. The Barabar rock-cut caves were (c) All of these Which of the statement(s) given above
originally made for Ajivikas by emperor (d) None of these is/are correct?
Chandragupta Maurya. (a) Only 1 (b) Only 2
20. In the context of cultural history of
3. At Ellora, caves were made for different (c) 1 and 2 (d) Neither 1 nor 2
India, a pose in dance and dramatics
faiths. called Tribhanga’ has been a favourite 23. With reference to the scientific
Which of the statement(s) given above of Indian artists from ancient times till progress of ancient India, which of the
is/are correct? today. Which one of the following statements given below are correct?
(a) Only 1 (b) 2 and 3 statements best describes this pose? 1. Different kinds of specialised surgical
(c) Only 3 (d) All of these (a) One leg is bent and the body is slightly instruments were in common use by
but oppositely curved at waist and AD 1st century.
17. The Chinese traveller Yuan Chwang
(Hiuen Tsang), who visited India neck 2. Transplant of internal organs in the
recorded the general conditions and (b) Facial expressions, had gestures and human body had begun by the
make-up are combined to symbolise beginning of AD 3rd century.
culture of Indian at that time. In this
context, which of the following certain epic or historic characters 3. The concept of sine of an angle was
statements is/are correct? (c) Movements of body, face and hands known in AD 5th century.
1. The roads and river-routes were are used to express oneself or to tell a 4. The concept of cyclic quadrilaterals
completely immune from robbery. story was known in AD 7th century.
2. As regards punishment for (d) A little smile, slightly curved waist and Select the correct answer using the
offences, ordeals by fire, water and certain hand gestures are emphasised codes given below.
poison were the instruments for to express the feelings of love or (a) 1 and 2 (b) 3 and 4
determining the innocence or guilt of eroticism (c) 1, 3 and 4 (d) 1, 2, 3 and 4
a person.
24. With reference to the history of ancient
3. The tradesmen had to pay duties at 2012 India, which of the following was/were
ferries and barrier stations. common to both Buddhism and
Select the correct answer using the 21. With reference to Dhrupad, one of the Jainism?
codes given below. major traditions of India that has been 1. Avoidance of extremities of penance
ART AND CULTURE Practice Questions

(a) Only 1 (b) 2 and 3 kept alive for centuries, which of the and enjoyment.
(c) 1 and 3 (d) All of these following statements are correct? 2. Indifference to the authority of the
18. Some Buddhist rock-cut caves are 1. Dhrupad originated and developed in Vedas.
called Chaityas, while the others are the Rajput kingdoms during the 3. Denial of efficacy of rituals.
called Viharas. What is the difference Mughal period.
Select the correct answer using the
between the two? 2. Dhrupad is primarily a devotional and
codes given below
(a) Vihara is a place of worship, while spiritual music.
(a) Only 1 (b) 2 and 3
Chaitya is the dwelling place of the 3. Dhrupad Alap uses Sanskrit syllables
(c) 1 and 3 (d) 1, 2 and 3
monks from Mantras.

Answers
1. (a) 2. (a) 3. (a) 4. (d) 5. (a) 6. (c) 7. (a) 8. (b) 9. (a) 10. (b)
11. (d) 12. (c) 13. (c) 14. (c) 15. (b) 16. (c) 17. (b) 18. (b) 19. (b) 20. (a)
21. (b) 22. (a) 23. (c) 24. (b)

You might also like